1-MANTIK Terim, Tanımsız Terim Önermeler

advertisement
1-MANTIK
Günümüzde, her bir bilim dalı ile ilgili bilgiler hızla artmakta olup bu bilgilerin
sistemli biçimde öğrenilmesi ve öğretilmesi gerekmektedir.
Matematik, doğru ve sistemli (Mantıklı) düĢünebilme alıĢkanlığı kazandırır.
Matematiğin temeli de mantığa dayanır. Mantık matematiğin dilini kurar, ona anlam ve
biçim kazandırır. Örneğin, insanlara faydası tartıĢılmayacak kadar fazla olan
bilgisayarların çalıĢma sistemi mantıksal kurallara göredir.
Terim, Tanımsız Terim
Bir bilim dalında devamlı kullanılan sözcükler vardır. ´küp’ sözcüğünün
matematik teki anlamı farklı konuĢma dilindeki anlamı farklıdır. Açı, çember, yay,
küme,…matematiksel terimlerdir. Dağ, ova, yayla, iklim,… coğrafik terimlerdir. Aynı
Ģekilde kütle, yerçekimi, ivme,…gibi sözcüklerde fiziksel terimleridir.
Bir terimin anlamını belirtmeye terimi tanımlamak denir.Herhangi bir terimi
tanımlamak için baĢka terimlere ihtiyaç duyarız. Ancak her bilim dalında
tanımlanamayan bazı terimler vardır. Bunlara tanımsız terimler denir.
Önermeler
Günlük yaĢantımızda kullandığımız cümlelerin belirttiği hükümlerin bazılarına
doğru, bazılarına ise yanlıĢ deriz. Bazı cümleler ne yanlıĢ nede doğrudur.
 Bir hafta yedi gündür. ( Doğru )
 Ay, Dünya dan büyüktür. ( YanlıĢ )
 Sınıfa giriniz. ( Ne yanlıĢ ve nede doğru )
Tanım
Bir cümlenin belirttiği hüküm kesinlikle doğru yada yanlıĢ ise , bu cümleye
önerme denir.
Önermeler p, q, r, s,... gibi harflerle gösterilir.
Örnek :
1.
2.
3.
4.
5.
6.
AĢağıdaki ifadelerin önerme olup olmadığına baklaım.
5  4  9 ( Doğru önerme)
6  5 ( YanlıĢ önerme)
Okula gidelim. ( Önerme değil )
Yeni mi aldın. ( Önerme değil )
9 asaldır. ( YanlıĢ önerme)
TBMM 23 Nisan 1920 yılında açıldı. ( Doğru önerme)
1
Tanım
Bir önermenin doğru yada yanlıĢ oluĢuna, o önermenin doğruluk değeri denir.
Bir önermenin doğruluk değeri için iki durum vardır.
p
Doğru
(D)
YanlıĢ
( Y)
Bu tabloyu daha kullanıĢlı hale getirmek için doğru yerine 1 ve yanlıĢ yerine 0
rakamları kullanılır.
p
Doğru
1
YanlıĢ
0
Burada 1 ve 0 ın sayısal bir değeri yoktur. Yalnızca önermenin doğruluk değerini
gösterir.
 Ay, Dünyanın uydusudur. Doğruluk değeri 1,
 Ġstanbul, Türkiye nin baĢkentidir. Doğruluk değeri 0 dır.
ġimdi de , iki önermenin doğruluk değerlerinin nasıl olacağına bakalım.
- p doğru , q doğru
- p doğru , q yanlıĢ
- p yanlıĢ , q doğru
- p yanlıĢ , q yanlıĢ
Bu durumu aĢağıdaki gibi belirtebiliriz.
p
q
1
1
0
1
0
0
Ġki önermenin doğruluk değeri için dört farklı durum ve üç önermenin doğruluk
değeri için 8 farklı durum söz konusudur.
2
p
1
1
0
0
p
1
1
1
1
0
0
0
0
q
1
0
1
0
q
1
1
0
0
1
1
0
0
r
1
0
1
0
1
0
1
0
n - tane önerme için 2n tane durum vardır.
EĢdeğer ( DENK ) Önermeler
Doğruluk değeri aynı olan iki önermeye eĢdeğer ( denk ) önermeler denir. p ile q
denk iki önerme ise, bunu p  q biçiminde yazarız. Ve
'' p denktir q '' veya '' p eşdeğerdir q '' Ģeklinde okuruz.
Örnek :
p : 23  8
q : 14  12
r : Ankara Ege Bölgesindedir.
s : Kızılırmak, Karadenize dökülür.
Önermelerini ele alalım. Burada, p  1, q  0 , r  0 , s  1 dir . Bu tanıma göre
p  s , q  r yazılır.
Bu önermelerde p ve q nun doğruluk değerleri aynı olmadığından p  q
biçiminden yazarız.
Bir Önermenin Olumsuzu ( Değili )
Tanım
Bir önermenin hükmünün değiĢtirilmesiyle elde edilen yeni önermeye ilk
önermenin olumsuzu ( değili ) denir.
3
Bir p önermesinin olumsuzu veya değili p , p veya
biri ile gösterilir. Bizim tercihimiz p olacaktır.
p sembollerinden
p
YanlıĢ
Doğru
p
Doğru
YanlıĢ
Veya
p
0
1
p
1
0
Örnekler :
1  p : '' Ülkemizde yaz aylarında havalar çok sıcak olur.''
p önermesinin olumsuzu ,
p : '' Ülkemizde yaz aylarında havalar çok sıcak olmaz.'' önermesidir.
2  r : 3  7  10 önermesinin olumsuzu r :  3  7  10   3  7  10 
3  s :15  12 önermesinin olumsuzu
s : 15  12   15  12 
Bir p önermesinin değilinin değili yine kendisidir. Yani,  p   p biçiminde ifade
ederiz.
p
1
0
p : '' Sigara sağlığa zararlıdır.''
p : '' Sigara sağlığa zararlı değildir.''
( p) : '' Sigara sağlığa zararlıdır.''
p
0
1
( p)
1
1
Yani, ( p)  p dir.
Örnekler :
1- AĢağıda verilen terimlerden tanımlı ve tanımsız olanları belirtelim.
a) Açı
b) Çember c)Doğru
d) Düzlem
e) Nokta
Çözüm :
Açı, Çember ve Düzlem tanımlı olup, Doğru ve Nokta tanımsız terimdir.
2- AĢağıdaki ifadelerin önerme olup olmadığına bakınız.
4
a) 1 ton 1000 kg dır.
b) Tuz Gölü Akdeniz Bölgesindedir.
c) Yarın yağmur yağar mı?
d) Lütfen sessiz olunuz.
e) Barometre sıcaklık ölçen bir alettir.
f) Kare özel bir dikdörtgen dir.
Çözüm .
a,b,e,f Ģıkları bir önermedir. c,d ise kesin bir hüküm belirtmediğinden önerme değilidir.
3- AĢağıdaki önermelerin doğruluk değerlerini söyleyiniz.
a) (2)3  8
b)  42  16
c) (4) 2  16
d )13  13
e) Karenin dört kenar uzunluğu eĢittir.
Çözüm :
a) 1
b) 0
c) 1
d) 0
e) 1
4- AĢağıdaki önermelerin denk olanlarını söyleyiniz ve yazınız.
p : 3  9  12 , q : 24  3 , r : 23  6 , s : 42  4  4
Çözüm :
p  0 , q  1 , r  0 , s :1 olduğuna göre p  r , q  s dir
5- AĢağıdaki önermelerin olumsuzunu söyleyiniz ve yazınız.
a) 1 hm 100 m eĢittir.
b) Salıdan sonra PerĢembe gelir.
c) Hakan, çalıĢkan bir öğrencidir.
d) 16  4  18
e) 6  8
Çözüm :
a) 1 hm 100 m eĢit değildir.
b) Salıdan sonra PerĢembe gelmez.
c) Hakan, çalıĢkan bir öğrenci değildir.
d) 16  4  18
e) 6  8
5
BileĢik Önermeler
Ġki yada daha fazla önermenin veya, ve, ise, ancak ve ancak gibi birer
önerme iĢlemi olan bağlaçlarla bağlanmasıyla oluĢan yeni önermeye bileĢik önerme
denir. Kolaylık olsun diye bunları birer sembolle gösteririz.
Bağlacın Adı
Simgesi
Veya

Ve

Ġse


ancak ve ancak
Tanım :
p ile q önermelerinden oluĢan '' p  q '' bileĢik önermesi, bileĢenlerden en az biri
doğru iken doğru, bileĢenlerden ikiside yanlıĢ ise yanlıĢtır. '' p  q '' nun doğruluk
tablosunu yapalım.
p
pq
q
1
1
1
1
0
1
0
1
1
0
0
0
1  1  1 ,1  0  1 ,0  1  1 ,0  0  0 olarak yazarız.
Ayrıca, p  1  1 , p  0  p , p  p  0 denklikleri söylenebilir.
Tanım :
p ile q önermelerinden oluĢan '' p  q '' bileĢik önermesi, bileĢenlerden her ikiside
doğru iken doğru, bileĢenlerden biri yanlıĢ ise yanlıĢtır. '' p  q '' nun doğruluk
tablosunu yapalım.
p
1
1
0
0
q
1
0
1
0
pq
1
0
0
0
1  1  1 ,1  0  0 ,0  1  0 ,0  0  0 olarak yazarız.
Ayrıca, p  1  p , p  0  0 , p  p  0 denklikleri yazılabilir.
6
'' '' ve '' '' işlemlerinin Özellikleri
Tek Kuvvet Özelliği
p önermesi ile p  p bileĢik önermesinin doğruluk değerlerini gösteren tablo
yapalım.
p
1
0
p p
1
0
p
1
0
Yukarıdan da görüleceği gibi p  p  p özelliğine '' '' iĢleminin tek kuvvet
özelliği denir.
p
p
p p
1
1
1
0
0
0
p  p  p özelliğine '' '' iĢleminin tek kuvvet özelliği denir.
DeğiĢme Özelliği
p  q, q  p bileşik önermeleri ile , p  q, q  p bileşik önermelerinin
doğruluk değerlerini gösteren tabloyu yapalım.
p
1
1
0
0
q
1
0
1
0
pq
1
1
1
0
q p
1
1
1
0
pq q p
7
p
1
1
0
0
pq
1
0
0
0
q
1
0
1
0
q p
1
0
0
0
pq q p
Buna göre, Örneğin, ‘’ Ankara’ya gittim ve Anıtkabir’i gezdim.’’ BileĢik
önermesiyle ,
‘’Anıtkabir’i gezdim ve Ankara’ya gittim.’’ BileĢik önermeleri eĢdeğerdir. Halbuki
konuĢma dilinde bu bileĢik önermeye farklı anlam verilir. Bu örnek matematikte doğru
sonuçlara ulaĢmak için, verilen tanımlara uygun olarak düĢünmemiz gerektiğini ortaya
koyar.
BirleĢme Özelliği
p, q, r herhangi üç önerme olsun. p   q  r    p  q   r özelliğine '' '' nın
birleĢme özelliği, p   q  r    p  q   r özelliğinede '' '' nin birleĢme özelliği
denir. Bu özelliklerin doğruluğunu gösteren tabloyu gösterelim.
p
q
1
1
1
1
0
0
0
0
1
1
0
0
1
1
0
0
r
1
0
1
0
1
0
1
0
qr
pq
1
1
1
0
1
1
1
0
1
1
1
1
1
1
0
0
p  (q  r )
1
1
1
1
1
1
1
0
( p  q)  r
1
1
1
1
1
1
1
0
p  (q  r ) = ( p  q)  r
8
p
q
1
1
1
1
0
0
0
0
1
1
0
0
1
1
0
0
r
1
0
1
0
1
0
1
0
qr
pq
1
0
0
0
1
0
0
0
1
1
0
0
0
0
0
0
p  (q  r )
1
0
0
0
0
0
0
0
( p  q)  r
1
0
0
0
0
0
0
0
p  (q  r )  ( p  q)  r
Dağılma Özelliği
Sayılarda tanımlanan çarpma iĢleminin toplama iĢlemi üzerine dağılma
özelliğinin varlığını biliyoruz. Örneğin, 5  (3  4)  (5  3)  (5  4) eĢitliği doğrudur.
Bunun gibi önermelerde '' '' nın '' '' bağlacı üzerine ve '' '' nın '' '' iĢlemi üzerine
dağılma özelliği vardır.
1 p  q  r    p  q   p  r 
2  p  q  r    p  q   p  r 
3  q  r   p  q  p  r  p
4  q  r   p  q  p  r  p
p
q
r
qr
pq
pr
p  q  r 
 p  q   p  r 
1
1
1
1
0
0
0
0
1
1
0
0
1
1
0
0
1
0
1
0
1
0
1
0
1
0
0
0
0
0
0
0
1
1
1
1
1
1
0
0
1
1
1
1
1
0
1
0
1
1
1
1
0
0
0
0
1
1
1
1
1
0
0
0
p  q  r    p  q   p  r 
Diğerleri de aynı Ģekilde tablo ile gösterilir.
9
Örnekler:
1  p  0 , q  1 , r  0 olmak üzere,
 p  q   r    r    p  q  
bileşik önermesinin doğruluk değerini bulalım.
Çözüm:
p  0  p  1 , q  1  q  0 , r  0  r  1 bu değerleri yerlerine yazalım.
 p  q   r    r    p  q    0  0   1  1  1  0   0  1  1  1  0  1  1
2  p   p  q   p  q olduğunu doğruluk tablosunu kullanmadan bulalım.
Çözüm:
p   p  q   p  q   p  p    p  q   0   p  q   p  q olur.
DE MORGAN KURALLARI
p ile q iki önerme olmak üzere,
 p  q   p  q
 p  q   p  q
Özellikleri De Morgan Kuralları olarak adlandırılır. Bu özelliklerin doğruluğunu
tablo yardımıyla vermeye çalıĢalım.
p
q
p
q
pq
 p  q 
p  q
1
1
0
0
1
0
1
0
0
0
1
1
0
1
0
1
1
1
1
0
0
0
0
1
0
0
0
1
 p  q   p  q
p
q
p
q
pq
 p  q 
p  q
1
1
0
0
1
0
1
0
0
0
1
1
0
1
0
1
1
0
0
0
0
1
1
1
0
1
1
1
10
 p  q   p  q
Örnek:
1  Aşağıda verilen önermelerin değillerini alalım.
a) ''Sinemaya gitmedim ve ders çalıĢtım.'' Önermesine De Morgan kuralını uygulayalım.
'' Sinemaya gittim veya ders çalıĢmadım.''
b) ''3  5 veya 6  7'' bileşik önermesinin değili de,
''3  5 ve 6  7'' şeklindedir.
2  p   q  r  bileşik önermesinin değilini alalım.
 p   q  r    p   q  r   p   q  r    ( p  q)  ( p  r) olur.
Totoloji Ve ÇeliĢki
Tanım:
Bir bileĢik önerme, kendisini oluĢturan önermelerin her değeri için daima ‘1’
değerini alıyorsa , bu bileĢik önermeye totoloji denir. Ve daima ‘0’ değerini alıyorsa
buna çeliĢki denir.
Bu tanıma göre, p  p bir totoloji, p  p de bir çelişkidir.
Mantık Kurallarını Elektrik Devrelerine Uygulanması
Mantık kurallarının elektrik devreleri üzerindeki uygulamaları, günümüzde büyük
geliĢmeler göstermiĢtir. Bilgisayarlar programları dünyasında önemli yer tutar.
Bu uygulamanın temelinde anahtarların paralel ve seri bağlanması yatar.
Paralel bağlamada
Seri bağlamada
'' '' bağlacı,
'' '' bağlacı arasında benzerlik vardır.
a) Paralel Bağlama
11
p
q
p anahtarı
q anahtarı
1
1
0
0
1
0
1
0
p  q Lamba
(ıĢık)
Yanar
Yanar
Yanar
Yanmaz
b) Seri Bağlama
p
q
p anahtarı
q anahtarı
1
1
0
0
1
0
1
0
p  q Lamba
(ıĢık)
Yanar
Yanmaz
Yanmaz
Yanmaz
Bu açıklamalardan sonra Ģu sonuca ulaĢabiliriz.
Elektrik devrelerinin özellikleri mantık kurallarıyla incelenebilir. Aynı zamanda
bir elektrik devresi  ile  ile yapılan önermelerle de gösterilebilir.
12
Örnek:
p
t
r
q
s
Yukarıdaki Ģemaya uygun bileĢik önermeyi yazarak doğruluk değerini bulunuz.
 p   q  (r  s)   t ve
0  1  (0  1)   1  0  1  1  1  0  1  1  1  1  1 ( Lamba Yanar )
KoĢullu önerme
Günlük yaĢantımızda iki önerme arasına ‘ise’ sözcüğünü ekleyerek çeĢitli bileĢik
önermeler yaparız. Örneğin,
‘Yağmur yağar ise yola çıkmam’
‘Misafir geldi ise sinemaya gitmem’
gibi önermeleri sık sık kullanırız. Yukarıdaki önermeler iki önerme arasına ‘ise’ bağlacı
konularak elde edilen önermelerdir. Matematikte ise ‘ise’ bağlacı sembol olarak ‘  ’ile
gösterilir. '' p  q '' önermesi ‘ p ise q ’ Ģeklinde okunur.
Tanım :
'' p  q '' biçimindeki bileĢik önermeye koĢullu önerme denir. p  q koĢullu
önermesi
p önermesi doğru q yanlıĢ iken yanlıĢ, diğer durumlarda doğrudur.
Bu tanıma göre , aĢağıdaki doğruluk tablosunu yazabiliriz.
p
1
1
0
0
q
1
0
1
0
pq
1
0
1
1
Ġki Yönlü KoĢullu Önerme
Tanım
 p  q  q  p
bileĢik önermesine ‘Ġki yönlü koĢullu önerme ’denir.
13
Ve  p  q  biçiminde yazılır. ‘ p ancak ve ancak q ’ diye okunur.
 p  q    q  p    p  q  olarak yazılabilir.
p
1
1
0
0
q
1
0
1
0
pq
1
0
0
1
dir.
Aksiyom, Teorem, Ġspat
Bir matematiksel yapı kurmak için aĢağıda belirtilen dört temel unsura gerek
vardır.
I. Tanımsız terimler
II. TanımlanmıĢ terimler
III. Doğru olduğunu sezgi ile anladığımız ve ispatsız olarak kabul ettiğimiz aksiyom
denilen önermeler.
IV. Aksiyomlara dayanılarak ve mantık kuralları kullanılarak doğrulukları gösterilen
teorem denilen önermelerdir.
Tanım:
p önermesi doğru ve p  q koĢullu önermesi de doğru ise , p  q
önermesine teorem denir.
p  q önermesine p önermesine hipotez , q ise hüküm denir. Bir teoremin
hükmü de doğru önermedir.
Bir matematik sistemi içindeki önermeleri aksiyomlar ve teoremler olarak ikiye
ayırırız. Aksiyomlar doğruluğu ispatlanmadan kabul edilen önermelerdir.
Teoremler ise doğruluğu ispatlanan önermelerdir.
Ġspat Yöntemleri
1. Doğrudan (Direkt) ispat Yöntemi
p  q teoreminde p nin doğru olduğu kabul edilerek q nun doğru
olduğu gösterilir.
Örnek:
Teorem: Tek sayının karesi yine tek sayıdır.
14
Hipotez ( p ) : x tek sayı
Hüküm ( q ) : x 2 tek sayı
Ġspat :
x tek sayı ise , x  2n  1 olacak Ģekilde bir n tamsayısı vardır.
p doğru  x 2   2n  1
2
 x 2  4n 2  4n  1
 x 2  4n   n  1  1
 x 2  2  2n  n  1   1
 x 2 tek sayıdır.
 q doğrudur.
2. Olmayana Ergi Yöntemiyle Ġspat ( Dolaylı Ġspat )
 p  q    q  p olduğunu biliyoruz.  p  q  teoreminin yerine
 q  p teoremi ispatlanır.
Örnek:
Teorem:
x  3  2 x  1  7 , p  q olarak yazalım.
p
q
q :  2 x  1  7    2 x  1  7 
 2x  1  7  2x  1  1  6  2x  6  x  3
p
q  p önermesi doğrudur. Öyleyse, p  q önermesi de doğrudur.
3. Tümden Gelim Yöntemiyle Ġspat
Tümden gelim yöntemiyle ispat için, genel durum ele alınarak teoremin
doğruluğu gösterilir.
Örnek:
Teorem:
‘’Ġki sayının çarpımı çift sayı ise bu sayılardan en az biri çift sayıdır.’’
15
Teoremini tümden gelim yöntemiyle ispatlayalım.
Bu teoremi,
‘ a  b çift sayı ise, a çiftsayı veya b çift sayıdır. ’ biçiminde ifade ederiz.
Ġspat:
a  b çift sayı  a  b sayısı 2 ile bölünür.
 a sayısı 2 ile bölünür veya b sayısı 2 ile bölünür.
 a çift sayıdır veya b çift sayıdır.
4. Deneme Yöntemiyle Ġspat
Bir önermedeki değerler değiĢik değerler alıyorsa ve bu değerlerin
önermede tek tek yerlerine konularak kontrol edilmesi mümkünse,
önermenin doğru olup olmadığı bu Ģekilde gösterilir. Bütün değerler
önermeyi doğruluyorsa, bu önerme doğrudur.
Örnek:
A  1,0,1 kümesinin çarpma işlemine göre kapalı olduğunu gösterelim.
Bunun için A kümesindeki her iki elemanın çarpımının
yine A kümesinde olmasına bakalım.
1  0  0,  1  1  1, 0  1  0, 0  0  0, 1  1  1, 1  1  1
O halde, A kümesindeki her iki elemanın çarpımının
yine A kümesinde olduğu için kapalıdır.
5. Aksine Örnek Verme Yöntemi ile ispat
Verilen önermenin doğru olmadığını gösteren en az bir değer varsa, bu
önermenin yanlıĢ olduğu ispatlanmıĢ olur.
Örnek:
Her tamsayının karesinin pozitif olmadığını gösterelim.
Bunun için tamsayılar kümesinde bulunan en az bir elemanın karesinin pozitif
olmadığını göstermek gerekir. Sıfır’ın karesi pozitif değildir. Öyleyse her
tamsayının karesi pozitif değildir.
16
2.BÖLÜM
KÜMELER
1- Küme :
Belirli özelliklere sahip nesneler topluluğuna küme denir. Kümeyi
meydana getiren nesnelerin her birine kümenin elemanı denir. Kümeler
A, B, C,... gibi büyük harflerle ve kümenin elemanları ise a, b, c,... gibi küçük
harflerle gösterilir.
Bir a elemanı A kümesine ait ise a  A Ģeklinde gösterilir. Eğer a
elemanı A kümesine ait değilse a  A Ģeklinde gösterilir.
Kümeler üç farklı Ģekilde gösterilebilir.
1. Liste biçiminde , A  {a, b, c, d , e} kümenin elemanları tek tek sırayla
yazılmıĢtır.
2. Venn ġemasıyla,
17
.a
.b
.c
A kümesinin elemanları
.d
.e
3. Ortak özellik yöntemiyle , burada kümeyi meydana getiren elemanların
ortak özelliği verilerek küme ifade edilir. Örneğin,
 {x / x bir doğal sayı} veya
K  {x / 1 ile 10 arasındaki çift tam sayılar} olsun. K  {2,4,6,8} dir.
E  {x / x  6 ve x doğal sayı} olsun. E  {0,1,2,3,4,5} olarak yazılabilir.
Her küme ortak özellik yöntemiyle gösterilemez. Örneğin,
A  { , , a, b, c,*,#} kümesini ortak özellik yöntemiyle gösteremezsiniz.
2- Sonlu ve Sonsuz Kümeler :
Bir kümenin elemanlarını saydığımızda sayma iĢlemi bir yerde sona eriyorsa, bu
kümeye sonlu küme denir. Sonlu olmayan kümelere sonsuz küme denir.
Örneğin,
A  {x / 3  x  8 ve x tam sayı}  {3,4,5,6,7} ve eleman sayısı S ( A)  4 tür.
B  {x / x tek sayı}  {1,3,5,7,9,11,13,15,...} olarak yazılır.
Ve eleman sayısı sonsuzdur. Burada, kümesi sonlu B kümesi sonsuzdur.
3- BoĢ Küme :
Hiçbir elemanı olmayan kümeye boĢ küme denir. Ve  sembolüyle gösterilir.
C  {x / x2  4 ve x tam sayı} kümesi boĢ kümedir. Çünkü , her tam sayının
karesi pozitiftir. S   ve S (C )  0 dır
4- Alt Küme :
A kümesinin her elemanı B kümesinin bir elemanı ise, A kümesine B
kümesinin bir alt kümesi denir. Ve A  B Ģeklinde gösterilir. A  B ise bunu iki
Ģekilde okuyabiliriz.
'' A kümesi B nin alt kümesidir'' veya ''B kümesi A yı kapsar'' dır.
A  B ifadesi B  A olarak ta yazılabilir.
Eğer , D kümesi E nin alt kümesi değilse bunu D  E olarak gösteririz.
Bir A kümesinin eleman sayısı n olsun. Bu kümenin altküme sayısı 2n ve özalt
küme sayısı ise 2n  1 dir.
Örnek :
18
A  {a, b, c, d , e, f } olsun.
A
A
2n  26  64 ( alt küme sayısı )
2n  1  26  1  64  1  63 dür. ( Öz alt küme sayısı )
Örnek :
A  {a, b, c, d} ve B  {a, b, c, d , e, f , g} olsun. A kümesinin her elemanı B kümesinde
bulunduğu için A kümesi B nin bir alt kümesidir. A  B dir. ġemayla gösterelim.
.e
.a
.b
.c
.d
Örnek :
.f
A
.g
B
D  {1,2,3,4} ve E  {1,2,3,5,7,9} olsun. D  E dir. Çünkü , 4  D
fakat 4  E dir. O halde D  E dir.
Alt Küme ĠĢleminin Özellikleri :
1. BoĢ küme her kümenin bir alt kümesidir.
 kümede olupta A kümesinde olmayan hiç bir eleman söyleyemeyiz. O halde
  A dır.
2. A bir küme olmak üzere A  A dır. A kümesinin her elemanı yine kendisinin
bir elemanı olduğundan tanım gereği A  A dır.
3. A ve B iki küme olsun, eğer A  B ve B  A ise A  B dir.
4. A  B ve B  C ise A  C dir. A  B olduğunda A nın her elemanı B ninde bir
elemanı ve B  C olduğunda B nin her elemanı C ninde bir elemanıdır. O halde
A nın her elemanı C nin bir elemanıdır. Ve A  C dir.
5- EĢit Kümeler :
Aynı elemanlardan oluĢan kümelere eĢit kümeler denir. A ve B eĢit kümeler ise
A  B olarak gösterilir.
Örnek :
A  {a, b, c, d}, B  {a, c, b, d} ve C  {a, b, c, e} olsun.
Burada A  B fakat A  C dir. Çünkü ,
A ile B nin elemanları aynı fakat A ile C nin elemanları farklıdır.
6- Özalt Küme :
19
Bir kümenin kendisinden farklı her alt kümesine bu kümenin öz alt kümesi
denir. Ve
Yukarıda verildiği gibi 2n  1 formülüyle sayısı bulanbilir.
KÜMELERDE ĠġLEMLER
Kümeler arasında birleĢim, kesiĢim, tümleme ve fark iĢlemlerini tanımaya
çalıĢacağız.
1. Kümelerde BirleĢim
A ve B iki küme olsun. A ile B nin bütün elemanlarından oluĢan kümeye A ile B
nin birleĢimi denir. A  B ile gösterilir.
A  B  {x / x  A veya x  B} biçimin de tanımlanır.
Örnek :
A  1,2,3,4 ve B  2,3,5,7,8,9 olsun. A  B  ?
A  B  1,2,3,4,5,7,8,9
olur.
.1
.4
.5
.2
.7
.3
.9
A
B
Örnek :
C  a, b, c, d  ve D  b, d , e, f , s kümeleri veriliyor. C  D kümesini liste biçiminde yazınız.
20
.a .b .c
.d .e . f
.s
CD
C  D  a, b, c, d , e, f , s dir.
Örnek :
E  3,4,5,9 ve F  1,2,3,4,5,6,7,8,9 olsun. E  F  ?
.1
.3
.4
.5
.9
.2
.6
.7
E
.8
F
BirleĢim ĠĢleminin Özellikleri
A, B ve C herhangi iki küme olsun.
1 A  A  A
( tek kuvvet özelliği )
2 A B  B  A
( DeğiĢme özelliği )
3  A  ( B  C )  ( A  B)  C ( Birleşme özelliği )
4 A   A  A
Bunlardan ilk üçünü ispatlayalım.
1  A  A  x / x  A ve x  A  x / x  A  A
2  A  B  x / x  A veya x  B  x / x  B veya x  A  B  A
3  A  ( B  C )  ( A  B)  C   x / x  A veya x   B  C 
  x / x  A veya  x  B veya x  C 
  x /  x  A veya x  B  veya x  C
 ( A  B)  C dir.
21
Örnekler :
1  A  a, b, c , B  a, b ve C  c, d , e kümeleri veriliyor. Aşağıda verilen kümeleri liste
biçiminde yazınız.
a) A  B
b) A  C
c) B  C
d ) A  (B  C)
2  A  B koşuluna uyan herhangi iki alt küme yazınız. A  B kümesini yazarak,
A  B  A  B  B önermesinin doğru olduğunu ispatlamaya çalıĢınız.
3  A  B    A    B   önermesini ispatlayınız.
4   A      A  B  kümesini daha sade biçimde yazınız.
5  A   x / x tamsayı ve  3  x  2 ,
B   x / x tamsayı ve 0  x  5 kümeleri veriliyor.
A  B kümesini ortak özellik yöntemiyle yazınız.
Kümelerde KesiĢim ĠĢlemi
A ve B herhangi iki küme olsun. A ile B nin ortak elemanlarından oluĢan kümeye,
A ile B nin kesiĢimi denir. Ve bunu, A  B Ģeklinde gösteririz.
A  B  x / x  A ve x  B biçiminde yazılır.
Örnek :
A  5,6,7 , B  6,7,8,9 ve C  8,9 kümeleri veriliyor.
A  B, A  C , B  C kümelerini liste biçiminde yazarak şemayla gösteriniz.
A  B  6,7
.
5
.6
.7
B  C  8,9
AC  
.8
.5
.6
.7
.8
.9
.9
A
B
Bu örnekten,
 A  B   A,
A
.6
C
.8
.9
.7
C
B
 A  B   B  C  B  B  C  C olduğu görülmektedir.
Ayrıca, A  C   ise, A ve C kümelerine ayrık küme denir.
KesiĢim ĠĢleminin Özellikleri
22
A, B ve C herhangi üç küme olsun.
1 A  A  A
( Tek kuvvet özelliği )
2 A B  B  A
( DeğiĢme özelliği )
3  A  ( B  C )  ( A  B)  C
(Birleşme özelliği )
4 A  
Bu özelliklerden ilk üçünü ispatlayalım.
1  A  A   x / x  A  x  A  A
2  A  B   x / x  A  x  B   x / x  B  x  A  B  A
3  A  ( B  C )   x / x  A  x  ( B  C )   x / x  A  x  B  x  C
  x / x  ( A  B)  x  C  ( A  B)  C dir.
Örnekler :
1  A  a, b, c , B  a, d  , C  c, d , e kümeleri veriliyor. Aşağıdaki kümeleri liste biçiminde yazarak
şema ile gösteriniz.
A  B b)
a)
A  C c) B  C d )
A  (B  C )
Çözüm:
a) A  B  a b) A  C  c c) B  C  d  d ) A  ( B  C )  
.a
.b
.a
.b
B
A B
.a
.d
.d
.c
A
.c
.d
.c
.e
.e
A
C
B
B C
C
AC
.d
B C
.a
.b
.c
A
23
2   A  B    A    kümesini en sade biçimde yazınız.
3  A   x / x tamsayı ve 21  x  30 ,
B   x / x tamsayı ve 11  x  27 kümeleri veriliyor. A  B kümesini liste biçiminde göstererek
şemayla gösteriniz.
Çözüm :
A  21,22,23,24,25,26,27,28,29 ve B  11,12,13,14,15,16,17,18,19,20,21,22,23,24,25,26
olarak yazılır. Öncelikle liste biçiminde yazalım.
A  B  21,22,23,24,25,26 , A  B   x / 21  x  26, x  A ve x  B dır.
KESĠġĠM VE BĠRLEġĠM ĠġLEMLERĠNĠN ORTAK
ÖZELLĠKLERĠ
A ,B ve C herhangi üç küme olsun.
1  A   B  C    A  B    A  C  ( Birleşim işleminin kesişim üzerine soldan dağılma özelliği )
2  A   B  C    A  B    A  C  ( Kesişim işleminin birleşim üzerine soldan dağılma özelliği)
3   B  C   A   B  A    C  A  ( Birleşim işleminin kesişim üzerine sağdan dağılma özelliği)
4   B  C   A   B  A    C  A  (Kesişim işleminin birleşim üzerine sağdan dağılma özelliği)
Bu özelliklerden, A   B  C    A  B    A  C  olduğunu ispatlayalım.
A   B  C    x / x  A veya x  ( B  C )    x / x  A veya x  B ve x  C
  x /  x  A veya x  B  ve  x  A veya x  C 
=  x / x   A  B  ve x   A  C 
=  A  B    A  C  dir.
ġimdi, A, B ve C kümelerini ele alalım.
1  S ( A  B)  S ( A)  S ( B)  S ( A  B)
2  S ( A  B  C )  S ( A)  S ( B)  S (C )  S ( A  B)  S ( A  C )  S ( B  C )  S ( A  B  C )
olarak yazılabilir.
24
Örnek :
1  35 kişilik bir sınıfta her öğrencinin Ġngilizce veya Fransızca bilidiği varsayılıyor. Bu sınıfta
Ġngilizce bilenlerin sayısı, Fransızca bilenlerin sayısının 2 katından 1 fazladır. Her iki dili bilen
5 kişi olduğuna göre, yalnızca Ġngilizce bilen kaç kişi vardır?
Çözüm:
Ġngilizce bilenlerin kümesini  İ
Fransızca bilenleri kümesini  F ile gösterelim.
S ( İ  F )  35, S ( İ  F )  5 tir. S ( F )  x ve S ( İ )  2 x  1 olarak yazılabilir.
S (İ  F )  S (İ )  S (F )  S (İ  F )
35  2 x  1  x  5
35  3x  4
35+4  3 x
39  3 x
39
x
 13 tür.
3
S ( İ )  2  13  1  27 kişi
Yalnızca Ġngilizce bilenlerin sayısı
S ( İ )  S ( İ  F )  27  5  22 kişidir.
22
5
İ
8
F
Ödev :
40 kiĢilik bir sınıfta 10 öğrenci voleybol, 9 öğrenci basketbol, 4 öğrenci hem
voleybol hem basketbol oynamaktadır. Bu sınıfta bu iki oyundan hiç birini oynamayan
kaç kiĢi vardır?
Örnek :
A  1, 2,3, 4,5 , B  0,1, 4,7 , C  3, 4,6,9 olduğuna göre,
a)
d)
g)
 A  B   C  ? b)  A  C   B  ? c )  A  C   A  ?
 A  B   C  ? e) B   A  C   ? f ) A   A  B   ?
 A  B    A  C   ? h)  A  B    A  C   ?
25
Çözüm:
Yalnızca, a, e, ve h Ģıklarını çözelim. Gerisi ödev bırakılmıĢtır.
a)
e)
h)
 A  B   C  1, 4  3, 4,6,9  1,3, 4,6,9
B   A  C   0,1, 4,7  3, 4  0,1,3, 4,7
 A  B    A  C   1, 4  3, 4  4
Örnek :
A, B, C herhangi üç küme olsun.  A  B    x, y, z, t ve
 A  C    x, a, b, y olduğuna göre A   A  C   ?
Çözüm :
A   B  C    A  B    A  C   x, y, z, t  x, a, b, y  x, y, z, t , a, b
Örnek :
A, B, C herhangi üç küme olsun.  A  B   1,2,3,4 ve
 A  C   4,5,6 olduğuna göre A   B  C   ?
Çözüm :
A   B  C    A  B    A  C   1,2,3,4  4,5,6  1,2,3,4,5,6 dır.
Örnek :
A   x / x  ,  6  x  0 , B   x / x  ,  2  x  7 , C  x / x  ,  3  x  5
kümeleri veriliyor.  A  B   C kümesini ortak özellik yöntemiyle yazınız.
26
Çözüm :
A
0
-6
-2
-2
B
0
7
Bu iki kümenin birleĢimi
isteniyor.
0
C
5
-3
 A  B
-3
5
 A  B   C  x / x 
,  3  x  5 dır.
Örnek :
AĢağıdaki kümeleri en sade biçimde gösteriniz.
a)  A     A  ? b)  A       ? c)  A      B     ?
Çözüm :
a)
b)
c)
 A    A    A  A
 A      A    
 A     B      B  B
EVRENSEL KÜME
Kümelerle ilgili iĢlemlerde, uygulamalarda evrensel küme denilen ve yeteri
kadar elemanı olan bir küme alınır. Yazacağımız tüm kümeler bu evrensel kümenin alt
kümesi olacaktır. Örneğin, bir okulun futbol takımını oluĢturmayı düĢünüyorsanız sizin
için evrensel küme O okulun öğrencileri olacaktır. Eğer iĢiniz doğal sayılarla ise , sizin
için evrensel küme doğal sayılar kümesi olacaktır.
Evrensel küme sonsuz elemanlı olabileceği gibi sonlu elemanlıda olabilir.
Evrensel kümeyi biz E ile göstereceğiz.
27
Örnek :
A   x / x  ,  x  2  3x  1  0 kümesinin elemanlarını yazalım.
Çözüm :
 x  2  3x  1  0   x  2   0 ve  3x  1  0

x  2

ve
x
1
bulunur.
3
Evrensel küme tamsayılar kümesi olduğuna göre  2  A fakat
A  2 dir.
1
 A olacağından
3
TÜMLEME
E evrensel kümesi içinde bir A kümesi verilsin. E nin elemanı olup A nın
elemanı olmayan elemanların kümesine A nın tümleyeni denir. Ve At ile gösterilir.
Bu tanıma göre, At   x / x  E ve x  A olarak yazılır.
Örnek :
E  0,1,2,3,4,5,6,7,8,9 evrensel kümesi ile A  0,1,3,8 kümesi verilsin.
At kümesini yazınız.
Çözüm :
.
.2
.4
.5
.6
.0 .1.3 .8
A
.7
.9
At
At  2,4,5,6,7,9
E
28
A
At
E
Yukarıda, A, At ve E kümesi şemayla verilmiştir. Buna göre aĢağıdaki önermelerin
doğru olduğunu görmeye çalıĢalım.
1  A  At  E 2  A  At   3  A  E  E
4  A  E  A 5  E t   6  t  E 7  At  E
8  A  E 9   At   A dır.
t
Bunlardan ilk ikisini ispatlayalım.


2  A  A   x / x   A  A    x / x  A  x  A    x / x   
1  A  At  x / x   A  At    x / x  A  x  At    x / x  E  E
t
t
t
DE MORGAN KURALLARI
A ve B iki küme olsun.
t
t
 A  B   At  Bt ve  A  B   At  Bt dir.
ġekille ispatlamaya çalıĢalım.
A
( A  B )t
B
A
B
At  Bt
29
Örnek :
A  a, b, c , B  a, d , e , C  d , e, a, b ve E  a, b, c, d , e olmak üzere,
a) At  Bt  C t  d , e  b, c  c  
b)  A  B   C t  a, b, c, d , e  c  a, b, d , e
c)  At  B t   d , e  b, c  b, c, d , e   a
t
t
t
Örnek :
A
B
Taralı bölgeyi ifade ediniz.
C
Çözüm :
B   A C
ĠKĠ KÜMENĠN FARKI
A ve B iki küme olsun. A nın elemanı olup, B nin elemanı olmayan elemanların oluĢturduğu
kümeye A dan B nin farkı veya A fark B denir. Ve A  B semboluyle gösterilir. Bu tanıma göre,
A  B   x / x  A  x  B dir.
Örnek :
A  a, b, c, d  , B  a, c, d , f , e , C   f , e kümeleri veriliyor.
A  B  ? B  A  ? A  C  ? C  B  ? kümelerini bulunuz.
30
Çözüm:
B  A   f , e
.a
.b
.f
.c
.d
A
.e
A  B  b
B
A ve C kümeleri ayrık olduğundan A  C  A ve C  A  C dir.
.a
.f
.b
.c
.e
.d
C
A
C  B  ? çözümü ödev olarak bırakılmıĢtır.
KÜMELERDE FARK ĠġLEMĠNĠN ÖZELLĠKLERĠ
1  A  A   2  A    A 3    A   4  A  B  A  Bt
5  E  A  At
6  A  B  B  A dır.
Örnek :
S ( A  B)  3 , S ( B  A)  7 , S  A  B   15 ise  A  B  kümesi kaç elemanlıdır?
Çözüm :
S  A  B   x olsun. S  A  B   3  x  7
A B
15  3  x  7
x  5 olur.
S  A  B   5 dir.
3
x
7
B A
A
B
31
3-BÖLÜM
TEMEL KAVRAMLAR
SAYI KÜMELERĠ
1- Doğal sayılar ( Natural Numbers ):
 {0,1, 2,3,...,9,10,11,...,99,100,101,...,999,1000,1001,...n, n  1,...}
kümesinin her bir elemanına bir doğal sayı denir. Doğal sayılar kümesinin en küçük
elemanı 0 dır.
kümesi üstten sınırsız olduğundan dolayı en büyük elemanı yoktur
diyebiliriz.
doğal sayılar kümesinden 0 (sıfır) elemanını çıkardığımızda geriye
kalan sayıların oluĢturduğu kümeye sayma sayıları kümesi denir. Sayma sayıları
kümesi,
S  {1,2,3,...}
Ģeklinde gösterilir.
0,1,2,3,4,5,6,7,8,9 sembollerinin her birine rakam (figure-Number) denir.
Bizler rakamları kullanarak sayıları yazarız.
Doğal sayıları, 0  1  2  3  4  ...  9  10  11...  99  100  ... Ģeklinde
küçükten büyüğe doğru sıralarız.
Ard arda gelen doğal sayılara ,ardıĢık doğal sayılar denir.
23 4
burada 2 nin ardıĢığı 3 ve 3 ün ardıĢığı 4 tür.
Ġki basamaklı bir ab sayısını ab  10  a  b ,
üç basamaklı bir abc sayısını abc  100a  10b  c ,
dört basamaklı bir abcd sayısını abcd  1000a  100b410c  d Ģeklinde
çözümleyebiliriz.
Örneğin, 3875 sayısını çözümleyelim.
3875  3  1000  8  100  7  10  5  3  103  8  102  7  101  5  100 dir.
3875
binlik
onluk
yüzlük
birlik
Örnek:
Ġki basamaklı ab ve ba sayıları arasında fark en fazla ne olabilir?
Çözüm:
ab  ba  10a  b   10b  a   10a  a  (10b  b)  9a  9b  9(a  b) olur.
ġu halde , ab  ba  9(a  b) dir . En fazla fark ne olabilir diye sorulduğundan
(a  b) nin alabileceği değer 8 olacaktır. O halde en fazla fark 9(a  b)  9  8  72 dir.
32
Örnek:
Ġki basamaklı bir sayı, rakamları toplamının 3 katıdır. Bu sayı kaçtır?
Çözüm:
Söz konusu sayı ab olsun.
ab  3(a  b)  10a  b  3a  3b  7a  2b  a  2, b  7
sayı ab =27 dir.
bulunur. O halde istenen
Örnek:
abc ve cba üç basamaklı sayılardır.
abc  cba  297
ise abc biçiminde yazılacak en küçük sayıyı bulunuz.
Çözüm:
abc  cba  297  (100a  10b  c)  (100c  10b  a)  297
 99a  99c  297  99.3  a  c  3 olmalıdır. Bu durumda a  4 ve c  1 olur.
abc  401 dir.
Örnek:
Beş basamaklı rakamları biribirinden farklı en büyük tam sayı ile
beş basamaklı en küçük tek doğal sayı arasındaki fark nedir?
Çözüm:
Beş basamaklı en büyük tam sayı 98765,
Beş basamaklı en küçük tek doğal sayı 10001 dir. O halde,
98765
 10001 tür.
88764
Örnek:
Toplamları 32 olan iki sayının çarpımı en çok kaçtır?
Çözüm:
Bir kural olarak diyebiliriz '' Toplamları verilen iki sayının çarpımlarının
en büyük olması isteniyorsa, bu iki sayının arasındaki fark en küçük olmalıdır.
Toplamları 32 ise biz 16 ve16 sayılarını alalım.Buradan,
16  16  0 ve 16 16  256 olarak bulunacaktır.
Örnek:
a, b, c birbirinden farklı rakamlar olsun. 3a  5b  4c ifadesinin alabileceği en büyük ve
en küçük değerin toplamı kaçtır?
Çözüm:
3a  5b  4c ifadesinin en büyük değer alması için b  9, a  8 ve c  0 olmalıdır.
O halde , 3  8  5  9  4  0  69
3a  5b  4c ifadesinin en küçük değer alması için b  0, a  1, c  9 alınmalıdır.
O halde, 3 1  5  0  4  9  33 olur.Sonuç olarak 69  (33)  36 olur.
33
Örnek: ArdıĢık 13 tam sayının toplamı 156 olduğuna göre,
bu sayılardan en küçük ile en büyüğünün toplamı nedir?
Çözüm: Öncelikle ortanca sayıyı bulalım. Ortanca sayı 
Sayıların Toplamı 156

 12 dir.
Terim Sayısı
13
6,7,8,9,10,11- 12  13,14,15,16,17,18


en küçük
sayı
en büyük
sayı
Toplamları, 6  18  24 olur.
Örnek:
Ġki basamaklı birbirinden farklı beĢ doğal sayını toplamı417 dir.
Buna göre, bu sayıların en küçüğü en az kaçtır?
Çözüm:
Sayılardan birinin en küçük olması için diğer sayıların büyük olması gerekir.
98  97  96  95  x  417
x  417  386  31  x  31 olur.
Yani, bu sayıların en küçüğü 31 olur.
Örnek:
a, b, c pozitif tam sayılardır.
a  c  35
c  b  42 olduğuna göre, a  b  c toplamının alabileceği enbüyük ve en küçük tam sayı değerlerinin
toplamı kaç olur?
Çözüm:
a  b  c toplamının küçük olması için verilen denklemlerde ortak bilinmeyen en büyük seçilir.
5  7  35 , 7  6  42
a  5, c  7, b  6 a  b  c  5  6  7  18 (en küçük)
a  b  c toplamının en büyük olması için verilen denklemlerde ortak bilinmeyen en küçük seçilir.
35 1  35 , 1 42  42 a  35, c  1, b  42 olur.
a  b  c  35  42  1  78 (en büyük)
18  78  96 olur.
34
Örnek:
x, y, z ardıĢık tam sayılardır. x  y  z olduğuna göre,
 x  z   y  z   x  y
2
3
5
ifadesinin sonucu nedir?
Çözüm:
x  y  z şartını sağlayan değerler seçelim.
1  3  5 olsun.  x  z    y  z    x  y   1  5    3  5   1  3   4    2    2 
2
3
5
2
3
5
2
3
5
=16   8    32 
 16   40   24 olur.
BĠR DOĞAL SAYININ KUVVETĠ
ve n 

olsun. n tane a nın çarpımında elde edilen sayı a n dir. Buna göre
a n  a  a  a  ...  a dır.
n - tane
n
a ifadesinde ; a ya taban , n ye üs veya kuvvet denir.
Özel olarak,
n  2  a 2 dir. ( a nın karesi Ģeklinde okunur.)
n  3  a 3 dir. ( a nın küpü Ģeklinde okunur.)
a
a  ve n  0 için a1  a ve a  1 olacağı açıktır.
Ayrıca 0n  0 ve 00 tanımsızdır.
0
Örnek:
24  32  05  70 ifadesini hesaplayınız.
Çözüm:
24  2  2  2  2  16, 32  9, 05  0, 70  1 olacağından
24  32  05  70  16  9  0  1  26 bulunur.
Bir doğal sayı baĢka bir doğal sayının karesi ise bu doğal sayı tam karedir denir.
Örneğin, 1,4,9,16,25…,100,121,144,… sayıları birer tam kare olan doğal sayılardır.
Çünkü,
12  1, 22  4, 32  9, ... ,42  16, ... ,122  144 gibi.
Doğal sayılarlarda üslü ifadelerin çarpma iĢlemi ile ilgili özelliklerini verelim.
m ve n sıfırdan farklı doğal sayılar olmak üzere her a ve b doğal sayıları için,
1- a m  a n  a m  n
2- a n  bn  (a  b)n
3- (a m )n  a mn  a nm dir.
Ġspat:
1- a m  a n  a  a  a  ...  a  a  a  a  ...  a  a m  n
35
m  tane
n  tane
2- a  b  (a  a  a  ...  a)  (b  b  b  ...  b)  (ab)  (ab)  (ab)  ...  (ab)  (ab) n
n  tane
n  tane
n  tane (ab)
n
n
n  tane
3- (a )  a  a  a  ...  a  a
n  tane
m
m n
m
m
m
m  m  m ... m
 a mn olur.
Örnek:
46  511 sayısı kaç basamaklıdır?
Çözüm:
46  511  (22 )6  511  212  511  2  211  511  2  (2  5)11  2 1011  200.000.000.000 dir.
Yani
sayımız 12 basamaklıdır.
Örnek: 812  26  4n  168 ise n kaçtır?
Çözüm:
2 
3 12
 26   22   168  236  26  22 n  232
n
236  6  22 n  32 tabanlar aynı olduğundan üslerinde eĢit olması gerekir.
Yani, 36  6  2n  32 dir. 42  32  2n  10  2n  n  5 dir.
Örnek: 3n  a ise 27 2 n 1 sayısının a cinsinden değeri nedir?
Çözüm:
 27 
2 n 1
  33 
2 n 1
 36 n  3  36 n  33   3n   27  a 6  27  27  a 6 dır.
6
1 den n ye kadar doğal sayıların 1  2  3  ...  n çarpımı kısaca n! ile gösterilir.
n! ifadesi '' n faktöryel '' biçiminde okunur. Örneğin,
3!  1  2  3  6 , 4!  1  2  3  4  24 , 5!  1  2  3  4  5  120 olur.
DOĞAL SAYILAR KÜMESĠNDE BÖLME ĠġLEMĠ
a, b  ve b  0 olmak üzere a  b  c olacak şekilde bir c doğal sayısı varsa,
a nın b ye bölümü c dir denir. a nın b ye bölümü c ise bu , a : b  c biçiminde gösterilir.
Buna göre, a : b  c  a  b  c dir.
a : b  c olduğunda '' a sayısı b nin c katıdır'' veya '' a sayısı b ye tam bölünür'' yada
'' b sayısı a sayısını böler'' denir.
b sayısı a sayısını bölerse bu b a biçiminde gösteririz. Ve '' b böler a '' biçiminde okuruz.
Buna göre, 2 6, 3 12,
5 55 tir.
a  b  c olduğunda, b ve c sayılarına a sayısının çarpanları denir. Örneğin,
36
5  6  30, 5  7=35 olduğu için 32 sayısı 5 ile tam olarak bölünemez.Bir başka deyişle 5
ile çarpıldığında 32 sayısını veren bir doğal sayı yoktur.Fakat 32=6  5+2 olarak yazılabilir.
Bu bize 32 sayısının 5 ile bölündüğünde kalanın 2 olacağını gösterir. Genel olarak,
a ve b doğal sayıları verildiğinde a  b  c  r , 0  r  b olacak şekilde bir c ve r tamsayısı
vardır.Bu c sayısına bölüm, r sayısına ise kalan denir.
bölünen
a
bc
r
bölen
b
c
bölüm
kalan
a  b  c  r eĢitliğine bölme eĢitliği denir.
Örnek:
15
5
15
3
0
r 0
21
20
1
r 1
5
4
32
5
30
6
2
r2
43
40
3
r 3
5
8
54
5
50
10
4
r4
r  0 ise '' b sayısı a saysını tam olarak böler'' b a biçiminde gösterilir.
Tanım: a, b, c birer doğal sayı olsun. c a ve c b ise c sayısı a ve b nin bir ortak bölenidir.
Örneğin, 1,3,6 sayıları 18 ve 24 sayısının ortak bölenleridir.
Tanım: a ve b doğal sayılarının doğal sayı olarak ortak böleni sadece 1 ise a ve b sayıları
kendi aralarında asaldır denir. Bu durum  a, b   1 biçiminde gösterilir.
Örneğin, 2 ile 5, 12 ile 17, 14 ile 27 sayıları kendi aralarında asaldır.
ÖRNEK: 24 ve 56 sayılarının doğal sayı olarak ortak bölenlerini bulunuz.
Kaç tane ortak böleni vardır? Ortak bölenlerin en büyüğü kaçtır?
Çözüm:
24 doğal sayısının doğal sayı bölenleri; 1,2,3,4,6,8,12,24 tür.
56 doğal sayısının doğal sayı bölenleri; 1,2,4,7,8,14,28,56 dır.
Buna göre ortak bölenlerin kümesi {1,2,4,8} olur. Ortak bölenlerin
en büyüğü ise 8 dir.
37
Örnek:
a, b 
ve a  b  11 olduğuna göre a  b çarpımının alacağı en büyük ve en küçük
değer kaçtır.
Çözüm:
Burada en büyük değeri bulmak için çarpanların biribirine olabildiği kadar yakın
olması gerekir. Yani, 5  6  11 dir. O halde 5  6  30 en büyük değer, en küçük değer içinde
çarpanların biribirine olabildiğince uzak olması gerekir. 0  11  11 dir. Çarpımı ise 0  11  11 dir.
2- Tamsayılar :
Gündüz hava sıcaklığı 8 0C olan bir ilimizde gece sıcaklık 12 0C düĢüyor. Gece
hava sıcaklığı kaç derecedir? diye sorulduğunda cevap olarak (8-12)
derecedir.Fakat( 8-12) sayısı bir doğal sayı değildir.Çünkü, 12 ye eklendiğinde 8 ‘i
veren bir doğal sayı yoktur.
x  a  b biçimindeki denklemlerin bazılarının doğal sayılar kümesinde çözümü
var, bazılarının çözümü yoktur.örneğin, x  8  6  x  6  8  x  2  dir.O
halde daha geniĢ sayı kümesine ihtiyacımız vardır.ĠĢte bu sayı kümesi ise,
 {..., 3  2, 1,01,2,3,...} dir.
 {..., 3  2, 1,01,2,3,...} kümesinin her bir elemanına bir tamsayı denir.
Tamsayılar kümesi üç kısımdan oluĢur.
Tamsayılar Kümesi
Negatif Tamsayılar
kümesi

 {..., 3, 2, 1}
-4
-3
-2
-1
Sıfır elemanı
{0}
Pozitif Tamsayılar
kümesi

 {1,2,3,...}
0
1
2
3
38
4
TAMSAYILARDA TOPLAMA ĠġLEMĠ
Tamsayılarda toplama iĢlemi aĢağıdaki kurallara göre yapılır.
1. Toplanan sayıların iĢaretleri aynı ise sayılar toplanır ve toplamın iĢareti sayıların
iĢaretleri ile aynıdır.
2. Toplanan sayıların iĢaretleri farklı ise bu durumda büyük sayıdan küçük sayı
çıkarılır ve büyük sayının iĢareti sonucun iĢareti olur. Ġkiden fazla tamsayı varsa
aynı iĢaretli sayılar öncelikle toplanır ve sonra ve iĢleme devam edilir.
Örnek: Aşağıdaki toplama iĢlemlerini yapınız.
a- (28)  (22)  ?
b- (43)  (67)  ?
c- (12  (34)  (35)  ?
d- (38)  (14)  (60)  ?
Çözüm:
a- (28)  (22)  (50)
b- (43)  (67)  (111)
c- (12  (34)  (35)  (46)  (35)  ( 11)
d- (38)  (14)  (60)  (52)  (60)  (8)
TAMSAYILARDA ÇIKARMA ĠġLEMĠ
Tamsayılarda çıkarma iĢlemi,
a  b  a  (b) biçiminde tanımlanır.
Örneğin, (3)  (5)  (3)  (5)  (8)  8 olur.
Örnek:
a- (34)  (20)  ?
b- (50)  (32)  ?
c- (44)  (34)  (100)  ?
d- (8)  (12)  (20)  (48)  ?
Çözüm:
a- (34)  (20)  (34)  (20)  (14)
b- (50)  (32)  (50)  (32)  (18)
c- (44)  (34)  (100)   (44)  (34)  (100)   (44)  (34)  (100)  (10)  (100)  (11
d- (8)  (12)  (20)  (48)   (8)  (12)   (20)  (48)   (8)  (12)   (68)
 (4)  (68)  (4)  (68)  (64)
39
TAMSAYILARDA ÇARPMA ĠġLEMĠ
Tamsayılarda çarpma iĢlemi aĢağıdaki kurallara göre yapılır.
Sayılar aynı iĢaretli ise çarpımın iĢareti '' '' , sayılar zıt iĢaretli ise çarpımın iĢareti '' ''
dir.
AĢağıdaki örnekleri inceleyelim.
a) (3)  (5)  (15)
b) (+6)  (+12)=(+72)
c) (8)  (7)  (56)
d) (30)  (2)  (60)
TAMSAYILARDA BÖLME ĠġLEMĠ
TANIM:
a, b, c  ve b  0 olsun. a  b  c ise ‘’ b sayısı a tamsayısını böler ’’ ve
bölüm c dir denir.Bunu, a : b  c biçiminde gösteririz.
Tamsayılarda bölme iĢlemi aĢağıdaki kurallara göre yapılır. Sayılar aynı iĢaretli
ise bölümün iĢareti '' '' , sayılar zıt iĢaretli ise bölümün iĢareti '' '' dir.
(8)
(15)
(20)
(100)
 (2)
 (5)
 (5)
 (10)
(4)
(3)
(4)
(10)
Örnek:
15  (15) : 3  (2)  (4)  ifadesinin değerini hesaplayınız.
Çözüm: 15   (5)  (8)   15  (5)  (8)   15  (13)  (2)  2
Örnek:  (18) : (6)  : (3) ve (-18):( 6):(-3)  işlemlerini hesaplayınız.
Çözüm:
(18) : (6) : (3)  (3) : (3)  (1)  1
(-18): (6):(-3)  (18) : (2)  (9)  9
O halde,  (18) : (6)  : ( 3)  (-18):( 6):(-3)  dir.
3- Rasyonel Sayılar:
a
 { / a, b  , b  0} dir
b
a
Her tamsayı aynı zamanda rasyonel sayıdır. Burada,
b
pay
dır.
payda
Tamsayılar ve doğal sayılarla bir bütünün tamamını veya katlarını belirtebiliriz. Fakat
bir bütünün parçalarını veya tam olmayan kısımlarını ancak rasyonel sayılarla belirleriz.
Bundan dolayı rasyonel sayılara kesirli sayılarda denir. Örneğin, bir bütünü 5 parçaya
40
ayırıp 3 parçasını belirlersek, bunu
3
rasyonel sayısıyla gösteririz. Kesirleri 3 gruba
5
ayırabiliriz.
a. Basit Kesir: Payı paydasından küçük olan kesirlere basit kesir denir. Örneğin,
5 18 3
,
, gibi
7 45 5
b. BileĢik Kesir: Payı paydasından büyük olan kesirlere bileĢik kesir denir.
4 18 9
Örneğin, , , gibi,
3 11 5
c. Tamsayılı Kesir: Bir tamsayı ve bir basit kesirden oluĢan kesre tamsayılı kesir
1 3 4
denir. 2 , 4 , 6 gibi,
3 7 9
b ac b
Bir tamsayılı kesir, a 
eĢitliği kullanılarak bileĢik kesre
c
c
dönüĢtürülür.
ÖRNEK:
4 5  3  4 19
a. 3 

5
5
5
2
57  2
37
b. 7  

5
5
5
22 34  25  22 872
c. 25


34
34
34
KESĠRLERDE SADELEġTĠRME VE GENĠġLETME
a
a k c
1. Bir k  sayısı a ve b nin ortak böleni olsun. k    olarak
b k b d
k
a
c
yazılabilir. Yani, kesri daha sade olarak
Ģeklinde yazılarak daha kullanıĢlı
b
d
duruma getirilebilir.
a ak e
a
2. k   olmak üzere 
kesrine
nin k ile geniĢletilmesi denir.

b
b bk
f
ÖRNEK:
144 144 : 2 72
96 8  12 12 6  2 6
a.
c.






180 180 : 2 90
80 8  10 10 5  2 5
b.
300 3  100 3


800 8  100 8
d.
3 3  6 18
(geniĢletme)


5 5  6 30
ÖRNEK:
Değeri
2
1
olan bir kesrin payından 1 çıkarılır ve paydasına 1 eklenirse kesir
3
2
ye eĢit oluyor.
Bu kesrin pay ve paydasının toplamı nedir?
ÇÖZÜM:
41
2 2k
olarak yazılabilir.

3 3k
2k  1 1
  2  2k  1  1   3k  1  4k  2  3k  1  4k  3k  1  2  k  3
3k  1 2
2k 2  3 6
Buna göre, kesrimizin değeri

 dur. Toplamları ise 6  9  15 dir.
3k 3  3 9
5
ÖRNEK: Bir kesrin değeri dir. Bu kesrin payına 6 eklenir ve paydasından 6
7
7
çıkarılırsa kesrin değeri oluyor.Bu kesri bulunuz.
5
ÇÖZÜM:
5 5k
Söz konusu kesir 
biçimindedir.
7 7k
7 5k  6

 5  5k  6   7  7 k  6   25k  30  49k  42
5 7k  6
72
 25k  49k  42  30  24k  72  k 
 3  k  3 olur.
24
5  3 15
Ġstenilen kesir

dir.
7  3 21
RASYONEL SAYILARDA DÖRT ĠġLEM
1. TOPLAMA VE ÇIKARMA ĠġLEMĠ
Ġki rasyonel sayının toplamında veya çıkarılmasında önce paydalara bakarız.
Eğer paydalar eĢit ise paylar toplanır veya çıkarılır ve ortak payda yazılır. Eğer
paydalar eĢit değilse öncelikli olarak paydalar eĢitlenir ve iĢlem yapılır.
ÖRNEK:
12 3 12  3 15
a.
 

8 8
8
8
34 67 34  67 33
33
b.




45 45
45
45
45
4 7 3  4  1  7 19
 

c. 6 18
18
18
(3) (1)
4 9  2  4 18  4 14
d. 2  


9
9
9
9
23 78  5  23 390  23 413
e. 5 



78
78
78
78
2. ÇARPMA VE BÖLME ĠġLEMĠ
Ġki kesrin çarpımında, paylar çarpılır paya ve paydalar çarpılır paydaya yazılır.
Bölme iĢleminde ise birinci kesri aynen yazar ikinci kesrin paya ve paydasını yer
değiĢtirerek çarpma iĢlemi yapılır.
ÖRNEK:
2 4 24 8
a.
 

3 5 3  5 15
42
1 12 1  12 12



2 38 2  38 76
12 20 3  4 4  5 3 5
3  5 15
c.



  

28 44 4  7 4  11 7 11 7  11 77
3 14 3 2  7
3 2 7
2
d.

 
   
7 45 7 3  15 7 3 15 15
ÖRNEK:
AĢağıdaki iĢlemleri yapınız.
8 2 8 3 24 12
a.
:   

5 3 5 2 10 5
4 4 4 5
b.
:   1
5 5 5 4
b.
2 10 3 10  3 30
  

 15
3 1 2 1 2
2
11
11 1 11
d.
:3   
12
12 3 36
c. 10 :
ĠġLEM ÖNCELĠĞĠ
Verilen bir problemde birden fazla iĢlemin bir arada yapılması gerekiyorsa bu
iĢlemler verilen sıraya göre yapılmalıdır.
1. Parantez varsa öncelikli olarak parantez içinden baĢlanır.
2. Üslü sayı varsa üs alma iĢlemi yapılır.
3. Bölme veya çarpma iĢlemi yapılır.
4. Toplama veya çıkarma iĢlemi yapılır.
ÖRNEK:
1
1  1
: 1    ?
2  2
ÇÖZÜM:
1
1  1
1 1
1 2
: 1    1  :    1    1  1  0
2  2
2 2
2 1
ÖRNEK:
2
3
5

2
3
5

2 1 2 5 2 10 2 50 2  50 48
48
      



3 5 1 3 15 3 15 15
15
15
15
(5)
ÖRNEK:
1
1
1
1
ÇÖZÜM:
işleminin sonucunu bulunuz.
1
1
3
43
1
1
1
1
ÖRNEK:
=1+
1
1
3
1
1
1
4
3
 1
1
3
1
4
 1
1
 1 4  5
1
4
1
1
1
6
7 işleminin sonucunu bulunuz.
5
6
1
1
6
7
8
1
1 7
1
1
6
7  6  7  7  8  15 olur.
5
6 1
1 1 1
1
1
6
7
6 7
1
ÇÖZÜM:
ÖRNEK:
1 
 1  1  1 
1    1    1      1 

 2   3   4   100   1 ise x  ?
1
5
x
ÇÖZÜM:
1 
 1  1  1 
1    1    1      1 

 2   3   4   100   1
1
5
x
99
 1   2   3   99  1 2 3
1
  
      

1
2
3
4
100
2
3
4
100
      

 100 
1
1
1
5
x
x
x
1 x 1
x
1
  
  5 x  100  x  20 bulunur.
100 1 5 100 5
ÖRNEK:
1

12   4    4
2
3


ifadesini hesaplayınız.
3
2
1
4
ÇÖZÜM:
1

 12  1 
 11 
12   4    4
12  
4
12     4

2
2
2
3

 3 
3





3
3

4

1
2
2
2
1
4
4
4
12  11
4
2  4 
8 44  4
3
   


 8  20  12 dir.
1  1 
2
1
2
ÖRNEK:
44
1 
 1  1  1 
1    1    1      1  
 3   4   5   50 
1 
 1  1  1 
1    1    1      1  
 5   6   7   48 
ÇÖZÜM:
1  4 5
 1  1  1 
1    1    1      1   3 
4
 3   4   5   50  
1  4 5
 1  1  1 

1    1    1      1  
 5   6   7   48  5 6
ifadesini hesaplayınız.
6
51
51

51 48
5
50
 3  
 17  12  204
4
3 4
6
47
 
48
48
7

ÖRNEK:
5
3
2
x 1
ÇÖZÜM:
kesrini tanımsız yapan x değerlerini bulunuz.
x  1  0  x  1 olmalıdır. Ayrıca 2 
3
 0 olmalıdır.
x 1
1
1
olacağından biz x   almalıyız.
2
2
1
O halde verilen kesri tanımsız yapan x değerleri 1 ve - dir.
2
RASYONEL SAYILARDA SIRALAMA
TANIM:
2 x  2  3  0  2 x  1  x  
ONDALIK SAYILAR
Paydası 10,100,1000, … gibi 10’ nun kuvvetleri Ģeklinde olan kesirlere ondalık
kesir denir.
3 46
346
gibi,
,
,
10 1000 10000
Bir kesrin payını paydasına böldüğümüzde ondalık kesir elde edilir.
Bir kesri ondalık sayıya iki Ģekilde dönüĢtürürüz.
I. Pay paydaya bölünür.
II. Kesir geniĢletilerek payda 10’nun kuvveti Ģekline getirir.
ÖRNEK:
7 7  5 35


 3,5
a. 2 2  5 10
b.
78 78  25 1950


 19,50
4
4  25
100
45
c.
17 17  25 425


 4, 25
4
4  25 100
NOT: Ondalık sayılar tam kısım ve ondalık kısımlar olmak üzere iki kısımdan
oluĢur.Tam kısımdaki basamaklar birler,onlar,binler,… basamağı olarak, ondalık
kısımdaki basamaklarda onda birler, yüzde birler, binde birler … basamağı olarak
adlandırılır.
ÖRNEK:
23,18
2
3
1
8
Onlar basamağı
Birler basamağı
Onda birler basamağı
Yüzde birler basamağı
BĠR ONDALIK SAYIYI KESRE ÇEVĠRME
Bir ondalık sayı kesre çevrilirken, ondalık sayıdan virgül silinerek kesrin payına
yazılır ve paydaya ise ondalık kısımdaki basamak sayısı kadar 10’nun kuvveti yazılır.
ÖRNEK:
2 1
345 69
2. 3, 45 
3.


10 5
100 20
8
4
2
1
dir.
0, 0008 



10000 5000 2500 1250
1. 0, 2 
ONDALIK SAYILARDA TOPLAMA VE ÇIKARMA ĠġLEMĠ
NOT: Burada yapacağımız sayılarda bulunan virgüllerin alt alta gelmesidir ve
normal toplama ve çıkarma iĢlemi yapılır.
15, 457
 23,563
39, 020
45789,9785
_ 478, 0482
0, 78945
 4,8475
457, 01

5, 63695
6,589
4, 45789
_ 4, 7895
461, 46789
1, 7995
45311,9303
ONDALIK SAYILARDA ÇARPMA VE BÖLME ĠġLEMĠ
Ondalık sayılarda çarpma iĢleminde virgüllerin alt alta gelmesi gerekmez. Virgül
yokmuĢ gibi çarpım yapılır ve virgül kaydırılarak sonuç bulunur.
Tam kısımdan sonra
2 basamak var
46
ÖRNEK:
4, 75
 2, 22
Tam kısımdan sonra 2
basamak var
950
950
 950
Sonuçta 4 basamak
virgül sola kaydırılır.
10,5450
ÖRNEK:
1, 2 : 0,16  ? iĢleminin sonucunu bulunuz.
120 16
120
7,5
00
ÖRNEK:
3,5
?
0, 007
ÇÖZÜM:
3,5
3500

 500
0, 007
7
ÖRNEK:
1, 2
0,1
1


?
0, 02 0, 02 10
ÇÖZÜM:
1, 2
0,1
1 120 10 1

 
   60  2  0,1  58.1 olur.
0, 02 0, 05 10
2
5 10
4-Ġrrasyonel Sayılar:
a, b 
ve b  0 olmak üzere
a
Ģeklinde yazılamayan sayılara Ġrrasyonel
b
sayılar denir.
2 = 1.41421356... ve log 6 5 , e ,  sayıları birer irrasyonel sayılardır.
Ne kadar rasyonel sayı var ise, aynı Ģekilde o kadar irrasyonel sayı vardır.
ġimdi
2 sayısının bir rasyonel sayı olmadığını görmeye çalıĢalım.
47
Bu sayıya x diyelim. x  2  x2  2 olarak yazılabilir. Eğer x bir rasyonel sayı
m
ise x 
dir.
n
( Burada m ve n sayılarının 1 dıĢında hiçbir ortak böleninin olmadığını
varsaymaktayız.)
2
m
 2  2n2  m2 dir. Burada birinci taraf çift olduğundan ikinci tarafında çift
2
n
olması gerekir. Yani m 2 ve m çifttir. k  olmak üzere m  2k olsun.
2
2n 2   2k 
2n2  4k 2
n2  2k 2
Ģeklinde yazılabilir. Bu ise bize n2 ve n nin de çift olmasını gerektirir. Bu durumda
m ve n nin çift olduğu ortaya çıkar. 2 de n ve m sayılarının ortak böleni dir
diyebiliriz. Halbuki biz n ve m sayılarına aralarında asal demiĢtik. Bu durum bir
m
çeliĢkidir. O halde 2 sayısını biz
Ģeklinde yazamayız. Bu durumda 2 sayısı
n
bir rasyonel sayı değil, bir irrasyonel (rasyonel olmayan) sayıdır.
2
1 br
0
1
2
2
1 br
4- Reel Sayılar ( Real Numbers) :
  t
t



ve  t   dir. O halde ,
  Ģeklinde yazılabilir.
t
48
Genellikle uygulamalarda reel sayılar kullanılır. Bizim kullandığımız sayıların
tamamına yakını reel sayıdır. a  b  p olacak Ģekilde pozitif bir p sayısı varsa bu
durumda a sayısı b den daha büyüktür denir. Bunu,
a  b ( a büyük b) veya b  a (b küçük a) Ģeklinde gösterebiliriz. a  b ve a  b ise
a  b Ģeklinde gösterilir.
Hernangi bir x 
için, x  0 ise x sayısı pozitiftir. x  0 ise x sayısı negatiftir.
1  Negatif iki sayının çarpımı pozitiftir. Yani, x  0 ve y  0 olsun. x  y  0 ,
2  Pozitif iki sayının çarpımı pozitiftir. Yani, x  0 ve y  0 olsun. x  y  0,
3  Eğer sayılar zıt iĢaretli ise bu durumda bu sayıların çarpımı negatiftir. x  y  0 dır.
MUTLÂK DEĞER
Sayı ekseni üzerinde -3, -2, -1, 0, 1,2,3 sayılarına karĢılık gelen A, B, C, O, D, E, F
noktalarını göz önüne alalım.
3 br
2 br
A
-3
B
-2
C
1 br
-1
O
0
D
E
F
1
2
3
OA  3, OB  2, OC  1,
OF  3, OE  2, OD  1 dir.
Yani, verilen tüm noktaların O noktasına olan uzaklığı devamlı pozitiftir.
TANIM:
Bir a reel sayısına karĢılık gelen noktanın O baĢlangıç noktasına olan
uzaklığına, a sayısının mutlak değeri denir ve a ile gösterilir.
OA  OF  3  3  3  3
OB  OE  2  2  2  2
OC  OD  1  1  1  1 olacaktır.
Bir a sayısının mutlak değeri,
a , a  0
a 
dir.
  a, a  0
Örnek:
AĢağıdaki ifadeleri hesaplayınız.
a)
3 1  1 2  3  2
b)
5
3
2
1 

2
2
2
Çözüm:
49
a)
3  1  0 olduğundan
3 1  3 1 ,


1  2  0 olduğundan 1  2   1  2  2  1 ,
3  2  0 kolduğundan
3 1  1 2 
3  2  3  2 dir. O halde,
3 2 
3 1 2 1
3  2  2 2  2 olur.
5
5
3
1  1 
,
2
2
2
b)
 3 2 2
3
2
3 2 2
3 2 2 2 2
3



  




  
2
2
2
2 
2
2
2
2
2
 2
( 2)
=
3 2 2
3 3 2 2  3



olarak bulunur.
2
2
2
2
ALIŞTIRMALAR
1.
2.
3.
4.
5.
6.
7.
1 
2

3    3  
3 
3

?
1 
6

2    3  
5 
5

1
2
 ? işleminin sonucunu bulunuz.
1
1
1
1
2
6
3
?
1
2
2
1
3
1  1
4  : 1    ?
2  2
1
5a 
?
1
1
1
1
a
1 1

3 6 :  2  11   ?

3 2 
5

4 3
 0,362  0,638   0,655  0, 445  ?
50
15
0, 6
1


?
0, 03 0, 002 10
0, 03  x
9.
2 x?
0, 25
8.
4-BÖLÜM
AÇIK ÖNERMELER VE NĠCELEYĠCĠLER
1- Açık önermeler
Mantık kısmında önermeler ve önermelerin doğruluk değerlerini gördük. ġimdi
ise, içinde değiĢken bulunan ve değiĢkenlere verilen değerlere göre doğru veya yanlıĢ
olan ifadelere bakacağız.
'' x, bir canlıdır.''
Ġfadesin de yerine insan yazarsak,
''Ġnsan , bir canlıdır.''
bu önerme doğru olur. Fakat x yerine TaĢ yazarsak,
''Taş , bir canlıdır.''
bu durumda yanlıĢ önerme olur. x yerine çeĢitli değerler yazılabilir. x ’e değiĢken
denir.
Tanım:
Ġçinde en az bir değiĢkeni bulunan ve bu değiĢkenlere verilen değerlere göre
doğru ya da yanlıĢ olan ifadelere açık önerme denir. Ġçinde sadece x değiĢkeni bulunan
açık önerme genel olarak p( x) ile, x , y değiĢkeni bulunan ifadeler ise p( x, y) ile
gösterilir.
p( x) : x , bir canlıdır. x yerine insan yazarsak,
p(insan) : insan , bir canlıdır  p(insan)  1 ,
x yerine taş yazarsak,
p(taş) : taş , bir canlıdır  p(taĢ)  0 dır.
51
p( x) 'de x yerine değer yazmadan önermenin doğruluğu veya yanlıĢlığı için bir Ģey
söylenemez. p( x, y) içinde x ve y yerine bir Ģey yazmadan önermenin doğruluğu veya
yanlıĢlığı için bir Ģey söylenemez.
Örnek:
F  {0,1,3,4} kümesinde p( x) : x  2  0 açık önermesinin doğruluk değerini bulalım.
Çözüm:
p(0) : 0  2  0  2  0  p(0)  1  Doğru
p(1) :1  2  0  1  0  p(1)  1  Doğru
p(3) : 3  2  0  1  0  p(0)  0  YanlıĢ
p(4) : 4  2  0  2  0  p(0)  0  YanlıĢ
olduğundan doğruluk kümesi D= 0,1 dir.
Örnek:
Tamsayılar kümesinde , p( x) : x2  0 açık önermesinin doğruluk değerini bulalım.
Çözüm:
Sıfırın karesi 0 ve diğer tüm tamsayıların kareleri pozitif olduğundan dolayı x 
p( x) : x 2  0 dır. O halde doğruluk kümesi D 
Örnek:
için
dir.
E  {0,1,2,3,4,5} kümesinde p( x, y) : x  y  2  0 açık önermesini doğru yapan
 x, y  sıralı ikililer kümesini bulalım.
Çözüm:
x  0 için y  2 olup , ( x, y)  (0, 2)  D
x  1 için y  3 olup , ( x, y )  (1,3)  D
x  2 için y  4 olup , ( x, y )  (2, 4)  D
x  3 için y  5 olup , ( x, y )  (3,5)  D
x  4 için y  6  D olup , ( x, y )  (4,6)  D
x  5 için y  7  D olup , ( x, y )  (5,7)  D dir.
Önermenin doğruluk değeri, D  (0, 2),(1,3),(2, 4),(3,5) dir.
2- Niceleyiciler
Bir özelliğin, bir kümenin bütün elemanları için mi, yoksa aralarında bazıları
için mi geçerli olduğunu göstermek üzere matematikte kullanılan bazı semboller vardır.
Örnek:
E  1,1,2,3 evrensel kümesinde , p( x) : x 2  0 açık önermesinin her x  E için doğru,
q( x) : x  1 açık önermesinin bazı değerleri için doğru olduğu açıktır.
52
Verilen bir özelliğin, bir kümenin bütün elemanları için geçerli olduğu gibi,
kümenin bazı elemanları içinde geçerli olduğundan söz edebiliriz. ĠĢte önüne gelen
elemanların niceliğini ( çokluğunu ) belirten ‘’ her’’ ve ‘’ bazı’’ sözcüklerine niceleyici
denir.
‘Her’ niceleyicisi, önüne geldiği elemanların tamamını anlattığından, bu
niceleyiciye evrensel niceleyici, en az bir elemanın varlığı belirten ‘bazı’ niceleyicisine
varlıksal niceleyici denir.
Evrensel niceleyici  sembolü ile varlıksal niceleyici  sembolü ile gösterilir.
ġimdi bunların matematikte nasıl kullanıldığına bakalım.
‘Her tamsayının karesi sıfıra eĢit veya sıfırdan büyüktür.’ Önermesini , x 
olmak üzere , x  için x2  0 biçiminde yazarız.
‘Bazı doğal sayıların karesi tektir.’ Önermesini , ’ x  , x için x2 tektir. ’
biçiminde yazarız.
Bir önermenin, kendisi doğru ise değil inin yanlıĢ olduğunu ve kendisi yanlıĢ ise
değil inin doğru olduğunu biliyoruz.
p( x) : x  , x 2  0 önermesinin doğru olduğunu biliyoruz. Bu önermenin
değili ise,
x  içinx2  0 olur. Buna göre, p( x) : x  , x 2  0   x  içinx 2  0 dir. Bu

 

durumu genelleĢtirelim.
 x, p( x)    x, p( x)  ve  x, p( x)   x, p( x)  olduğunu görebiliriz.
Örnekler
1. x  ,2 x2  1  0 önermesinin olumsuzu x  ,2 x 2  1  0 dır.
2.
'' Bazı çift doğal sayılar asaldır'' önermesinin olumsuzu,
'' Her çift doğal sayı asal değildir.''olur.
3.
 x, p( x)  x, q( x)   x, p( x)  x, q( x)  tir.
4.
x, x
2
 1  0  x,2 x  1  0    x, x 2  1  0  x,2 x  1  0  dır.
53
5.BÖLÜM
ORAN VE ORANTI
Tanım: b  0 olmak üzere
a
ifadesine oran denir. Eğer iki oran birbirine
b
eĢitse, buna orantı denir.
1
2
1 in 5 'e oranı  ,  2 nin 3'e oranı  ,… gibi yazılabilir. Orantıyı
4
3
a c
a c
verelim. Burada,  dır.
içler dıĢlar çarpımı yaptığımızda

b d
b d
a  d  b  c yazılabilir. Örneğin,
3 12
3 12

 
 3  20  5  12  60  60 dır.
5 20
5 20
ORANTININ ÖZELLĠKLERĠ
1.
ab cd
a b c d

veya

,
b
d
b
d
54
a b a b

cd cd
a c ac
3.
 
dir.
b d bd
a c e
4.
   ...  k dir. k ya orantı sabiti denir.
b d f
Burada, a  b  k , c  d  k , e  f  k , .... olarak yazılabilir.
2.
ġimdi bu özelliklere örnek verelim.
a c
4 12
4  7 12  21 11 33
  


 
b d
7 21
7
21
7 21
1.
a c
4 12
4  7 12  21
3 9
3 9
  




 
b d
7 21
7
21
7
21
7 21
2.
a c
4 12
47
47
11 3
11 3
ise
  





b d
7 21
12  21 12  21 33 9
33 9
3.
4 12 4  12 16
dir.



7 21 7  21 28
4.
10 15 50
5


 k  dir.
16 24 80
8
5
5
5
10  16  , 15  24  , 50  80  dir.
8
8
8
Örnek:
2 3x  4

 x?
5
20
Çözüm:
2 3x  4

 2  20  5   3 x  4   40  15 x  20
5
20
20 4
 40  20  15 x  20  15 x  x 
 olur.
15 3
Örnek:
x y z
  olduğuna göre 3x  y  z  72 olduğuna göre x, y, z sayılarını bulunuz.
3 5 2
Çözüm:
x  3k , y  5k , z  2k olarak yazalım.
55
3x  y  z  72  3  3k  5k  2k  72  9k  5k  2k  72
72
12k  72  k 
 6 dır.
12
x  18 , y  30 ve z  12 olarak bulunur.
Orantı 3 grupta incelenebilir.
ORANTI
DOĞRU
TERS
ORANTI
ORANTI
BİLEŞİK ORANTI
1- DOĞRU ORANTI
Birbiri ile orantılı iki ifadeden biri artarken diğeri de artıyorsa bu
ifadeye doğru orantı denir.
Doğru orantının özellikleri
A ve B sayıları doğru orantılı ve k orantı sabiti olsun.
1.
A  k  B dir.
2. x, y, z sayıları sırasıyla a, b, c sayılarıyla doğru orantılı ise
3.
x y z
   k olur.
a b c
AkB
A ile B sayıları doğru orantılı ise
A ile B sayıları arasındaki bağıntının
grafiği yandaki Ģekilde olduğu
gibidir.
2- TERS ORANTI
Birbiri ile orantılı iki ifadeden biri artarken diğeri azalıyor ise bu iki
ifadeye ters orantı denir.
Ters orantının özellikleri
k
veya k  A  B dir.
B
2. x, y, z sırasıyla a, b, c ile ters orantılı ise ax  by  cz  k olur.
A ve B sayıları ters orantılı ise aralarındaki
3.
bağıntının grafiği aĢağıdaki gibidir.
1. A ve B sayıları ters orantılı ise A 
56
A
A
k
B
B
Not: A sayısı B sayısı ile doğru, C sayısıyla ters orantılı ise A 
B
 k olur.
C
Örnek:
y sayısı x  2 ile doğru, x  3 ile ters orantılıdır.
y  36 için x  4 oluyor ise x  5 için y kaç olur.
Çözüm:
y sayısı x  2 ile doğru, x  3 ile ters orantılı ise,
x2
42
y
k  36 
k  36  6k  k  6 dır.
x3
43
x2
52
7  6 42
y
ky
6  y 

 21 dir.
x3
53
2
2
Örnek:
Üç ortaklı bir iĢletmede ortakların sermayeleri 8,9 ve 10 sayıları ile orantılıdır.
ĠĢletmenin yıllık kârı 54000 lira ise her bir ortağın kâr miktarını bulunuz.
Çözüm:
Ortakların kâr paylarını x, y, z ile gösterirsek, x, y, z sayıları 8,9,10 sayıları ile
doğru orantılıdır.
x y z
 
 k  x  8k , y  9k , z  10k toplam kâr 54000 lira ise
8 9 10
x  y  z  8k  9k  10k  54000  27k  54000  k  2000 liradır.
x  8  2000  16.000 lira 

O halde, y  9  2000  18.000 lira  dır.
z  10  2000  20.000 lira 
Örnek:
57
Un  u  , Yağ  y  , ġeker  ş  olmak üzere,
u 5
y 3

,
 oranlarında karıĢtırılarak 580 gr helva yapılıyor.
y 2
ş 4
Bu helvada kaç gr yağ kullanılmıĢtır?
Çözüm:
Her iki oranda da ortak değiĢken yağdır. Yağ miktarlarını eĢitlemek
için birinci oranı 3 ile ikinci oranı 2 ile genişletelim.
u 53
y 3 2 6

,

 dir.
y 23
ş 42 8
O halde, y  6k , u  15k , ş  8k dır.
u  y  ş  6k  15k  8k  580  29k  580  k  20 gr
O halde, y  6  20  120 gr , u  15  20  300 gr , ş  8  20  160 gr dır.
ORTALAMALAR
1. ARĠTMETĠK ORTALAMA
Tanım:
a1  a2  a3  ...  an
dir
n
ab
özel olarak iki sayı varsa aritmetik ortalama A 
2
abc
üç sayı varsa aritmetik ortalama A 
tür.
3
NOt: n tane sayının aritmetik ortalaması A ise bu sayıların toplamı n  A dır.
a1 , a2 , a3 ,..., an sayılarının aritmetik ortalaması A 
Örnek:
10,12,34,45,67,88,90 sayılarının aritmetik ortalamasını bulunuz.
Çözüm:
10  12  34  45  67  88  90  198 544
A

 68
8
8
Örnek:
Bir öğrencinin ilk iki sınav ortalaması 60, üçüncü sınavı 90 dir. Bu
öğrencinin üç sınavının ortalaması kaçtır?
Çözüm:
ilk iki sınavı ortalaması 60 ise toplamı 2  60  120 dir.
A
120  90 210

 70
3
3
58
Örnek:
30 kiĢilik bir sınıfta 14 kız öğrenci vardır. Kız öğrencilerin not
ortalaması 3.8, erkek öğrencilerin ise 3.4 ise bu sınıfın not ortalaması
kaçtır?
Çözüm:
14  3.8  53.2
16  3.4  54.4
A
14 kız öğrencinin not toplamı 
16 erkek öğrencinin not toplamı 
53.2  54.4 107.6

 3.58 dir.
30
30
Örnek:
Notlar
Öğrenci sayısı
1
3
2
6
3
12
4
13
5
6
Yukarıdaki tabloda bir sınıftaki öğrencilerin notlarının dağılımı verilmiĢtir. Bu
sınıfın bu sınavdaki aritmetik not ortalaması nedir?
Çözüm:
A
1  3  2  6  3  12  4  13  5  6 3  12  36  52  30 133


 3.32 dir.
3  6  12  13  6
40
40
2. GEOMETRĠK ORTALAMA
Tanım:
a1 , a2 , a3 ,..., an sayılarının geometrik ortalaması G olsun.
G  n a1  a2  a3  ..  an
Özel olarak, iki sayının geometrik ortalaması G  a  b
Üç sayının geometrik ortalaması G  3 a  b  c
Örnek:
2,4,5 ve 6 sayılarının aritmetik ve geometrik ortalamasını bulunuz.
Çözüm:
4  5  6  9 24
A

6
,
G  4 4  5  6  9  4 1080  5,73
4
4
Örnek:
1
a 2 , , a5 sayılarının geometrik ortalaması 3 ise a ' yı bulunuz.
a
Çözüm:
1
G  3 a 2   a5  3 a 6  a 2  a 2  3  a  3 tür.
a
3. HARMONĠK ORTALAMA
Tanım:
59
1 1 1 1 1
1
     ...  
H n  a1 a2 a3
an 
1 1 1 1 1
Özel olarak üç sayının harmonik ortalaması
    
H 3 a b c 
a1 , a2 , a3 ,..., an sayılarının harmonik ortalaması
Ġki sayının harmonik ortalaması
1 11 1
    dir.
H 2 a b
Örnek:
2,4,5 ve 6 sayılarının harmonik ortalamasını bulunuz.
Çözüm:
1 1 1 1 1 1 
1 11 1 1 1
1 1  30 15 12 10 
     
     
  

 
H 4 a b c d 
H 4 2 4 5 6
H 4  60 60 60 60 
1 1  67 
1
67
240
  

H 
 3.58
H 4  60 
H 240
67
Örnek:
60 ve 90 sayılarının harmonik ortalamasını bulunuz.
Çözüm:
1 1 1
1 
1 1 3
2 
1 1 5 
360
   
 

 
 72

 H 
H 2  60 90 
H 2  180 180 
H 2  180 
5
Örnek:
0.11
x

orantısında x  ?
0.21 0.63
Çözüm:
0.11
x
0.0693

  0.11   0.63   0.21  x  0.0693   0.21  x  x 
 0.33
0.21 0.63
0.21
11 100 x
11  63 693
veya

x

 0.33
21
63
2100 2100
Örnek:
x y z
  ve 3x  z  12 ise x, y, z sayılarını bulunuz.
3 6 7
Çözüm:
x y z
  ve 3x  z  12 ise,
3 6 7
x z
  7 x  3z  0 ve 3 x  z  12 elde edilir.
3 7
7 x  3 z  0
denklem sistemi çözülürse, x  18, z  42 ve y  36 dır.

3x  z  12
60
Örnek:
2 x  3 y  5z ve 4 x  3 y  360 olduğuna göre x  y  z  ?
Çözüm:
2 x  3 y  5 z ve 4 x  3 y  360 olduğundan
4 x  3 y  360 denkleminde 3 y yerine 2 x yazılabilir.
4 x  2 x  360  6 x  360  x  60, y  40, z  24 bulunacaktır.
x  y  z  60  40  24  124
Örnek:
y  3 sayısı, 2 x  1 ile doğru x  2 ile ters orantılıdır.
y  7 için x  1 oluyorsa x  4 için y  ?
Çözüm:
2x  1
2 1  1
1
k  73
k  4  k  k  12 olacaktır.
x2
1 2
3
2x  1
2  4 1
7
84
y 3
k  y 3
 12  y  3   12  y  3 
x2
42
6
6
y  3  6  y  3  6  9 bulunur.
y 3
Örnek:
40.000 m2 tarlayı 3 kiĢi 3,4 ve 7 sayılarıyla orantılı olarak paylaĢıyorlar.
Her birine kaç m2 düşer?
Çözüm:
x
1. kişi
 x  3k
x y z

y z olmak üzere,
   k   y  4k  x  y  z  3k  4k  7k  70.000
3 4 7
 z  7k
2. kişi 3. kişi

 x  15000 m 2
 x  3k


14k  70.000  k  5000 olacaktır.  y  4k   y  20000 m 2
 z  7k
 z  35000 m 2


Örnek:
6 tane sayının aritmetik ortalaması 15 dir. Bu sayılara hangi sayı eklenirse aritmetik
ortalama 18 olur?
Çözüm:
a1  a2  a3  a4  a5  a6
a  a2  a3  a4  a5  a6
 15  1
6
6
 a1  a2  a3  a4  a5  a6  90 olacaktır.
A
18 
 a1  a2  a3  a4  a5  a6   a7  18  90  a7  126  90  a
7
7
7
a7  36
Örnek:
61
kg fiyatları 3,6,5 ve 9 lira olan dört çeĢit çerez sırasıyla 6,4,10 ve 2 kg alınarak
bir karıĢım oluĢturuluyor. Bu karıĢımın kg fiyatı ne olur?
Çözüm:
KarıĢımın kg fiyatı 
3  6  6  4  5  10  9  2 18  24  50  18 110


 5 lira
6  4  10  2
22
22
Örnek:
2 x ve 3x sayılarının geometrik ortalaması 12 ise aritmetik ortalaması kaçtır?
Çözüm:
12 12 6
G  2 x  3x  12  6 x 2  12  x 6  x 

 2 6 dır.
6
6
a  2x  2  2 6  4 6
A
Örnek:
, b  3  2 6  6 6 olacaktır.
a  b 4 6  6 6 10 6


5 6
2
2
2
9,12,18 sayılarının aritmetik, geometrik ve harmonik ortalamasını bulunuz.
Çözüm:
A
9  12  18
 13
3
G  3 9  12  18  3 1944  12, 48
1 1 1 1 1 
1 1 4
3
2 
1 1 9 
1 1 1
    
  
 
 
   H  12 dir.

H 3  9 12 18 
H 3  36 36 36 
H 3  36 
H 3 4
Örnek:
Aritmetik ortalaması 10, geometrik ortalaması 6 olan iki doğal sayının harmonik
ortalaması nedir?
Çözüm:
ab
ab
A
 10 
 a  b  20
2
2
G  a  b  6  a  b  a  b  36 olacaktır.
a  b  20 
  a  18, b  2 olarak bulunur.
ab  36 
1 1 1 1
1 1 10
1 1 5
18
   
 

  H 
H 2  18 2 
H 2 18
H 2 9
5
62
6. BÖLÜM
ÖZDEġLĠKLER VE ÇARPANLARA AYIRMA
ÖZDEġLĠKLER
DeğiĢebilen, yani farklı değerler alabilen bir büyüklüğe değiĢken, her zaman
aynı kalan büyüklüğe sabit ve bazen değiĢebilen bazen de sabit olarak iĢlem gören
büyüklüğe parametre denir. Örneğin,
3x  5 ifadesinde 3,  5 sabitlerdir.
ax  5 de a parametredir.
DeğiĢken, parametre, sabit ve bunların farkları, toplamları, çarpımları,
bölümleri, kökleri vs. içeren ancak eĢitlik ve eĢitsizlik içermeyen ifadelere cebirsel ifade
denir. Cebirsel ifadede, , , , ,  gibi semboller bulunmaz.
Örneğin,
1
x  a, 3x  , bx  1 , x3  x 2  cx  1
2
gibidir.
DeğiĢkenlerin aldığı her değer için birbirlerine eĢit olan iki cebirsel ifadeye
özdeĢtir denir. Bu eĢitliğe özdeĢlik denir. ÖzdeĢ iki cebirsel ifadeden biri diğerinin
yerine kullanılabilir.
x, y  için,
 x  y
2
 x 2  2 xy  y 2 eşitliği doğru olduğundan  x  y  ifadesi yerine
2
x 2  2 xy  y 2 kullanılabilir.
Bazı önemli özdeĢlikler aĢağıda verilmiĢtir.
1  x 2  y 2   x  y  x  y 
2  x 2  2 xy  y 2   x  y    x  y  x  y 
2
3  x 2  2 xy  y 2   x  y    x  y  x  y 
2
4  x3  y 3   x  y   x 2  xy  y 2 
5  x 3  y 3   x  y   x 2  xy  y 2  ,
6   x  y   x 3  3x 2 y  3xy 2  y 3
3
7   x  y   x 3  3x 2 y  3xy 2  y 3
Örnek:
3
63
1  x2  4 
  x  2   x  2
x 2  22
x 2  y 2   x  y  x  y 
2
1
1
1
 1

2  2  64     82    8     8 
x
x
x
 x

x 2  y 2   x  y  x  y 
3  9  a 2  32  a 2   3  a    3  a 
4  x 6  y 6   x3    y 3    x3  y 3    x3  y 3 
2
2
5  x4  y 4   x2    y 2    x2  y 2    x2  y 2 
2
6
x y
2
 x  y  
2
2
2
x y

x y

2
1
1 1 1 1 1 1 1
7  2  2            
a
b a b a b a b
8  16  9a 2  42   3a    4  3a    4  3a 
2
Örnek:
2
1   x  2  x2  2  2 x  4  x2  4x  4
2   3  4a   9  24a  16a 2
2
3   3 x  2    3 x  2    9 x 2  12 x  4    9 x 2  12 x  4 
2
2
 9 x 2  12 x 4 9x 2  12 x 4  24 x
4   2 x 2  3 y 3   4 x 4  12 x 2 y 3  9 y 6
2
Örnek:
1  a3  1 
a 3  13

x3  y 3   x  y  x 2  xy  y 2

  a  1  a 2  a  1
2  8  x 3  23  x 3   2  x   4  2 x  x 2 
3  x3  27 
x3  33

x3  y 3   x  y  x 2  xy  y 2

  x  3  x 2  3 x  9 
4  64  27a 3  43   3a    4  3a  16  12a  9a 2 
3
Örnek:
 x  3  x3  3  x 2  3  3  x  32  33  x3  9 x 2  27 x  27
3
3
2
2
3
2   2 x  1   2 x   3   2 x   1  3   2 x    1   1  8 x3  12 x 2  6 x  1
1
3
3   4  5 x 2   43  3  42  5 x 2   3  4   5 x 2    5 x 2   64  240 x 2  300 x 4  125 x 6
3
2
3
4   2 x  3 y    2 x   3   2 x   3 y   3   2 x  3 y    3 y 
3
3
2
2
3
 8 x3  36 x 2 y  54 xy 2  27 y 3
64
5   3a  4b    3a  4b   ?
3
3
 3a 3  3   3a 2  4b   3   3a  4b 2   4b 3    3a 3  3   3a 2  4b   3   3a  4b 2   4b 3 

 

2
 27a 3  108a 2b 144ab  64b3 27a 3  108a 2b 144ab 2  64b3
 216a 2b  128b3    216a 2b  128b3   128  2a 2b  b3 
ÇARPANLARA AYIRMA
1-GRUPLANDIRMA VE ORTAK ÇARPAN PARANTEZĠNE ALMA:
Bu yöntem çarpmanın toplama ve çıkarma iĢlemi üzerine dağılma özelliğinin bir
uygulamasıdır.
ax  bx  x  a  b    a  b  x
ax  by  ay  bx  ax  bx  ay  by  x  a  b   y  a  b    x  y  a  b 
Örnek:
1  4 x3  5 x 2  x 2  4 x  5 
2  2a 3b  ab 2  ab  2a 2  b 
3  a 2b  b  b  a 2  1  b  a  1 a  1
4  xy  3 y  2 x  6  y  x  3  2  x  3   x  3 y  2 
5  x3  x 2  x  1  x 2  x  1   x  1   x 2  1  x  1   x  1 x  1 x  1   x  1
2
6  6 xy  4 x  3 y  2  2 x  3 y  2    3 y  2    2 x  1 3 y  2 
2- ÜÇ TERĠMLĠLERĠ ÇARPANLARA AYIRMA
Üç terimliler, ax2  bx  c Ģeklindeki ifadelerdir.
I. a  1 ise m  n  c ve m  n  b olmak üzere ax 2  bx  c   x  m  x  n  dir.
Örnek:
x 2  7 x  12 burada b  7 ve c  12 dir.
m
n
m
n
c  12  3  4 ve b  3  4 olarak yazılacağından,
x 2  7 x  12   x  4  x  3 dir.
65
Örnek:
x 2  8 x  15 burada b  8 ve c  15 dir.
m
n
n
m
c  15  3  5 ve b  3   5  olarak yazılacağından,
x 2  8 x  15   x  3 x  5  dir.
Örnek:
x 2  5 x  36 burada b  5 ve c  36 dir.
m
n
m
n
c  36  9   4  ve b  9   4  olarak yazılacağından,
x 2  5 x  36   x  9  x  4  dir.
Örnek:
x 2  x  20 burada b  1 ve c  20 dir.
m
n
m
n
c  20  5   4  ve b  5   4  olarak yazılacağından,
x 2  x  20   x  5  x  4  dir.
II. a  1 olsun.
ax 2  bx  c ifadesinde kx  px  ax 2 , m  n  c ve m  kx  n  px  bx olmak üzere,
ax 2  bx  c   kx  m  px  m  biçiminde yazılır.
Yani,
ax 2  bx  c   kx  n  px  m 
kx
px
n m  kx  n  px  bx

m
Örnek:
6 x2  23x  21 , ax2  6 x2 , bx  23x , c  21 dir.
66
6 x 2  23x  21   2 x  3 3x  7 
3  7   2 x   3  3x  23x
2x
7
3x
Örnek:
3x2  5x  2 , ax2  3x2 , bx  5x , c  2 dir.
3x 2  5 x  2   3x  1 x  2 
3x
1
x
2
 2  3x  1  x  5x
Not.
ax 2  bx  c ifadesinde,
1 a  b  c  0
ise ax 2  bx  c   x  1 ax  c 
2  a  c  b ise ax 2  bx  c   x  1 ax  c  dir.
Örnek:
1  2 x 2  5 x  3   x  1 2 x  3
2  3x 2  2 x  1   x  1 3x  1
3- TAM KAREYE TAMAMLAMA
67
ax 2 bx şeklindeki ifadelere terim ekleyip çıkarma yöntemiyle tam kareye
tamamlanabilir.
2
b 
b2

2
olduğundan ax 2 bx Ģeklindeki ifadelere
 ax
  ax bx 
2a 
4a

2
b
terimi ekleyip çıkararak tam kareye tamamlanabilir.
4a
ax
2
bx  ax
2
b2 b2 
bx 

  ax
4a 4a 
2
b  b2
dir.
 
2a  4a
Örnek:
x 2  10 x ifadesini tam kareye tamamlayınız.
b 2 102 100
a  1 , b  10 ,


 25
4a 4  1
4
x 2  10 x  25  25   x  5   25
2
Örnek:
9 x 2  12 x  1 ifadesini tam kareye tamamlayınız.
a  9 , b  12 ,
b 2 122 144


4
4a 4  9 36
9 x 2  12 x  4  4  1   3x  2   3
2
RASYONEL ĠFADELERĠN SADELEġTĠRĠLMESĠ
Verilen rasyonel ifadeler çarpanlarına ayrılarak ve gerekli olan kısaltmalar yapılarak
ifade daha sade hale getirilir.
Örnek:
x2  4
x
kesrini en sade hale getiriniz.
x2
Çözüm:
x
 x  2  x  2
x2  4
 x
 x   x  2  x  x  2  2
x2
x2


Örnek:
 x  3
2
  x  3
x
2
kesrini sadeleştiriniz.
68
Çözüm:
 x  3
2
  x  3
2
x

 x 2  6 x  9   x 2  6 x  9 x 2  6 x 9  x 2  6 x 9


x
x
12 x
 12
x
Örnek:
x 2 y  3xy  2 y
kesrini en sade şekilde yazınız.
xy 2  y 2
Çözüm:
2
x 2 y  3xy  2 y y  x  3x  2  y  x  1  x  2  x  2



xy 2  y 2
y 2  x  1
y
y  y  x  1
Örnek:
8a 2  ab
a  2b  0 olduğuna göre
kesrinin eĢiti nedir?
3ab  a 2
Çözüm:
a  2b  0  a  2b dir.
8a 2  ab 8  2b    2b  b 32b 2  2b 2
30b 2



 3 tür.
3ab  a 2 3  2b  b   2b 2 6b 2  4b 2 10b 2
2
Örnek:
1  x 2  4   x  2  x  2 
2  9 x 2  y 2   3x  y  3x  y 
3  x 4  y 4   x 2    y 2    x 2  y 2  x 2  y 2 
2
2
4   x  3   x  3  x 2  6 x  9  x 2  6 x  9  12 x
2
2
5  x 3  8  x 3  23   x  2   x 2  2 x  4 
6  a 3  27b3  a 3   3b    a  3b   a 2  3ab  9b 2 
3
2
x3
x 
x
 x  x
3
7   1     1    1    1
8
2
 2  4 2 
3
8  x10  y10   x5    y 5    x 5  y 5  x 5  y 5 
2
2
Örnek:
69
1   x  3  x 2  6 x  9
2
2   2 x  1  4 x 2  4 x  1
2
3   3 x  2 y   9 x 2  12 xy  4 y 2
2

4x

2
3
3x
9
9
2
 2  x2  2  6
  x 2
x
x
x
x
5   2 x  3  8 x 3  18 x 2  36 x  27
3
6   4 x  1  64 x 3  48 x 2  12 x  1
3
7   2  x   8  12 x  6 x 2  x 3
3
3
1
1
1
1
3 1

8   x    x3  3x 2  3x 2  3  x3  3x   3
x
x
x
x
x x

Örnek:
AĢağıdaki ifadeleri çarpanlara ayırınız.
1  x3  x 2  x  1  x 2  x  1   x  1   x 2  1  x  1
2  x3  xy  y 2  x 2 y  x  x 2  y   y  y  x 2    x  y   x 2  y 
3  3x 2  2 y 2  6 x  y 2 x  3x 2  6 x  2 y 2  y 2 x  3x  x  2   y 2  2  x    3x  y 
4  x 2  8 x  12   x  2  x  6 
5
x 2  3x  10   x  5  x  2 
6  6 x 2  5 x  1   3 x  1 2 x  1
Örnek:
AĢağıdaki ifadeleri sadeleĢtiriniz.
x2  2 x  8  x  4  x  2 x  4
1


x2  4
 x  2  x  2 x  2
2
 x  1  x 2  x  1 x  1
x3  1


x3  x 2  x
x
x  x 2  x  1
2
 x  1  x  1
1   x3  1   x  x  1

3  x 
:


1

 
x 1   x2 1 

 x  1  x  1  x 2  x  1
4
2
x3 y  x 2 y 2 x y  x  y  x  x  y  x  y  x



x 2 y 2  xy 3 xy 2  x  y  x  y  y  x  y  y
7-BÖLÜM
ÜSLÜ VE KÖKLÜ SAYILAR
Tanım:
a
ve n 
olmak üzere a  a  a...  a  a n yazılıĢında a n sayısına üslü sayı
n-tane
denir. Burada a ' ya taban, n ' ye üs denir.
an
üs
taban
70
Örnek:
 2 
32  3  3  9
3
  2    2    2   8
103  10  10  10  1000
1
 1   1   1   1   1  1 1 1 1

        
 3   3   3   3   3  3  3  3  3 81
4
 
5
3
 5 5 5 5 5
Üslü Sayıların Özellikleri
0
1. Sıfırdan farklı her sayının 0. kuvveti 1 dir. 20  1 ,  5  1
2. Her sayının 1. kuvveti kendisine eĢittir. Yani,
1
1
1 1
4  4 ,      ,  3   3
5 5
3. p  a n  q  a n  r  a n   p  q  r   a n dir. ( Toplama-Çıkarma iĢlemi)
Örnek:
1
1  7  62  4  62  2  62   7  4  2   6 2  9  6 2
2  5  63  4  63  8  63   5  4  8   62  62
3  3   3  4   3  2   3   3  4  2    3  5   3
5
5
5
5
5
an
 a n  m ( Çarpma – Bölme iĢlemi )
am
4. a n  a m  a n  m ,
Örnek:
2   4    4    4    4    4 
1  33  35  33 5  38
4
n
,
5.
a
n
1
 n
a
2
1
4  3  2 1
  4 
10
 3  3 4  2  3 2  3  3  9
4
 
    
2
 3
4
56
3  4  56  4  52  25
5
a n  b n   ab 
3
an  a 
 
bn  b 
1
a 
a
1
n
n
a b
   
b a
n
Örnek:
2
2
2
1   5    4    5  4   202  20  20  400
4
4
34  3   1 
1
2 4     
6  6   2  16
1
1
2
3   7  

2
 7  49
71
4  10  
1
1
1

 0,1
1
10 10
2
2 3
5   
3 2
2
6. Bir sayının kuvvetinin kuvveti alındığında üsler çarpılır.Yani,
a 
n m
 a nm  a mn   a m  dir.
n
Örnekler:
 2  3 210 310
610
1  8 9  8 9  8  9  2108  3109  22  31  4  3  12
2 3
2 3
2 3
10
2  225  224  224  2  224  224  2 1  224 1  224
3

478  479 2101  299 4  477  42  477 298  23  2  298



477
298
477
298
477   4  42 
477
298  23  2 

298
  4  16    8  2    12   6  6
 
4  3x 1  a ve 2 x 1  b olduğuna göre 0, 4
x
ifadesini a, b cinsinden yazınız.
3x  31  a ve 2 x  2  b
3x
b
 a  3x  3a ve 2 x  dir.
3
2
2
 0, 4 
x
x
x
 4   2  
      
 9   3  
2
 b  
x 2
 2   2  
b2
  x      
olur.
36a 2
 3   3a 


Örnekler:
1.
314  316
?
315
14
2
314  316 314  31432 3  1  3  1  9 8




315
31431
3
3
314  3
 64   2 : 24  ?
2.
 64   2 : 24   26  
2
  26   25  26  1  25  25  2  1  1
4
2
 25   3 
72
2
3.
3
15 x  2 y    ?
 y
2
3
y
y 2 30 xy 3 10 xy 3

15 x  2 y    15 x  2 y    15 x  2 y 
9
9
3
3
 y
 9x 
 3y 
3 
x y
 9x 
4.  3 y 
3 
x y
2
 27 y 
  2x 
3 
x y
 27 y 
  2x 
3 
x y
  32 x 3 y  33 y  2 x 
33 x  2  125 
 75
5.
 75
 32 x 
  3y 
3 

x y
 3
x y
 33 y 
  2x 
3 
2 x 3 y  3 y  2 x

x y
x y
  32 x 3 y 
  30 
x y
x y
  33 y 2 x 
x y
 1x  y  1
2 x 1
?
3 x 1
33 x  2  125 
?
2 x 1
3 x 1

33 x  2   53 
2 x 1

3  5 
2 3 x 1
33 x  2  56 x 3 33 x  2 56 x 3
 3 x 1  6 x  2  33 x  23 x 1  5 6 x 3 6 x  2
3 x 1
6 x2
3 5
3
5
 31  51  15
 3 6 
 8
8 
 6  12 
8
8
1
4
?
1
4
 38  68  4  68  128 
1
1
6.  8
 8


4
8 
8 
8
  2  3   22  38 
 68  128 
 6  12 
 3 6 


 8
8 
 3 6 
 38   2  38 


1
1
1
1
 8 8

 8 
16
8
256
 2  3  2  3   28  38 1  28   2
 8
 

8
8
 3  2 3  
8
8
 3 (1  2 ) 


20,4  51,5  40,3  250,75  ?
7. 2
0,4
5 4
1,5
40
150
0,3
 25
0,75
60
2
40
100
150
150
100
5
40
 2100  5100   2 100   5 100  2100
4

30
100
60
100
 5
75
2 100

150 150

100
 5100
2
40
100
100
150
100
5
 2
30
2 100
75
2 100
  5 
300
 2100  5100  21  53  2  125  250
73
1  2a  : 1  3x 
8
5
5
1  

1 
1 
 1  
 2a   3 x 
?
 3x  1 
7
8
5
8 

1  2a   1  2a   2a  3x  1
1

2
a
1

3
x
1

2
a






1
3x 

8.
:





5
5
1  
 3x

 1  2a  1  3 x 
1 
1  2a 
 3x  1

1 
 1  


 2a   3 x 
 2a 
5
 1 
 1  2a   2a   
 3x 
5
7
0,1  0, 2 
2  0,5 



?
0,01 0,02 0, 2 0,05
2
2
2
2
 2
 5
1


0,1  0, 2 
2  0,5 
2  10 
10



 10    

9. 
1
2
2
5
0,01 0,02 0, 2 0,05
100
100
10
100
1 100
4 100 2 10 25 100
 


  

 10  2  10  5  17
10 1 100 2
1 2 100 5
2
2
a 1  b 1
?
a 2  b 2
10. 

1
1
a b
a 2  b 2
 a  b
1 1

 a  b   a 2b2   a  b   ab  ab
 a b  2 ab 2 
1
1
ab
ab
b a 
b2  a 2 
b  a  b  a 



2
2
2 2
a
b
ab
ab
b  a 
4 x  6 olduğuna göre  0, 25
1 x
11. =  0, 25 
1 x
 0,5
 4 x 1  22 x 1 
2 x 1
 0,5
1 x
 25 


 100 
2 x 1
 5
 
 10 
ifadesinin değerini bulunuz.
2 x 1
1 x
1
 
4
1
 
2
2 x 1
  41 
1 x
  21 
4 x 22 x 4 x 22 x 6 6 6 12

 
   
 18
4 21 4 1
4 1 4 1
2
2
74
2 x 1




5
a  2 x 1 ve b  3x  2 olduğuna göre 6 x 1 ifadesinin değerini
12. a, b cinsinden hesaplayınız.
=6 x 1   2  3
x 1
 2 x 1  3x 1  2 x 1 
3x  2
b ab
 a 
3
3 3
22000  21999  21998  21997  k  21997 eşitliğini sağlayan k sayısını bulunuz.
13.
=21997  23  21997  22  21997  21  21997  k  21997
21997  (23  22  21  1)  k  21997
k  8  4  2 1  3
3x  125 , 3 y  5 ise
x y
ifadesinin değerini bulunuz.
x y
3x  53 , 3 y  5  3x  5  5  5
 3x  3 y  3 y  3 y
14.
 3x  33 y
 x  3 y bulunur.
x  y 3y  y 4 y


2
x  y 3y  y 2 y
32 x  34 x 5  3  x  ?
32 x  34 x 5  3
15.
16.
32 34 x

3
3x 35
34 x 32
 5  3  34 x  x  33  31  33 x 3  31
x
3 3
4
 3x  4  x  
3
24 x 3  32  x  ?
24 x 3  32  24 x 3  25  4 x  3  5  4 x  8  x  2
75
1
 
2
2 x 1
1
17.  
2
2 x 1
 0,125  x  ?
5
 0,125   
 10 
5
 
 10 
2 x 1
2 x 1

125
1000
3
 5
    2x 1  3  x  2
 10 
1
1

 2x  x  ?
x
x
1 3
1 3
1
1
1
1

 2x 

 2x
x
x
x
1
1 3
1 3
1 x 1 3
3
18.
x
1
1
3
1

 2x 

 2x
x
x
x
x
1 3 1 3
1 3 1 3
x
3
1  3x
 2 x  1  2 x  x  0 dır.
1  3x
1
 25
19.
x
 0, 00032  x  ?
x
 0, 00032 
1
 25
1
5 
2 x

32
100000
5
5
 10 
2x
5
2x
    5   5  5  5  2 x  x  olacaktır.
2
 2
15x  6 x 16
  x?
20 x  8 x 9




9   3  5    2  3
15x  6 x 16
15 x  6 x 16




1
x
x
20 x  8x 9
20 x  8 x
9
16

4

5

2

4




20.

4    4  5
2
x
x
  1  3  3  5    2  3   3  3 5  2   1
4   4  5    2  4  4  4 5  2 
  2  4 
32   3  5    2  3
x
x
x
2
x
x
x
x
2
x
x
x
x
2
x
x
x
x
2
x
x
x
32  x  4 x  2  x  2 dir.
76
KÖKLÜ ĠFADELER
Tanım:
a ve b negatif olmayan iki reel sayı olmak üzere , a 2  b ise a sayısına b
sayısının karekökü denir. Ve a  b Ģeklinde gösterilir.
42 ,
9 3 ,
0 0 ,
0,25 
25
25
5


 0,5
100
100 10
Bilindiği gibi,
2
a 2  b ve  a   b olarak yazarız. Dolayısıyla, a   b ve a  b dir.
Bu nedenle, pozitif a, b sayıları için a 2  b ise b ve  b sayılarına sırasıyla b
sayısının pozitif ve negatif kökleri denir. Bir b sayısının karekökü denilince pozitif
karekökünün anlaĢılması gerekir. Örneğin,
4  2 , 4  2 dir.
 5
2
 5    5  ve
52  5 dir
Genel olarak,
a  0 için a 2  a ve a  0 için a 2  a olur.
a , a  0
a2  
a , a  0
a
ve | a | 
 a
, a0
,
a0
olduğundan
a 2 | a | dir.
Örnek:
AĢağıdaki ifadeleri hesaplayınız.
a  32 
 4
2

 4 
2
3
 6
2
?
b
 x  1
2
 |1  x | 2
Çözüm:
a  32 
b
 x  1
2
 6 
2
| 3 |  | 4 | 3 | 6 | 3  4  3  6  11
 |1  x | 2 | x  1|  |1  x | 2  2
Örnek:
x  2 için
Çözüm:
x 2  3x  2  x 2  4 x  4 ifadesini hesaplayınız.
77
x  2 için
x 2  3x  2  x 2  4 x  4 ,
x2  4 x  4 
 x  2
2
| x  2 |   x  2    x  2
 x  2
x 2  3x  2  x  2 = x 2  4 x  4 
2
| x  2 |  x  2 bulunur.
Örnek:
 2 x  1
2

2  x
2
2

2  x
2
denkleminin çözüm kümesini bulunuz.
Çözüm:
 2 x  1
| 2 x  1|| 2  x | dir.Bu durumda,
2 x  1  2  x veya 2 x  1    2  x  olmalıdır.
x  1 ve x  1 bulunur. O halde, Ç  {1,1} dir.
Karekökün Özellikleri
1.
a 2 | a |  karekök işleminin sonucu daima pozitiftir.
2. Negatif sayıların karekökü yoktur. Yani, 16  4
3.
a b  a  b
4.
a b a
,
a
a

b
b
,
16 
, 16 
dir
a, b  0
b dir.
Örnek:
1 1
 ?
16 9
Çözüm:
1
1
9  16
25
25
5





dir.
16
9
16  9
144
144 12
(9) (16)
Örnek:
2,25  0,25
1,44  1,21
2,25  0,25
1,44  1,21
Örnek:
48  27
12  3
Çözüm:
48  27
12  3

 ? sonucu en sade Ģekilde yazınız.
225
25

100
100 
144
121

100
100
225
25 15 5
20


100
100  10 10  10  20  20
1
1
144
121 12  11

10
10
10
100
100
 ? sonucu en sade Ģekilde yazınız.

16  3  9  3
43  3

4 33 3
2 3 3

7 3
7
3
78
Paydayı Rasyonel Yapma ( EġLENĠK ĠLE ÇARPMA )
Bir köklü sayıyı rasyonel yapan çarpana, bu irrasyonel sayının eĢleniği denir.
1.
a sayısının eĢleniği a dır. Yani, kendisidir.
a  a  a dır.


2.

b
eşleniği
a  b 
a


a  b dir.

a  b  a  b dir.
Örnek:
4
sayısını rasyonel biçimde yazınız.
3
Çözüm:
4 3
4 3
4 3


2
3
3 3
3
 
Örnek:
0,9  4,9  ?
Çözüm:
9
49
9
49
3
7






10
10
10
10
10
10
0,9  4,9 
37
10
10

10


10 10
10  10


10 10
 10
10
Örnek:
4
ifadesini rasyonel biçimde yazınız.
3 1
Çözüm:
4
3 1

Örnek:
5 3
3 5
Çözüm:
4




3 1

3 1 
3 5
3 5

3 1

4

3 1


15  5  3  15
35
2
3 1
2


3 1
?


5 3
3  5 ( 5  3) 


3 5
3 5
3 5 

  4
3 1
  3 
15 
35
  ( 3  5)   3  5  
3  5  ( 3  5)   3  5 
15  5   2 15  8  2 15  8  16


8
3 5
2
2
Örnek:
79
1
3
 3 2 ifadesini hesaplayınız.
2 3 1 2
Çözüm:
1
3
1 3
3(1  2)
3 33 2

3 2 

3 2 

3 2
6
1 2
2 3 1 2
2 3  3 (1  2)  (1  2)


3
3
3  18
3 3 2  3 2 
3
6
6
6
Örnek:
6
32  18 
2
?
Çözüm:
6
32  18 
2
 4 2 3 2 
6 2
4 2 3 2  3 2 4 2
2
Örnek:
4
ifadesini hesaplayınız.
5
Çözüm:
4
4 5
4 5


5
5
5 5
Örnek:
3
?
53
Çözüm:
3
3
3
3
3 5
15
3 5





veya
dir.
3
3
5
25
25
5  ( 5  5) 5 5 5 5 5
5
Örnek:
1
?
2 3 5
Çözüm:
 2  3  5
1  2  3  5 
1





2  3  5  2  3  5   2  3  5
52 6 5

 

 2  3  5  2  3  5  6  2  3  5  6





dir.
12
2 6
2 6 6
Örnek:
40
40
40 10 2  10 10  40 10 60 10
40 
2 10 
2 10 
10 
10 
10 
10
 10
2
2
2
0, 4
4
10
10
10
10














60 10 10 60  10


 30
10
2
20
Örnek:

80
74 3
ifadesini sade şekilde yazınız.
6  2 1
Çözüm:




(7  4 3)    6  2  1
(7  4 3)    6  2  1
74 3

 

 



=
 6  2  1   6  2  1
6  2 1
8  2 12  1

 



3)   

(7  4

6
 
2   1
 

(7  4 3)




6  2  1 dir.

Ġç Ġçe Kareköklü Sayılar
a 2 b şeklindeki köklü sayılar,
1. m  n  b ve m  n  a olsun.
m  n olmak üzere a 2 b  m
n dir.
b şeklindeki köklü sayılar,
a
2. c  a  b olmak üzere,
2
a
2
b
ac
2
ac
dir.
2
Örnek:
1 5  2 6  ?
5 2 6
a  5  3  2 , b  6  3  2 dir
n
m
n
m
O halde, 5  2 6  3  2 dir.
2  9  2 14  ?
9  2 14
a  9  7  2 , b  6  7  2 dir
m
n
m
n
O halde, 9  2 14  7  2 dir.
81
3  16  4 7  ?
16  4 7  16  2 28
a  16  14  2 , b  28  14  2 dir
m
n
m
n
O halde, 16  2 28  14  2 dir.
4  8  60  ?
8  60  8  2 15
a  8  5  3 , b  6  5  3 dir
m
m n
n
O halde, 8  2 15  5  3 dir.
5  3  5 sayısının eĢitini bulunuz.
3  5 , a  3 , b  5 , c2  a2  b
olduğundan
c  3  5  9  5  4  c  4  c  2 dir. O halde,
2
2
3 5 
2
3 2
3 2
5
1
5 1






2
2
2
2
2
2


5 1  2
2 2

10  2
2

14  2
2

14  2
ve
2
6  4  7  4  7 sayısının eĢitini bulunuz.
4  7 , a  4 , b  7 , c2  a2  b
olduğundan
c  4  7  16  7  4  c  9  c  3 dir. O halde,
2
2
2
4 7 
43
43
7
1
7 1






2
2
2
2
2
2
4 7 
43
43
7
1
7 1






2
2
2
2
2
2
4 7  4 7 



7 1  2
2 2

7 1  2
2 2
14  2
14  2  2
2 2 2




  2 dir.
2
2
2
2
2
Reel Sayıların rasyonel Kuvvetleri
Tanım:
a, b negatif olmayan reel sayılar, n pozitif tamsayı olmak üzere b n  a ise
b sayısına a 'nın n. kuvvetten kökü denir ve n a ile gösterilir. Buna göre,
b  n a  b n  a dır.
n
a ifadesinde n sayısına kök kuvveti denir. n a ifadesine ise ''n. kuvvetten kök a ''
diye okunur.
82
1
a yerine a n gösterimide kullanılır. BaĢka bir gösterimle,
n
1
n
n
a  a dir.
3
8  83  2 dir. Çünkü, 23  8 dir.
5
32  2 , 4 256  4 olduğu rahatça görülebilir.
1
Uyarılar:
1. a negatif bir reel sayı ve n bir tek sayma sayısı ise n a bir negatif reel
sayıdır.
2. a negatif olmayan bir reel sayı ve n bir tek sayma sayısı ise n a bir
negatif olmayan reel sayıdır.
3. a negatif bir reel sayı ve n bir çift sayma sayısı ise n a bir reel sayı
değildir.
Çünkü,
4
16  dir. x  4 16 olsun. x 4  16 olacak şekilde hiç bir reel sayı bulunamaz.
Yani, x 
dir.
n
a ifadesinde,
n  2 ise 2 a  a dır. ( Karekök a Ģeklinde okunur.)
n  3 ise 3 a dır. ( Küpkök a Ģeklinde okunur.)
Köklü Sayıların Bazı Özellikleri
a  0 ve n   olsun.
1.
 a
n
n
 n a n  a dır.
a  0 ise,
2. n tek sayma sayısı ise n a n  a
n çift sayma sayısı ise n a n | a | dır.
Örnek:
1.
8
 2 
 4 16  3 5 32  3  5   6  2  ifadesini hesaplayınız.
8
 2 
 4 16  3 5 32  3  5   6  2  =|-2|+2+3   2   5 | 2 |
8
8
3
3
6
6
 2  2  6  5  2  5
4
2.
4
 
1  2 
1 2
4
 7 x7  6 x6  ?
4
 7 x 7  6 x 6  1  2  x  | x | 2  1  x | x | dir.
 2  1  x  x,
2  1  x  | x | 
 2  1  x  x,
2 1
, x0
2  1  2x
, x0
olur.
Köklü bir ifadenin Kuvvetini alma
83
1
a  0 , m ile n pozitif tamsayılar olsun. n a  a n sayısının her iki tarafının m. kuvveti
alınırsa,
 a
n
m
m
m
 1
  a n   n a m  a n olarak yazılır.
 
m
n
a  0 olduğunda a  a sayısının bir reel sayı olması için, m  çift ve n  tek doğal
n
m
sayı olmalıdır.
Örnek:
AĢağıdaki ifadeleri hesaplayınız.
5
3
1. 8 
 8
5
3

   2  32
3
5
23
 32  5   5  2 
3
2.
3
5

3
4
3. 16 
3
24
3

4
   3  81
   2     2   8
 27  3  3  3
5.
 128 7
3
6. 9
3

   2   8

  2 8
4
4.
3
2
5
1

9
3
2
1
 3
2
3
4
3
7
7

4
3

3
1
1

3
3
27
Negatif olmayan bir a sayısı ve m, n, p 
m
n
am  a n  a
m p
n p

np

olmak üzere,
a mp dir.
Bu ise bize kök kuvvetinin ve üssün bir p 

sayısıyla geniĢletilip veya
sadeleştirileceğini gösterir.
Örnek:
a  2 , b  3 3 , c  6 6 sayılarını küçükten büyüğe doğru sıralayınız.
Çözüm:
a  2  3 2 2 3  6 8 


2 3 2
3
6
b  3  3  9  6 6  6 8  6 9  c  a  b dir.
a
b
 c
c  6 6=16 61 = 6 6 

Köklü Bir Terimin Kökü
84
1
a  0 olsun. n a  a n eşitliğinin her iki tarafının m.kuvvetten kökünü alalım.
1
1
 1n  m
m n
nm
a   a   a  nm a  nm a dır.
 
a  0 olduğunda m ve n sayıları tek doğal sayı olmalıdır.
Örnek:
2 2
3
1.
8
3
3 2
 1
 1
2.  64 3   6  64 3  




 0,0016 
3.
3
6
3
6
6 3
1
64  18 64  18 26  218  2 3  3 2
 8  0,0016  
2
1
2
8 2 2 2  2
1
6
6
2
4
 0,0016   4
4
16
2

 0,2
10000 10
Payı 1 Olan Köklü Ġfadeleri Üslü YazılıĢı
1
n
a
1

a
a
1
n
1
n
,
1
n
am

1
a
m
n
a
m
n
dir.
Örnek:
AĢağıdaki ifadeleri üslü biçimde yazınız.
1
1
1
1. 4  1  5 4
5
54
3
1
1
1
2
2.



2
1
3
8
3 2
 2  22
3.
1
3
4

1
1
2 3
2 

1
2
2
23
23
Köklü Ġfadelerde ĠĢlemler
1. Toplama-çıkarma iĢlemi:
Hem kök kuvveti ve içi aynı olan ifadelere benzer ifadeler denir. Örneğin,
1
5 3 2 ,  7 3 2 , 3 2 benzer köklü ifadelerdir. Bunlar toplanır veya
5
çıkarılırken katsayılar toplanır veya çıkarılır. Buna göre,
p  n a m  q  n a m  r  n a m   p  q  r   n a m dir.
Örnek:
5 3 a  4 3 a  12 3 b  2 3 a  8 3 b  5 3 a  4 3 a  2 3 a  12 3 b  8 3 b

 

  5  4  2  3 a   12  8 3 b  7 3 a  4 3 b dir.
Kök Kuvvetleri aynı olan Köklü Ġfadelerin Çarpımı
85
Kök kuvvetleri aynı olan iki köklü ifade çarpılırken, ortak kök altına kök
içlerinin çarpımı
yazılır. Buna göre,
n
a m  n bk  n a m  bk olur.
Örnek:
AĢağıdaki ifadeleri hesaplayınız.
1.
4
52  4 5  4 53  4 52  5  53  4 56 
2.
5
38  5 33  5 383  5 311
2 2
5 2 3  53  5 5
Kök Kuvvetleri Aynı Olan köklü ifadelerin Bölümü
Bir köklü ifade, kök kuvvetleri aynı olan bir köklü ifadeye bölünürken, ortak
kök altında kök içleri bölünür. Yani,
n
a na

,
n
b
b
yazılabilir.
a
  
n m
b
b
n
am
n
m
n
am
am
,

olarak
n k
bk
b
n
Örnek:
AĢağıdaki iĢlemleri yapınız.
3
128 3 128 3
1. 3

 64  3 43  4
2
2
8a 2b5  3ab 4
2.
6a3b 2
8a 2b5 3ab 4

6a3b 2

8a 2b5 3ab 4
24 a 3 b9

 4b7  2b3 b
3 2
3 2
6a b
6a b
Köklü Denklemler
Köklü denklem çözümünde aĢağıda verilen yol takip edilir.
m
1. n a m  a n özelliği kullanılarak denklem üslü biçimde yazılır.
2. Denklemde her iki tarafın uygun kuvveti alınarak kökler kaldırılır.
Örnek:
3
2 x  1  5 denkleminde x  ?
Çözüm:
3
2x  1  5 

3

2 x  1  53   2 x  1  125
3
 2 x  125  1  2 x  126  x  63
olarak bulunur.
Örnek:
2x
3
2x
 2 denklemini çözünüz.
Çözüm:
86
6
 23 x 
23 x
6
2
 2   6 2 x   26  23 x  2 x  26  2 x  26  x  6
2x
x
 2 
2
2


2x
3
Örnek:
92 x  y 1
3
27 x  y  2

1
 x?
3
Çözüm:
6
6
1  6 323 2 x  y 1   1 
312 x  6 y  6
   32 x  y  2      6 x  6 y 12  36  312 x  6 y  6  6 x  6 y 12  36
3
 3
3
27 x  y  2 3  3

36 x 18  36  6 x  18  6  6 x  24  x  4 
92 x  y 1
Örnek:
2  x  2 , x 
x 2  4 x  4  x 2  4 x  4 ifadesini
olmak üzere
en sade biçimde yazınız.
Çözüm:
x2  4x  4  x2  4 x  4 
 x  2
2
 x  2

2
 x  2  x  2 dir.
2  x  0  x  2 olup, x  2  x  2 dir.
x  2  x  2  0 olup,
x  2   x  2 bu değerler yerlerine yazılırsa,
x2  x2  x2 x24
Örnek:
x2  6 x  9  2 eşitsizliğinin
de çözüm kümesini bulunuz.
Çözüm:
x2  6 x  9  2 
1  x  5 tir. Ç   x 
 x  3
2
 2  x  3  2  2  x  3  2
/ 1  x  5  1,5 tir.
Örnek:
4 x2  4 x  1  2 x  1  çözüm kümesini bulunuz.
Çözüm:
4 x2  4 x  1  2 x  1 
 2 x  1
2
 2x  1  2x  1  2x  1
1
olmalıdır.
2
1  1

/ x       ,   dur.
2  2

2 x  1  0 olmalıdır. x  

Ç  x 

Örnek:
4 x2  8x  4  x 2  2 x  1  6 denkleminin çözüm kümesini bulunuz.
Örnek:
87
 x  c
2
 y2 
 x  c
 y 2  2a  ve a 2  b 2  c 2
2
x2 y 2
olduğuna göre 2  2  1 olduğuna bakınız.
a
b
Örnek:
3 3 9 4 27  3x  x  ?
Çözüm:
3
 3
 2 
4
3 3 9 4 27  3x  3 3  32  4  33   3 3  32   3 4  3 3
1
 3
11
12
23
23
23
23
3
 312  3122  3 24 bulunur. 3 24  3x  x 
3
11
11
 3 3 4  3  312
23
tür.
24
Örnek:
3
x, y 

 y x 
 ifadesini en sade şekilde yazınız.
için 4 
 3 x2 y 


Çözüm:
3
3
 y x 
4 
 
 3 x2 y 


4
y3 x 2

x2 y
4
2
y x
3
2
2

4
2
y x
1
2
2
4
 y x
1
8

y. 8
1

x
y
8
x
Örnek:
3 x 1
9  2 x  4 27  x  ?
Çözüm:
3 x 1
9
2x4
27  3
2
3 x 1
3
3
2x4

2
3

 9 x  3  4 x  8  5x  5  x  1
3x  1 2 x  4
Örnekler:
I.AĢağıdaki ifadeleri en sade biçimde yazınız.
1
1  3 16  3
?
128
1
2  3 3 3  2 3 24  3 375  ?
5
3
2
2
3
 3 ?
2
2
4
3
3 6
:
?
3 36
88
II. AĢağıdaki ifadelerin paydalarını rasyonel yapınız.
1
1 3
2 1
1
2 3
3
25  10  3 4
1
3
4
17  12 2
1
4
4
29  20 6
5
2
4
23
III. AĢağıdaki ifadeleri en sade biçimde yazınız.
1 3
16 x 2 y 5
2 x 4 y 4
2 3
12 xy 2
x 2 y 1
3

3 x 1 y 3 16 x 3 y
3 3
a 4c
a 2c 3
a 4c x 2


b3
bx 3
bx
4 3
13 1
a4
2
a a
IV. AĢağıdaki denklemleri çözünüz.
1  3 3x 1  9 x  2  x  ?
2  3 2 84
1
 16 x  x  ?
4
3  4 x 8 x  16 x 1 3 2 x  2  x  ?
4  x 1 5  25  x  ?
V.
x  2 ve y  2 için
x
x x  y y y  6 eşitliğini sağlayan x, y doğal sayılarını bulunuz.
89
8-BÖLÜM
DENKLEM ÇÖZÜMLERĠ
Bir Bilinmeyenli Denklemler
Tanım:
a  0 olmak üzere a, b  olsun.
ax  b  0 şeklinde olan ifadeye 1. dereceden bir bilinmeyenli denklem denir.
ax  b  0 denkleminde bilinmeyen x 'i çekme işlemine denklemi çözmek
b
denir. Ve , x   dır. Denklemin çözüm kümesi Ç olsun. O halde,
a
b

Ç   x    olarak yazarız.
a

Verilen denklemde,
1  a  0 ise çözüm kümesi vardır. Bu küme tek elemanlıdır.
2  a  0 ve b  0 ise çözüm kümesi boş kümedir.
3  a  0 ve b  0 is çözüm kümesi sonsuzdur.
Örnek:
3x  6 
Çözüm:
1
 x  8  x  ?
2
1
 x  8  2  3x  6    x  8  6 x  12  x  8
2
6 x  x  8  12  5 x  20  x  4  Ç  4
3x  6 
Örnek:
3 5x  7   x  4  x  ?
Çözüm:
3  5 x  7   x  4  15 x  21  x  4  21  4  x  15 x
 17  16 x  x  
Örnek:
Çözüm:
16
 16 
 Ç   
17
 17 
x2 x5

 1 denklemini çözünüz.
3
4
x2 x5

 1 denkleminde paydaları eĢitleyelim.
3
4
 4   3
4 x  8 3x  15
4 x  8  3 x  15

1
 1  x  23  12  x  12  23  35'tir.
12
12
12
90
Örnek:
3x  2

3x  1
Çözüm:
3x  2

3x  1
x 1
 x?
x 5
x 1
  3 x  2  x  5    3 x  1 x  1
x5
3x 2  15 x  2 x  10  3x 2  3x  x  1
 13x  10  2 x  1
 10  1  2 x  13x
 9  15 x
x
Örnek:
Çözüm:
9
3
 
15
5
2n  5 n  2 4 n  1


n?
4
3
6
2n  5 n  2 4n  1 6n  15 4n  8 4n  1





 6n  15  4n  8  4n  1
4
3
6
12
12
12
 10n  7  4n  1
 3
 4
 2
 10n  4n  1  7  6n  6
 n  1'dir
Örnek:
2
3

 3 denkleminin köklerinden biri 2 ise a  ?
x 1 x  a
Çözüm:
Örnek:
2
3

 3 denklemin köklerinden biri 2 ise,
x 1 x  a
bu durumda x yerine 2 yazmalıyız.
2
3
2
3
3

3  
3 2
3
2 1 2  a
1 2a
2a
3

 3  2  3   2  a   1  3  2  a  a  1 dir.
2a
Hangi sayının 3 katının 6 eksiğinin yarısı 39 dur?
Çözüm:
91
Aranan sayı x olsun.
 3x  6   39
2
 3x  6 
şeklinde denklem oluşturulur. Ve çözüme geçilir,
 39   3x  6   2  39  3 x  6  78  3 x  78  6
2
 3x  84  x  28 dir.
Örnek:
Ali’nin 4 yıl önceki yaĢı, 6 yıl sonraki yaĢının
yaĢındadır?
1
ü olacağına göre Ali bugün kaç
3
Çözüm:
Ali nin bu günkü yaşı 4 yıl önceki yaĢı 6 yıl sonraki yaĢı olarak alınabilir.
x
 x  4
 x  6
1
3
1
 x  4    x  6   3   x  4    x  6   3x  12  x  6  3x  x  6  12
3
2 x  18  x  9 dur.
 x  4    x  6  denklemi yazılır. Ve denklem çözümüne geçilir.
Örnek:
4
 4 denkleminde x  ?
2x
1
x 1
Çözüm:
4
2x
1
x 1
 4 öncelikle çözüme dikdörtgensel bölge içindeki kısımdan baĢlayacağız.
4  x  1
1
4 
 4 yazılarak içler ve dıĢlar çarpımı yapılır.
x  1  2x
3x  1
x 1
4
4 x  4  12 x  4  4  4  12 x  4 x  0  8 x  x  0 dır.
Örnek:
1
3x 
 0 denklemini çözünüz.
x
1
x2
Çözüm.
1
1
1  1 x  2   x  2  0  x  2  0  x  2
3x 
 3x 



1  2 
2
x
x2 x
2
1
x2
x2
x2
1
92
Örnek:
AĢağıdaki denklemleri çözünüz.
3
1
1

 3  x  2   1  1  2 x   3 x  6  1  2 x 
1  2x x  2
6  1  2 x  3x  5  5 x  x  1
10 x  3 5
  2 10 x  3  5  7 x  1  20 x  6  35 x  5
7x  1 2
11
6  5  35 x  20 x  11  15 x  x  
15
2
x
1
2
3
3
 
2x 3
5
3
x3
3
15  2 x
3

2
önce bölme işlemi yapacağız.Yani,
3
Bölme işlemi yapılacak
x3
3
2

  3  x  3  2 15  2 x   3 x  9  30  4 x
3 15  2 x 3
 9  30  4 x  3 x  39  x dir.
2x
20  2 x
5
5
4
2

2

4
1 4

4
2 4
4
20  2 x
5
6
4
 2
20  2 x 4
 2
5
6
Bölme işlemi önce yapılacak
80  8 x
20 10 5
 2  80  8 x  60  20  8 x  x 

 dir.
30
8
4 2
3
5
kesrinin pay ve paydasında hangi sayı çıkarılırsa kesrin değeri olur?
5
3
Çözüm :
3 x 5
  3  3  x   5  5  x   9  3 x  25  5 x  3 x  5 x  25  9
5 x 3
 2 x  16  x  8 dir.
5
6  6 x  3  x  1  x  1 denkleminin çözüm kümesini bulunuz.
Çözüm:
6 x  3  x  1  x  1  6 x  3x  3  x  1  3x  3  x  1
3x  x  1  3  2 x  4  x  2  Ç  2 dir.
93
7
4x 2x  1 1

 denkleminin çözüm kümesini bulunuz.
15
10
2
Çözüm :
4x 2x  1 1


15
10
2
 2   3 15


 8 x  6 x  3 15

 8 x  6 x  3  15
 
30
30
30


 2 x  15  3  12
 x  6 , Ç  6
8  3ax  5b  3bx  5a denkleminin çözüm kümesini bulunuz.
Çözüm:
3ax  5b  3bx  5a  3ax  3bx  5a  5b  3 x  a  b   5  a  b 
3x  5  x 
5
5 
, Ç 
3
3 
9  2 x3  4 x 2  8 x  16  0 denkleminin çözüm kümesini bulunuz.
Çözüm:
2 x3  4 x 2  8 x  16  0   2 x 3  4 x 2   8 x  16   0
 2 x2  x  2  8 x  2  0
  2 x2  8  x  2   0
 2  x2  4  x  2  0
  x  2  x  2  x  2   0
  x  2  x  2  0
2
 x1  2 , x2  2 dir.
 Ç  2, 2
10  x 2 
ab
ab
x
 0 denkleminin çözüm kümesini bulunuz.
2
4
Çözüm:
ab
ab
x2 
x
 0  4 x 2   2a  2b  x  ab  0
2
4
2
 4 x  2  a  b  x  ab   2 x  a  2 x  b   0
2x  a , 2x  b  x 
a
b
a b 
, x   Ç   ,  dir.
2
2
2 2
ALIġTIRMALAR
I. AĢağıdaki denklemleri çözünüz.
94
1. 7 x  3  17
2.  2 x  5 x  93
3. 13  5 x  1  44  x
8   7  x   33
4.
5. 4 x   3  2 x   17
x 2  15  x  2   3  2  x  5  3 x  x 2
6.
7. 2 x  x  5   4  x  5 
8.
 2 x  3 x  5 
2
 9  x  5
9. x 3  x 2  x  1  0
10. 4 x 3  4 x 2  x  1  0
II. AĢağıdaki denklemleri çözünüz.
1. x 3  6  150  x 27
2.
3.
4.
5.
6.
250  x 3 32  3 4 x 3  3 2
x 1 x 1 5


2
6
3
4x  3 4x  1 1


6
9
2
2 x  15 3 x  2 6 x  53


0
12
9
36
4 | x | 1  2 | x | 19  1
3
7. | 2 x | 4 x  5
8. 6ax  4 x  3a 2  5a  2
9.  2 x  14  2  4a    2 x  18  3  4a 
abx abx m  n


m
n
mn
xa
x
2ab
11.

 2
0
a  b a  b a  b2
x  15 x  6 7a  9
12.


0
a  1 a  1 a2  1
10.
RASYONEL DENKLEMLER
Tanım:
95
P  x  , Q  x  iki polinom olsun. Q  x   0 ve x 
P  x
olmak üzere
Q  x
biçimindeki
ifadeye rasyonel fonksiyon denir.
P  x
Q  x
 0 ifadesi bir denklem ve bu şartı sağlayan x lerin kümesine rasyonel denklemin
kökleri denir.
Rasyonel denklemin çözümünde P  x  ' in kökleri bulunur ve bulunan köklerde Q  x  'i
sıfır yapanlar çıkarılır geriye kalanlar denklemin kökleridir.yani,
P  x
Q  x
 0  P  x   0 ve Q  x   0 olur.
Örnek:
Çözüm:
3
2
x
 
denkleminin çözüm kümesini bulunuz.
x2 3 x2
3
2
x 
 
2  x  2
9
3x

x2
3 x  2


0
3 x  2 3 x  2  3 x  2 

 3  x  2   3 
9  2 x  4  3x
5  5x
0
 0  5  5 x  0 ve 3  x  2   0
3 x  2
3 x  2
denklemlerinin çözümünü yapalım.
5  5x  0  x  1
 x  2   0  x  2 olarak yazılır. Ve, Ç  1 dir.
Örnek:
x
x 1
5
x
x 1
5

 2



x  1 x  4 x  3x  4
x  1 x  4  x  1 x  4 
x  x  4    x  1 x  1  5
 x  1 x  4 
0
6 x  6
0
 x  1 x  4 
6 x  6  0  x  1 ve  x  1 x  4   0 
x  1
x4
olmalıdır.
O halde çözüm boş kümedir.Ç   dir.
Örnek:
2a
2b
b 
 a denkleminin çözüm kümesini bulunuz.
x 1
x 1
Çözüm:
96
2  a  b
2a
2b
2a
2b
b 
a

 ab
  a  b
x 1
x 1
x 1 x 1
x 1
2
 1  x  1  2  x  1 dir. Ç  1
x 1
BĠRĠNCĠ DERECEDEN ĠKĠ BĠLĠNMEYENLĠ DENKLEMLER
Tanım:
a, b, c sabit sayılar olmak üzere, ax  by  c biçimindeki denklemlere
1.dereceden iki bilinmeyenli denklem denir.
Örnek:
3x  2 y  4 ,  x  y  1 , 3t  5s  0 , 6u  3v  2
gibi denklemler 1.dereceden iki bilinmeyenli denklemlerdir.
Örnek:
2 x  3 y  5 denklemini ele alalım. Bunlardan birine x  t  t ye parametre denir  olsun.
O halde, 2 x  3 y  5  2t  3 y  5  2t  5  3 y  y 
2t  5
olacaktır.
3
Ġki denklemden oluĢan,
ax  by  c
ifadesine iki bilinmeyenli denklem sistemi denir.

dx  ey  f
Bunların çözümü için 3 yol vardır.
1. Yerine koyma yöntemi,
2. Yok etme yöntemi,
3. Grafik yöntemi
1- YOK ETME YÖNTEMĠ
Bu yöntemde verilen denklemlerde değiĢkenlerin birinin katsayıları eĢitlenerek,
taraf tarafa çıkarılır veya toplanarak yok edilir. Bulunan diğer değiĢkenin çözümü
yapılır. Bulunan değer herhangi bir denklemde yerine yazılarak çözüm tamamlanır.
Örnek:
3x  4 y  18
denklem siteminin çözüm kümesini bulunuz.

2 x  5 y  11
Çözüm:
y ' yi yok edelim.
15 x 20 y  90
5 / 3 x  4 y  18


  23x  46  x  2

4 / 2 x  5 y  11
8 x 20 y  44
23x  46
3   2   4 y  18  6  4 y  18  6  18  4 y  12  4 y  y  3
Ç   x, y   x  2, y  3
97
2-YERĠNE KOYME YÖNTEMĠ
Verilen denklemlerden birinden, bilinmeyenin biri diğeri cinsinden bulunarak
diğer denklemde yerine yazılır. Denklem çözümüne devam edilir.
Örnek:
3 x  4 y  18
denklem siteminin çözüm kümesini bulunuz.

2 x  5 y  11
Çözüm:
y ' yi çekelim.
 18  3x  18  3x

3 x  4 y  18 
 3 x  4 y  18  4 y  18  3 x  y 

bulunur.

4
4
2 x  5 y  11 

Diğer denklemde y yerine
18  3x
yazalım.
4
 18  3x 
2x  5
  11  8 x  90  15 x  44  23 x  44  90  23 x  46  x  2 dir.
 4 
Verilen denklemlerin birinde x ' in değerini yazalım.
3   2   4 y  18  6  4 y  18  6  18  4 y  12  4 y  y  3
Ç   x, y   x  2, y  3
3-GRAFĠK YÖNTEMĠ
ax  by  c
verilen denklemlerin ayrı ayrı grafikleri çizilir.

dx  ey  f
Çizilen doğruların kesim noktası bizim aradığımız denklemin kökleridir.
Örnek:
x y 3
y  3x  1 denklemini grafik yöntemiyle çözünüz.
Çözüm:
y  3x  1
3
Kesim
noktası
Ç  1,2
2
-1
1
3
x y 3
98
Örnekler:
1-Ahmet, Can'dan 6 yaş büyüktür. Ġki yıl sonra Ahmet'in yaşı Can'ın yaĢının 3 katı
olacağına göre Ahmet ve can 'nın yaĢları nedir?
Çözüm:
x
y olsun.
Ahmet'in Yaşı
Can'ın yaĢı
x y6
x  2  3 y  2
Ahmet can' dan 6 yaş büyüktür.
Ġki yıl sonra Ahmet'in yaĢı Can'ın yaĢının 2 katı olacağına göre
şeklinde denklemler yazılabilir.
 x  y  6
çözelim.  y  6   2  3  y  2   y  8  3 y  6

 x  2  3  y  2 
8  6  3 y  y  2  2 y  y  1 Can'ın yaĢı 
x  y  6  x  1  6  7  Ahmet'in yaşı 
2  A ile B şehirleri arası 600 km dir. A dan ve B den aynı anda birbirlerine
karşı hareket eden iki araç 4 saat sonra karşılĢıyor. Araçlardan birinin hızı
diğerinin
2
si kadar olduğuna göre bu araçların hızlarını bulunuz.
3
Çözüm:
4v1  4v2  600

v1 , v2 olsun. 
olarak yazabiliriz.
2
v1  3 v2
I. araç
II. araç
v1  v2  150

2 / v1  v2  300 2v1 2v2  300



 5v1  300  v1  60 km saat

2v2
3
v

2
v

0
v


0
 1
2
 1 3
3v1 2v2  0
2
2
180
v1  v2  60  v2  v2 
 90 km saat
3
3
2
3  Toplamları 120, farkları 20 olan sayıyı bulunuz.
Çözüm:
x , y olsun.
I. sayı
II. sayı
 x  y  120
 2 x  140  x  70 ve y  50 bulunur.

x

y

20

RASYONEL ĠFADELERĠN BASĠT KESĠRLERĠN TOPLAMI OLARAK
YAZILMASI
Tanım:
99
a, b, c, A, B, C  , m, n   ve ax 2  bx  c asal bir polinom ise,
A
Bx  C
,
m
n
2
 ax  b 
 ax  bx  c 
biçimindeki rasyonel fonksiyonlara basit kesir denir.
Örnek:
4
 3x  2 
,
5
3
4x  2
2
fonksiyonları birer basit kesir
,
2
2
x x2
x 2
fonksiyonlardır.
3
5x2  3
5x  4
fonksiyonları ise basit kesir fonksiyon
,
,
2
4
x  2x  3
x  4  x  a  x  b 
değildir.
I.
ġimdi,
P  x
Q  x
indirgenemez rasyonel fonksiyon
II. der  P  x    der Q  x  
III. Q  x  polinomu aralarında asal M  x  ve N  x 
polinomlarının çarpımı ise,
P  x
Q x
rasyonel fonksiyonu
A x  B  x 
,
indirgenemez rasyonel
M  x N  x
kesirlerin toplamı olarak yazılabilir.
Örnek:
12
rasyonel ifadesini basit kesirlere ayıralım.
 x  2  x  2 
Çözüm:
A x  2  B  x  2
12
A
B



 x  2  x  2   x  2   x  2 
 x  2  x  2 
Ax  2 A  Bx  2 B x  A  B   2  A  B 
12


 x  2  x  2 
 x  2  x  2 
 x  2  x  2 

 A  B  0 

 2 A  6  A  3, B  3 bulunur.
2  A  B   12  A  B  6 
denklemini çözelim 

12
3
3


olur.
 x  2  x  2   x  2   x  2 
A B  0
Örnek:
5x  3
rasyonel ifadesini basit kesirlere ayıralım.
3x  10 x  8
Çözüm:
2
100
5x  3
, 3 x 2  10 x  8   3 x  4  x  2 
3 x  10 x  8
A  x  2   B  3x  4  Ax  3Bx  2 A  4 B
5x  3
A
B




2
3 x  10 x  8  3x  4   x  2 
 3x  4  x  2 
 3x  4  x  2 
2
A  x  2   B  3x  4 
 3x  4  x  2 

5x  3
 A  x  2   B  3x  4   5 x  3
 3x  4  x  2 
polinomları özdeĢ olduğu için,
x  2 için A  2  2   B  3  2  4   5  2  3  2 B  7  B 
7
2
4
4
2
11
11
4

 4

için A   2   B  3   4   5   3   A   B   dir.
3
3
3
3
2
3

 3

11
7
5x  3
2

 2
dir.
2
3 x  10 x  8  3x  4   x  2 
x
Not:
Kök olan x ler belirlenirken verilen rasyonel ifadenin paydasını sıfır yapan
değerler kök olarak belirlenecektir.
Örnek:
2x  7
rasyonel ifadesini kesirlerin toplamı olarak yazalım.
8x  2 x  1
Çözüm.
2
101
2x  7
, 8 x 2  2 x  1   4 x  1 2 x  1
8x  2 x  1
2x  7
A
B


yazılır.
2
8 x  2 x  1  4 x  1  2 x  1
2
1
2x  7
2x  7
A yı bulmak için  4 x  1  0  x   

4
 4 x  1   2 x  1  2 x  1
 1
2
2  28 26
2    7
 7
1
26
13
4
4
4
de x yerine  yazalım. 
 4



  A
2  4
6 6
4
3
  1    2 1
 2   4   1
4
4
4
 
 
1
2x  7
2x  7
B yi bulmak için  2 x  1  0  x  

de x yerine
2
 4 x  1   2 x  1  4 x  1
1
2  7
1
8
2
yi yazalım.
  B dir. O halde,
2
 1  3
 4   1
 2 
13
8

2x  7
3 
3

yazılır.
8 x 2  2 x  1  4 x  1  2 x  1
NoT: Paydanın gerçek kökleri yoksa yukarıda verilen kural geçerli değildir.
Örnek:
Çözüm:
5x  2
rasyonel ifadesini basit kesirlere ayırınız.
x3  2 x
5x  2
 x3  2 x  x  x 2  2 
3
x  2x
A   x 2  2    Bx  C  x
5x  2
5x  2
A Bx  C

 

x3  2 x x  x 2  2  x  x 2  2 
x  x2  2
2
Ax 2  Bx 2  2 A  Cx  A  B  x  2 A  Cx 5 x  2

 3
x  2x
x  x2  2
x  x2  2
A B  0 
A 1

C  5  B  1 bulunur.
2 A  2  C  5
5x  2
5x  2
1 x  5

  2
olarak yazılır.
3
2
x  2x x  x  2 x  x  2
ALIġTIRMALAR
102
A, B nin alacağı değerleri bulunuz.
1
4
 x  1 x  4 

A
B

x 1 x  4
2 x  11
A
B


2
x  x6 x2 x3
Aşağıdaki rasyonel ifadeleri basit kesirlerin toplamı biçiminde yazınız.
2
3x
a
2
4 x  16 x  15
1  3x
d 3
x  x  x2  1
4x  1
b 2
x  13x  3
2 x 2  3x  2
e 3
x  x2  2x
2 x2  x  3
c
x3  8
2x2  7 x  3
f  3
x  2x2  x  2
ĠKĠNCĠ DERECEDEN BĠR BĠLĠNMEYENLĠ DENKLEMLER
Tanım:
a, b, c  ve a  0 olmak üzere ,
ax 2  bx  c  0 denklemine ikinci dereceden bir bilinmeyenli denklem denir.
Bu denklemin iki reel kökü vardır. Bu kökler x1 , x2 olmak üzere çözüm kümesi,
Ç   x1 , x2  dir.
Örnek:
AĢağıda verilen denklemler ikinci dereceden bir bilinmeyenli denklemlerdir.
a  3x 2  5 x  1  0
b  x2  x  1  0
1 2
1
x x 0
2
4
2
d  2 x 2  3x   0
3
2
e  0,002 x  2,3x  1,54  0
c
Örnek:
103
1  2 x3  3x 2  x  1  0 3.dereceden bir denklem
2- 4 x 2  x 1  1  0 denklemi ax 2  bx  c  0 denklemine benzemez
3- x 2  y 2  2 x  3  0 denklemi 2.dereceden fakat iki bilinmyenlidir.
ax 2  bx  c  0 denkleminde   b 2  4ac olmak üzere,
1    0 ise denklemin iki reel kökü var. Bunlar , x1,2 
b

2a
dir.
Fonksiyon x  eksenini x1 ve x2 noktasında keser.
2    0 ise denklemin birbirine eşit iki kökü var. x1  x2  
b
dir.
2a
Fonksiyon x  eksenini x1  x2 noktasında keser.
3    0 ise denklemin reel kökü yoktur. Fonksiyon x  eksenini
kesmez.
Örnekler:
7
1   x 2  0 denklemini çözünüz.
8
Çözüm:
7
 x 2  0  7 x 2  8  0  7 x 2  0  x 2  0  x  x  0
8
x1  x2  0 dır. Ç  0
2  5 x 2  4 x  0 denkleminin çözüm kümesini bulunuz.
Çözüm:
5 x 2  4 x  0  a  5, b  4, c  0 dır.
  b 2  4ac   4   4  5  0  16  0  16    16  0
2
olduğundan iki reel kök vardır.
b     4   16 4  4 0



0
2a
25
10
10
b     4   16 4  4 8 4
x2 




2a
25
10
10 5
 4
Ç  0,  dir.
 5
x1 
104
3  27 x 2  3  0 denklemini çözünüz.
Çözüm:
27 x 2  3  0 , a  27, b  0, c  3
  b 2  4ac  02  4  27  3  0  324  324  0
olduğundan denklemin reel kökü yoktur.
Ç 
Çözümü 2.yol olarak , aşağıdaki Ģekildede yapılabilir.
1
27 x 2  3  0  27 x 2  3  9 x 2  1  x 2  
9
1
karesi  eşit olan hiçbir reel sayı olmadığına göre
9
çözüm kümesi boş kümedir.
4  4 x 2  16  0 denkleminin çözüm kümesini bulunuz.
Çözüm:
4 x 2  16  0 , a  4, b  0, c  16 dır.
  b 2  4ac  02  4  4   16   256  0
olduğundan denklemin iki reel kökü vardır.
x1 
b     0   256 16


 2
2a
24
8
b     0   256 16


2
2a
24
8
Ç  2, 2 dir.
x2 
veya,
4 x 2  16  0  4 x 2  16  x 2  4  x1  2 ve x2  2 dir.
5  x 2  4 x  21  0 denklemin çözüm kümesini bulunuz.
Çözüm:
x 2  4 x  21  0 , a  1, b  4, c  21
  b 2  4ac   4   4  1   21  16  84  100  0
2
o halde iki reel kök vardır.
b     4   100 4  10 6



 3
2a
2 1
2
2
b     4   100 4  10 14
x2 



7
2a
2 1
2
2
Ç  3,7 dir.
x1 
105
6  6 x 2  5 x  1  0 denklemimin çözüm kümesini bulunuz.
Çözüm:
6 x 2  5 x  1  0 , a  6, b   5  , c  1
  b 2  4ac   5   4  6  1  25  24  1  0
2
o halde iki reel kök vardır.
b     5   1 5  1 4 1




2a
26
12 12 3
b     5   1 5  1 6 1
x2 




2a
26
12 12 2
1 1 
Ç   ,  dir.
3 2 
x1 
7  3x 2  2 x  7  0 denkleminin çözüm kümesini bulunuz.
Çözüm:
3x 2  2 x  7  0 , a  3, b  2, c  7
  b 2  4ac   2   4  3  7  4  84  80  0
2
o halde reel kök yoktur. Ç   dir.
8  abx 2   a  b  x  1  0 denklemini çözünüz.
Çözüm:
abx 2   a  b  x  1  0 ,
     a  b    4  ab  1  a 2  2ab  b 2  4ab  a 2  2ab  b 2   a  b 
2
2
o halde iki reel kök vardır.
x1 
x2 
    a  b   
a  b
2  ab
    a  b   
a  b
2  ab

a  b  a  b 

a  b  a  b 
2
2
2ab
2ab
2b
1

2b a a
2a
1

2a b b
1 1
Ç   ,  dir.
a b
9  mx 2   2m  3 x  m  1  0 denkleminin köklerinin birbirine eşit olması için
m?
Çözüm:
  b 2  4ac  0 olmalıdır.
     2m  3   4  m   m  1  0
2
  4m 2  12m  9  4m 2  4m  0
  4m 2  12m  9 4m 2  4m  0  8m  9  0  m  
9
dir.
8
106
10   m  3 x 2  2mx  5m  1  0 denkleminin köklerinden birinin 2 olması
için m 'nin alacağı değeri bulunuz.
Çözüm:
x  2 olarak alalım.
 m  3 x 2  2mx  5m  1  0   m  3 22  2m  2  5m  1  0
 m  3 4  4m  5m  1  0  4m  12  9m  1  13m  13  0
m  1 dir.
11  mx 2  2 x  1  0 denkleminin iki reel kökü olması için m nin
alacağı değerler kümesini bulalım.
Çözüm:
mx 2  2 x  1  0 ,   b 2  4ac  0 olmalıdır.

 2
2
 4m  1  0  2  4m  0  4m  2  m  
1
dir.
2
1


Ç  m / m   , m   dir.
2


4
12  Değeri olan bir kesrin payına 2 eklenir , paydasında 5 çıkarılırsa
5
kesrin değeri 2 oluyor. Bu kesri bulunuz.
Çözüm:
x 4

5 x  4 y
2 / 5 x  4 y  0 10 x 8 y  0
x  y 5






x  2  2 y  10 4 / x  2 y  12 4 x 8 y  48
y x  2


2
Kesir olsun.  y  5

6 x  48  x  8 , y  10 bulunur.
13  ax  bx  a   b 2 x  ab denkleminin çözüm kümesini bulunuz.
Çözüm:
ax  bx  a   b 2 x  ab  abx 2  a 2 x  b 2 x  ab
 abx 2   a 2  b 2  x  ab  0 haline getirilir.
  b 2  4ac olduğundan  
 a
2

 b 2   4  ab  ab  a 4  2a 2b 2  b 4  4a 2b 2
2
 a 4  2a 2b 2  b 4     a 2  b 2  dir.
2
2
2
2
2
b    a  b    a  b 
a 2  b 2  a 2  b 2 2b 2 b
x1 




2a
2ab
2ab
2ab a
2
2
2
2
2
b    a  b    a  b 
a 2 b 2  a 2 b 2 2a 2 a
x2 




2a
2ab
2ab
2ab b
b b 
Ç   ,  dır.
a a 
2
107
9- BÖLÜM
SAYI EKSENĠ VE REEL SAYILARDA SIRALAMA
SAYI EKSENĠ
Reel sayıların geometrik modelini oluĢturmak çok faydalıdır. Bunu oluĢturmak
için önce bir doğru çizilir ve bir baĢlangıç noktası ( orijin ) seçilir. a  olsun.
1. a  0 ise baĢlangıç noktasının sağında ve baĢlangıçta a birim
uzaklıktaki noktaya
2. a  0 ise baĢlangıç noktasının solunda ve baĢlangıçta a birim
uzaklıktaki noktaya
3. a  0 ise baĢlangıç noktasına karĢılık gelir.
Bu Ģekilde her bir reel sayı sayı-eksenine yerleĢtirilir. Elde edilen, sayı ekseni
veya sayı doğrusudur.
Örnek:
5
3, 1,0,2, 3, ,  sayılarını sayı doğrusu üzerinde yerleştiriniz.
2
Çözüm:
3
Burada ,
1
0
3
2
5
2

3  1.732 ve   3,14 olarak alınabilir.
REEL SAYILARDA SIRALAMA ÖZELLĠKLERĠ
a, b reel sayıları için a  b ve b, a nın sağında yer alıyorsa, bu durum için
'' a küçüktür b '' denir. a  b şeklinde gösterilir. Eğer, a  b veya a  b ise
a  b şeklinde yazılarak '' a küçük-eĢit b '' diye okunur.
a  b  0  b  a veya b  a  0 olduğu açıktır.
a, b, c  olsun.
1  a  b ve b  c  a  c dir.
2  a  b, c  d  a  c  b  d dir.
3  a  b ve k  olsun. a  k  b  k dır.
4  a  b, k  0  ak  bk
k  1  a  b dir.
ve k  0  ak  bk dır.
1
0
a
1 1
60 a b  .
b a
7  a  b ise a  b veya a  b dir
a  0  a  0 veya a  0 dır
5a  0
108
Örnek:
AĢağıdaki sayıları sıralayınız.
3 7
a)
,
5 4
1
b)  4, 
2
1 3 13
c)
, ,
2 4 4
Çözüm:
3 7
,
, sayıların paydalarını eĢitleyelim.
5 4
12 28
,
dir. Pozitif kesirlerde,
20 20
payda eşit ise payı büyük olan kesir diğerinden büyüktür.
a)
12 28
3 7

  dir.
20 20
5 4
1
1
b)  4,  ,  4  
2
2
1 3 132
c)
, ,
, Negatif kesirlerde,
2 4
4
payda eşit ise payı büyük olan kesir diğerinden küçüktür.
O halde,

132
3 1
   dir.
4
4 2
Örnek:
AĢağıda verilen sayıları sıralayınız.
2 21 11 4
a)
, , ,
3 30 15 5
b)   , 3.14
c)
  , 3.14, 3.14, 
Çözüm:
a)
b)
c)
2 21 11 4
20 21 22 24
2 21 11 4
, , ,  paydaları eĢitleyelim.
, , ,  
  dir.
3 30 15 5
30 30 30 30
3 30 15 5
  , 3.14    3.14
  , 3.14, 3.14,     3.14  3.14   olur.
ARALIKLAR
ġimdi
’nin aralık adını vereceğimiz alt kümeleri üzerinde duralım.
109
ye ait a, b  a  b  gibi herhangi iki reel sayı arasındaki tüm reel sayılardan oluĢan
nin bir alt kümesine bir aralık denir. Buradaki a, b sayılarına aralığın uç noktaları denir.
a sayısına aralığın alt ucu, b sayısına aralığın üst ucu denir.
Geometrik olarak aralık bir doğru parçasıdır. Uç noktaların oluĢturulan kümeye
ait olup olmamasına göre aralıklar çeĢitli isim alırlar.
 Uç noktalarının her ikisi kümeye ait değilse bu aralık Ģekline açık aralık denir.
 Uç noktalarının her ikisi kümeye ait ise bu aralık Ģekline kapalı aralık denir.
 Uç noktalardan yalnızca biri kümeye ait ise bu aralık tipine yarı-açık aralık denir.
Bunlarda sağdan kapalı soldan açık veya sağdan açık soldan kapalı olarak iki farklı
Ģekilde bulunur.
Aralığın bir ucu açık ise açık parantezle, kapalı ise köĢeli parantezle gösterilir.
Kümesel
yazılıĢı
x 
x 
/ a  x  b
/ a  x  b
GösteriliĢi
 a, b 
 a, b 
OkunuĢu
Geometrik modeli
a, b açık aralığı
a
b
a
b
a, b kapalı aralığı
x 
/ a  x  b
 a, b 
a, b yarı açık
x 
/ a  x  b
 a, b 
a, b yarı açık
a
a
b
b
Aralıklar pozitif veya negatif yöne doğru sınırsız olarak geniĢletilebilirler.  eksi
sonsuz ve  sonsuz olmak üzere,
SONSUZ ARALIKLAR
Kümesel
yazılıĢı
GösteriliĢi
OkunuĢu
x 
/ x  a
 a, 
a da kapalı
x 
/ x  a
 a,  
a da açık
x 
/ x  b
 ,b
b de kapalı
x 
/ x  b
 ,b 
b de açık
Geometrik modeli

a

a

b

b
Örnek:
AĢağıdaki aralıkları sayı doğrusunda gösteriniz.
110
a)
b)
c)
d)
 1,3
 2,5
 3,3
 0, 4
Çözüm:
-1
3
2
-3
5
3
0
4
111
10-BÖLÜM
EġĠTSĠZLĠKLER
Tanım:
a, b 
, a  0 olmak üzere ,
ax  b  0 , ax  b  0 , ax  b  0 , ax  b  0
biçminde yazılan eĢitsizliklere 1.dereceden bir bilinmeyenli
eşitsizlikler denir.
Bir eĢitsizliğin çözüm kümesi, onu sağlayan x  lerin kümesidir.
Bir eĢitsizliğin çözümü için iki yol vardır.
1. ĠĢaret tablosu oluĢturarak eĢitsizliği çözmek
2. Direkt çözüm
1- ĠġARET TABLOSU ĠLE ÇÖZÜM
ax  b  0  x  
b
dır.
a

ax  b

b
a
a ' nın iĢaretinin 0
tersi

a 'nın iĢaretinin aynısı
Örnek:
3  5x  3x  9 eşitsizliğinin çözüm kümesini iĢaret tablosunu inceleyerek bulunuz.
Çözüm:
3  5 x  3x  9  5 x  3x  9  3  2 x  12  x  6  x  6  0
x  6  0  x  6 bulunur.
x
x6
6


0


Ç çözüm kümesi
Ç  x / x  6, x 
Örnek:
   , 6
f  x   3x  6 ifadesinin işaretini inceleyiniz.
Çözüm:
Tablo ile iĢaretini inceleyelim.
3x  6  0  3x  6  x  2 olarak denklemin kökünü buluruz.
112
x
3x  6

2


0

2 den küçük x ler için  3 x  6   0 ve
2 den büyük x ler için  3 x  6   0 dır.
Örnek:
7  2  5x  9 eşitsizliğinin çözümünü bulunuz.
Çözüm:
7  2  5 x  9  7  2  5 x  9  2  5  5 x  7 
5
7
7
 x    1  x  
5
5
5
7
 7

   x  1  Ç    , 1
5
 5

Örnek:
5  x  2   9 x  3 2 x  4  eşitsizliğinin çözüm kümesini bulunuz.
Çözüm:
5  x  2   9 x  3  2 x  4   5 x  10  9 x  6 x  12  5 x  10  3x  12
5 x  3x  12  10  2 x  22  x  11  x  11  0 şeklinde yazarız.
x  11  0  x  11 dir.
x

11
 x  11

0


Çözüm kümesi
Yani, Ç  11,   dir.
Örnek:
Bir malın alıĢ fiyatı x ve satıĢ fiyatı y liradır. SatıĢ için 2 durum söz konusudur.
I.Durum  y  3x  1300
II.Durum  y  7 x  1100
II. Durum I. Durumdan daha kârlı ise x tamsayı olarak en az kaç lira olmalıdır?
Çözüm:
II. Durum > I. Durum  7 x  1100  3x  1300
7 x  3x  1300  1100  4 x  2400  x  600 olur.
Yani, x  601 lira en az olmalıdır.
x  600 alınmıĢ olsaydı II. Durum  I. Durum olurdu.
Örnek:
113
A ve B gibi iki oto kiralama firmasından A firması bir arabayı günlük 30 lira ve
km başına 25 krĢ'a, B firması ise aynı marka bir arabayı günlük 40 lira ve km baĢına
20 krş kiraya veriyor. A firmasından bir haftalığına araba kiralayan bir kişinin bu
firmaya ödeyeceği paranın, B firmasına ödeyeceği paradan az olması için bu kiĢi aracı
en fazla kaç km aracı kullanmalıdır?
Çözüm:
7  20  x  25  7  40  x  15  140  25 x  280  15 x
25 x  15 x  280  140  10 x  140  x 
140
 x  14
10
Yani, en fazla 13 km kullanmalıdır.
Örnek:
4 x  2  12
 eşitsizliğinin çözüm kümesini bulunuz.
2 x  1  17 
Çözüm:
Önce iki eşitsizliğin ayrı ayrı çözüm kümesini bulacağız. Sonra bu kümelerin
kesişimini alacağız.
4 x  2  14  4 x  14  2  4 x  12  x  3  Ç   x / x  3 , x 

a  4  0 dır.
x
3


0
2 x  1  17  2 x  17  1  2 x  16  x  8  Ç   x / x  8, x 
8
x
2 x  16



x  3 ve x  8  3  x  8 veya Ç   3,8 
2- DĠREKT ÇÖZÜM
Örnek:
12 x  20  15x  4 eşitsizliğinin çözüm aralığını bulunuz.
Çözüm:
12 x  20  15 x  4  20  4  15 x  12 x  24  3x  8  x
Ç   x / 8  x, x 
  8,  
Örnek:
6  5x  2 x  8 eşitsizliğinin çözüm aralığını bulunuz.
Çözüm:
114
6  5 x  2 x  8  6  8  2 x  5 x  14  7 x  2  x
Ç   2,  
BĠR BĠLĠNMEYENLĠ ĠKĠNCĠ DERECEDEN EġĠTSĠZLĠKLER
ax 2  bx  c  0, ax 2  bx  c  0 , ax 2  bx  c  0 , ax 2  bx  c  0
biçimindeki eşitsizliklere 2.dereceden bir bilinmeyenli eşitsizlikler denir.
Bu tür eşitsizliklerin çözümü için ax 2  bx  c üç terimlisinin işareti
incelenmelidir.
I. DURUM
  b2  4ac  0 ise ax 2  bx  c  0 denkleminin iki farklı reel kökü vardır.
x1  x2 olmak üzere kökler x1 , x2 dir.
x
ax2  bx  c
x1

a nın iĢaretinin
aynısı
x2
a nın iĢaretinin
tersi

a nın iĢaretinin
aynısı
II. DURUM
  b2  4ac  0 ise ax 2  bx  c  0 denkleminin iki eşit kökü vardır.
Kökler x1 , x2 dir.
x
ax2  bx  c
x1  x2  

b
a
a nın iĢaretinin
aynısı

a nın iĢaretinin
aynısı
III. Durum
  0  ax 2  bx  c  0 ın reel kökü yoktur. Bu durumda,
i  a  0  daima ax 2  bx  c  0
ii  a  0  daima ax 2  bx  c  0
Örnek:
a  3x 2  3x  4
 işareti daima
 işareti daima
b  x2  5x  4
pozitif 
negatif  olur.
c  x 2  6 x  9  0 d  3x 2  x  4  0
eşitsizliklerini çözünüz.
Çözüm:
115
a  x 2  3x  4  x 2  3x  4  0 ,
  b 2  4ac   3  4  1   4   9  16  25  0
2
x1 
b

2a
bulunur.

  3
25
2 1
x

3 5
 x1  1, x2  4
2
1


x 2  3x  4
4



Ç   , 1   4,  
b
x2  5x  4  x2  5x  4  0 ,
  b 2  4ac   5   4  1   4   25  16  9  0
2
x1 
b

2a
bulunur.

  5 
x
x2  5x  4
2 1
9

5 3
 x1  1, x2  4
2
4
1





Ç  1,4
116
c  x 2  6 x  9  0,
  b 2  4ac  62  4  1  9
x
 36  36  0
x1  x2  
-3


b
6
6 x2  6 x  9


2a
2 1 2


x1  x  3
Ç  3 tür.
d  3x 2  x  4  0  3x 2  x  4  0 olduğundan,
 =b 2  4ac  12  4  3  4  1  48  47  0 olarak
bulunur. Bu denklemin reel kökü yoktur.
a  3  dır. O halde 3x 2  x  4 daima pozitiftir.
Örnek:
3x  6
 0 eşitsizliğinin çözüm kümesini bulunuz.
x5
Çözüm:
3x  6  0  x  2 ve x  5  0  x  5 tir.
x
-5
3x  6
x5



3x  6
x5
EĢitsizliklere kullanılarak
Ç   , 5    2,  
2






x  5 paydanın kökü olduğundan
çözüm kümesine dahil edilmemiştir.
p  x   q  x  biçimindeki denklemler çözülebilir.

 p  x  0
p  x  q  x  
ve p  x   q 2  x  dir.

q  x   0
Uygulamada p  x   q 2  x  denklemi çözülür ve çıkan çözümden orijinal denklemi
sağlayan değer çözüm kümesi olarak alınır.
Örnek:
117
4 x  3  2 denklemini çözünüz.
Çözüm:
4x  3  2 
4x  1  x 

4x  3

2
  2  4x  3  4  4x  4  3  1
2
1
1 
Ç  
4
4
Örnek:
x  2  5  0 denkleminin çözüm kümesini bulunuz.
Çözüm:
x2 5 0 x2 5

x2

2
 52  x  2  25  x  25  2  27
Ç  27
Örnek:
3
x  2  3  0 denkleminin çözüm kümesini bulunuz.
Çözüm:
3
x  2  3  0  3 x  2  3 

3
x2

3
  3  x  2  27  x  27  2  29
3
Ç  29
Sağlama:
3
x  2  3  0  3 29  2  3  0  3 27  3  0 
3
 3
3
 3  0  3  3  0 dır.
Örnek:
x  1  x  4  5 denklemini çözünüz.
Çözüm:
x 1  x  4  5 

x 1  x  4

2
 52  x  1  2  x  1  x  4  x  4  25
x  1  2    x  1   x  4    x  4  25  2 x  3  2   x 2  3 x  4   25
2 x  2 x 2  6 x  8  22  2 x 2  8 x  30  0  x 2  4 x  15  0
  b 2  4ac  42  4  1   15   16  60  76  0 o halde denklemin iki farkı reel kökü
vardır.
x1 
b   4  76 4  4  19 4  2 19



 2  19
2a
2 1
2 1
2
x1 
b   4  76 4  4  19 4  2 19



 2  19
2a
2 1
2 1
2


Ç  2  19, 2  19 dir.
Sağlaması ile doğruluğu görülmeye çalıĢılabilir.
AĢağıda verilenler kullanılarak mutlâk değerli denklemler çözülebilir.
118
p  x   a  p  x   a olan x ler,
p  x   a  a  p  x   a olan x ler,
p  x   a  p  x   a ve p  x   a olan x değerleri çözüm olur.
Örnek:
1
4
2x  3  1
x3
eşitsizliklerinin çözüm kümesini bulunuz.
Çözüm:
2 x  5  3
2 x  8  x  4
a)
2x  5  3  


 Ç  1, 4
2 x  5  3 2 x  2  x  1
1
1
1
1
b)
4
 4   x  3  x  3  haline getirilir.
x3
x3
4
4
2x  5  3
1
1
ve x  3 
yazılabilir.
4
4
1
1
11 
11 
x  3    x    3  x    , 
4
4
4
4

Burada ,
x3
x3 
1
1
13  13

 x   3  x    ,  
4
4
4
4

11   13


Ç   ,    ,   dir.
4 4


c) 2 x  3  1  1  2 x  3  1  1  3  2 x  1  3  4  2 x  2
 2  x  1  Ç   2, 1
Örnek:
a)
3x  2  7
b)
x  2 x  18
c)
x 1  3  6
denklemlerinin çözümlerini yapınız.
Çözüm:
3x  2  7 3x  7  2 3x  5 
5
3x  2  7  


 x 
, x  3 bulunur.
3

3x  2  7
3x  7  2
3x  9
 5 
Ç   ,3 tür.
 3 
a)
b) x  2 x  18  x  2 x  18 | x | 2 | x | 18  3 | x | 18 | x | 6
x  6 ve x  6 bulunur. Ç  6,6
c)
| x  1| 3  6 
| x  1| 3
x 1  3  6  


| x  1| 3  6
| x  1| 9
 Mutlâk değer negatif olamaz.
 x  1  9
 x  8
| x  1| 9  

 Ç  8,10 dur.
x 1  9
 x  10
119
Örnek:
1  x  3 olduğuna göre x  1  x  3  x ifadesinin eĢiti nedir?
Çözüm:
1  x  3 olduğuna göre x  1  x  3  x 
 x  1  x  1

1  x  3 olduğundan 
 olarak yazılır.
 x  3    x  3  3  x 
x  1  x  3  x  x  1  3  x  x   x  2  2  x dir.
Örnek:
x 1
 5 eşitsizliğinin çözüm kümesini bulunuz.
2
Çözüm:
x 1
x 1
 5  5 
 5  10  x  1  10  10  1  x  10  1
2
2
9  x  11  Ç   9,11
Örnek:
1  2 x  4 eşitsizliğinin çözüm kümesini bulunuz.
Çözüm:
5

5
x


1  2 x  4 2 x  5  x 

2
1  2x  4  


2 
1  2 x  4
2 x  3
2 x  3  x   3

2
3 5


 Ç   ,     ,  
2 2


120
10. BÖLÜM
TRĠGONOMETRĠ
YÖNLÜ AÇILAR
Tanım:
BaĢlangıç noktaları aynı olan iki ıĢının meydan getirdiği düzlemsel Ģekle açı ve
bu açıyı meydana getiren ıĢınlara açının kenarları denir. BaĢlangıç noktasına açının
köĢesi denir. Açı düzlemsel noktalar kümesidir.
B
Bitim kenarı
D
BaĢlangıç kenarı
O
BaĢlangıç kenarı

O
A

Bitim kenarı

AOB  OA, OB
C

DOC  OD , OC
Açıyı oluĢturan iki ıĢından birine baĢlangıç ve diğerine bitim kenarı denir. Açılar
adlandırılırken önce baĢlangıç daha sonra bitim kenarı yazılır.
Açının köĢesi etrafında, baĢlangıç kenarından bitim kenarına gitmek için iki yol
vardır. Bunlardan biri saatin dönme yönünün tersi diğeri saatin dönüĢ yönü ile aynıdır.
Saatin dönme yönünün tersi pozitif , saatin dönme yönü ise negatif yön dür.
Örnek:
121
B
Bitim kenarı
O
D
BaĢlangıç kenarı
A
BaĢlangıç kenarı

O
Bitim kenarı
C

AOB yönü pozitif
DOC yönü negatif
  
m  AOB 


  
m  DOC 


  
  
m  AOB   m  DOC  dir.




YÖNLÜ YAYLAR
Bir kesen çemberi iki yay parçasına ayırır. AB keseni O merkezli çemberi A ve B
noktalarında kessin. Çember üzerinde A noktasından B noktasına gitmek için iki yön
vardır.
B
C
.
O
A
Saat ibresinin dönme yönünün tersi pozitif yön,
saatin dönme yönü ise negatif yöndür.  A, B 
kriĢinin çemberde ayırdığı iki yayı birbirinde ayırt
etmek için yayların üzerine birer nokta daha
koyabiliriz. Böylece pozitif yönde oluĢan AB yayını
ACB yayı olarak, negatif yönde oluĢan AB yayını
ise ADB yayı olarak adlandırabiliriz.
D
Yarı çapları eĢit olan iki çembere eĢ çemberler denir. Aynı ve eĢ çemberlerde, eĢ
kiriĢlerin ayırdığı yaylara eĢ yaylar denir. Aynı veya eĢ çemberlerde eĢ yayları gören
merkez açılar birbirine eĢittir.
 A,B kiriĢi O merkezden geçerse  A,B kiriĢi çap olur. Bir  A,B çapının
çemberde ayırdığı ACB yayının uzunluğu ADB yayının uzunluğuna eĢit olur. Yani bir
çemberi çap iki eĢit parçaya böler. Bir  A,B çapının ayırdığı yaylardan her birine yarı
çember denir.
122
Uzunluğu yarı çemberden küçük olanına küçük (minör) yay, uzunluğu yarı
çemberden büyük olanına büyük ( major ) yay denir.
BĠRĠM ÇEMBER
Analitik düzlemde, merkezi baĢlangıç noktasında ve yarıçapı 1 birim uzunlukta
olan çembere birim çember denir. Birim çember üzerindeki her P  x, y  noktası için
x 2  y 2  1 bağıntısını sağlar. Dolayısıyla birim çember,
Ç   x, y  / x, y  , x 2  y 2  1 dir.
r 1
 1,0 
O
P  x1 , y1 
1,0 
Q  x2 , y2 
AÇI ÖLÇÜ BĠRĠMLERĠ
Bir açının büyüklüğünü veya küçüklüğünü nasıl belirleyebiliriz. Bunu
yapabilmek için açı ölçü birimi tanımlamak gerekir. Çember üzerinde hareketli bir P
noktası A dan baĢlayarak pozitif yönde bir tam dönme açısı yapsın ve tekrar A
noktasına gelsin. P noktasının çizdiği
ABA tam çember yayını gören merkez açıya tam açı denir. Tam açı evrenseldir. Yani,
açının çizildiği yere ve zamana veya açıyı çizene bağlı değildir.Bir çember yayının
uzunluğu da 2 olduğuna göre bir tam açı 2 ye eĢittir.
Açı ölçüleri üç farklı Ģekilde gösterilir.
1.
Derece :
Birim çember yayını 360 eĢit parçaya bölelim. EĢ yayları gören merkez açılar
biribirine eĢit olduğundan 360 tane eĢ merkez açı oluĢur. Bunlardan birinin merkez
açı ölçüsüne 1 derece denir. Dereceyi ( o ) simgesiyle göstereceğiz. Buna göre,
bir tam açı 3600 dir.
Daha duyarlı ölçümler için derecenin as katları kullanılır.
1 nin 60 ta birine 1 dakika denir. Dakika ' simgesiyle gösterilir.
1' nin 60 ta birine 1 saniye denir. Dakika '' simgesiyle gösterilir.
0
2. Grad :
Tam çember yayını 400 eĢit parçaya böldüğümüzde bu parçalardan her birini
gören merkez açının ölçüsüne 1 grad denir.
123
3. Radyan :
Yarıçap uzunluğuna eĢit uzunluktaki bir yayı gören merkez açının ölçüsüne 1
radyan denir. Radyan R simgesiyle gösterilir.
Not: Açı ölçü birimlerinden derece günlük hayatta, grad denizcilikte, radyan ise bilimsel
alanlarda daha sıkça kullanılmaktadır.
Açı ölçülerini birbirine çevirmek için,
D
G
R
formülünü kullanacağız.


180 200 
Örnek:
450 ve 750 derecelik açıları grad ve radyana çeviriniz.
Çözüm:
Formülümüz,
D
G
R
dir.


180 200 
Burada, D yerine 45 ve 75 yazarak G ve R yi bulacağız.
45
G
45 R
ve
yi


180 200
180 
çözelim.
45
G

 G  180  45  200  4G  200  G  50 grad
180 200
4
1
45 R

  R  180  45    4 R    R  dir.
180 
4
4
1
Örnek:
2
6
ve
radyanları grad ve dereceye çeviriniz.
5
7
Çözüm:
2
2
R
G
G
2
G
 
 5 


 2  200  G  5  G  80 grad
5
 200

200
200
5
40
2
2
R
D
D
2
D
 
 5 


 2  180  D  5  D  720 derece
5
 180
 180 5  180
36
6
R

7

6
R

7

6
G
G
6
G
6  200

 7 


 6  200  G  7  G 
 171.428 grad
200

200
7
7  200
6
D
D
6
D
6  180

 7 


 6  180  D  7  D 
 154.2850 derece
180
 180 7  180
7
124
Örnek:
A

4
9
2
B
ġekildeki çemberin yarıçapı 2 birimdir.  açısının gördüğü
4
yayın uzunluğu
olduğuna göre,  açısının derece
9
cinsinden değerini bulunuz.
Çözüm:
N
A
2 4
 9 9
2 M B
Önce  açısının radyan olarak karĢılığını bulalım. Bunun
için yarıçapı 1 birim olan bir çember daha çizelim. NM
1
1 4 2
yay uzunluğu ise NM  AB  
olur. O

2
2 9
9
2
halde  açısının radyan olarak değeri
radyandır.
9
2
D
R
D
 
 9
Derece olarak değeri
180 
180 
D
R
D
2
2  180
 

D
 400 dir.
180 
180 9 
9
Genel olarak, r yarıçaplı bir çemberde merkez açının gördüğü yayın uzunluğu
ise, aynı açının birim çemberde gördüğü yayın uzunluğu
bir çemberde
olacağından, r yarıçaplı
r
uzunluğunda bir yayı gören merkez açının ölçüsü  ise,

radyan olur.
r
Bu bağıntıdan faydalanarak ,    r olduğu rahatça görülebilir. Yani, r yarıçaplı bir
çemberde  açısının gördüğü yayın uzunluğu, yarıçapla açı değerinin çarpımına eĢittir.
Örnek:
125
Yandaki
A
r  12

çemberde AB

3
B
Çözüm:
   r olduğundan 

3
 12
 4 birim
yay uzunluğu
nedir bulunuz.
YÖNLÜ AÇILARIN ÖLÇÜLMESĠ

Ox - ekseni ile çakıĢan bir Ot  sayı ekseni düĢünelim. Bir AOP açısı verilsin.



m  AOP   t olduğunu varsayalım. Birim çember üzerinde AP yayının yönlü


uzunluğu t olacaktır. Ot  sayı ekseni üzerinde bir tek T noktası vardır. Yani her açının
radyan cinsinden değerine karĢılık bir tek gerçek sayı vardır. Tersine olarak her t
gerçek sayısına karĢılık olarak Ot  sayı ekseni üzerinde bir tek T  t ,0  noktası vardır.
Bu noktaya karĢılık, yönlü uzunluğu t olan bir AP yayı vardır. Bu yayı gören merkez
açının ölçüsü t radyandır.
Burada Ģuna dikkat edilmelidir. Açı ölçüleri ile gerçek sayılar arasında birebir
eĢleĢme vardır.
Yarıçapları eĢit olan iki yay toplanıp, çıkarılabilir. Aynı zaman da bir skaler ile
çarpılabilir.

Bir çemberin tamamını ( tam yay ) T ile gösterelim. AOP açısının ölçüsü 
olsun.
AP , AP  T , AP  2T , AP  3T , ..., AP  nT
n  
yaylarını düĢünelim.

Bu yayları gören merkez açılar AOP açısı ile çakıĢır.
Bu açılar derece cinsinden,
 ,   360,  2  360,  3  360,...,  n  360 dır.
Bu açılar radyan cinsinden,
 ,   2 ,  2  2 ,  3  2 ,...,  n  2 dir.
00 ile 3600 arasında olan ölçüye AOP nin esas ölçüsü denir. Diğer taraftan bir
T tam yayının uzunluğu 2 olduğuna göre P  n  2 noktası n defa çemberi pozitif
yönde sarma iĢleminde sonra P noktası ile çakıĢır. Yani, P  n  2 noktası AP  n  T
yayının bitim noktasına gelir. Negatif yönde sarma iĢlemi de aynı sonucu verir.
Dolayısıyla,
S  P  k  2   P
,  k   dir.
sarma
Örnek:
AĢağıdaki açıların esas ölçülerini bulunuz.
a) 7380 b) 13450 c) 6750
126
Çözüm:
a ) Birim çember üzerinde OA başlangıç
90
kenarı olmak üzere 7380 lik
açının bitim kenarını bulmak için,
P
A noktasında pozitif yönde 2 defa
3600 lik yay çizdiğimizde A noktasına geliriz. 180
Bunun için, 7380 den 7200
A
0
O
çıktığımızda kalan 18 lik yayı çizdiğimizde
0
7380 lik açının bitim kenarının
çemberi kestiği P noktasını buluruz.
OP ıĢını 7380 lik açını bitim kenarı
270
ve AOP açısının ölçüsü de
7380 lik açının esas ölçüsü olur.
 
m  AOP   18
m AOP  7380  7200  7380  2  3600
0
bulunur.
Problem daha kısa yoldan da çözülebilir.
7380    k  3600    2  3600    7200
7380    7200    7380  7200  180 dir.
Diğer verilen Ģıklarda ödev olsun.
Örnek.
32
71
radyan ve 
radyanlık açıların esas ölçülerini bulunuz.
3
4
Çözüm:
32
Yapılacak ,
ifadesinde 2 nin katlarını çıkarmak olacaktır.
3
32  32 
2
2 
2


     10     10    olacağından esas ölçü
radyan dır.
3
3
3 
3
 3 



71  71 
3
3



    17      18  1   
4
4
4
 4 





=  18   olacağından esas ölçü
radyan dır.
4
4

Örnek:
Esas ölçüsü 36 derece olan açının açı ölçülerini bulunuz.
127
Çözüm:
  k  360 / k 
  36  k  360 / k   dir.
k  0, 1, 2, 3,... dir.
BĠRĠM ÇEMBER VE AÇILARIN RADYAN CĠNSĠNDEN ÖLÇÜMÜ
AĢağıdaki Ģekli inceleyelim.
B
P

A'
A
0

2

3
2
2
B'
Reel sayılar ile birim çember arasında olduğu gibi, birim çember ile  0,2  aralığındaki
radyan cinsinden sayıları birebir eĢleyelim. Bu eĢlemde birim çember üzerinde bir defa
dönmüĢ olduk. Benzer Ģekilde 2 defa döndüğümüzde  2 ,2  2  , üç defa
döndüğümüzde  4 ,3  2 ... aralığındaki radyan cinsinden sayıları birim çember
üzerindeki noktalarla birebir eĢleĢtirmiĢ oluruz.
Bu eĢlemelerin her birinde P noktasına gelen radyan cinsinden sayıların kümesi;
P    k  2 / k   olur.
Aynı eĢlemede, B ve A ' noktalarına karĢılık gelen radyan cinsinden sayıların kümesi,
7 3  5   


B  ...,  , ,  , ,...    k  2 / k  
2 2
2 2  2


A '  ..., 3 ,  ,  ,3 ,...    k  2 / k   dir.
TRĠGONOMETRĠK FONKSĠYONLAR
128
1. SĠNÜS VE KOSĠNÜS FONKSĠYONU
KöĢesi birim çemberin merkezi, baĢlangıç kenarı x  ekseni ve ölçüsü  olan bir açı
çizelim. Açının bitim kenarının çemberi kestiği nokta P  x, y  olsun. P noktasının
apsisine  nın kosinüsü denir ve kısaca cos ile gösterilir.
B  0,1
P  x, y 
II. Bölge
I. Bölge
sin 
A '  1,0 
A 1,0 

cos
III. Bölge
IV. Bölge
B '  0, 1
Eğer P noktası,
I. Bölgede ise cos  0 , II. Bölgede cos  0 , III. Bölgede cos  0 ve son olarak
IV. Bölgede ise cos  0 dir.
ġekilde de görüleceği gibi,
cos0  1
cos

2
cos   1
0
cos
3
0
2
Yukarıdaki verilenler yardımıyla aĢağıdaki tablo oluĢturulabilir.
Bölgeler
0
I.
cos
1
+
azalıĢ

2
0
II.

III.

-1

azalıĢ
artıĢ
3
2
0
IV.
+
2
1
artıĢ
P noktasının apsisi daima -1 ile 1 arasında bulunacağından, 1  cos  1 dir.
Eğer P noktası,
I. Bölgede ise sin   0 , II. Bölgede sin   0 , III. Bölgede sin   0 ve son olarak
IV. Bölgede ise sin   0 dir.
ġekilde de görüleceği gibi,
129
sin 0  0
sin

2
sin   0
1
sin
3
 1
2
Yukarıdaki verilenler yardımıyla aĢağıdaki tablo oluĢturulabilir.
Bölgeler
0
I.
sin 
0
+
artıĢ

2
1
II.

III.
+
0

azalıĢ
3
2
-1
IV.

2
0
artıĢ
azalıĢ
P noktasının apsisi daima -1 ile 1 arasında bulunacağından, 1  sin   1 dir.
Birim çemberin denklemi x 2  y 2  1 olduğundan sin 2   cos2   1 olacaktır.
  k  2 ile  ve   k  360 ile  ölçülü açılar birim çemberde aynı açıyı
göstereceğinden,
sin   k  2   sin 
sin   k  360   sin 
cos   k  2   cos
cos   k  360   cos
dir.
Örnek:
AĢağıdaki ifadelerin en küçük ve en büyük değerlerini bulunuz.
a) 1  2cos
b)  3  4sin 
c) 2  3cos
Çözüm:
a)  1  cos  1  2  2cos  2  1  2  1  2cos   1  2
 1  1  2cos  3 o halde ifadenin alacağı en küçük değer -1 ve
en büyük değer 3 olacaktır.
b)  1  sin   1  4  4sin   4  3  4  3  4sin   3  4
 7  3  4sin   1 o halde ifadenin alacağı en küçük değer -7 ve
en büyük değer 1 olacaktır.
c)  1  cos  1  3  3cos  3  2  3  2  3cos   3  2
 1  2  3cos  5 o halde ifadenin alacağı en küçük değer -1 ve
en büyük değer 5 olacaktır.
Örnek:
AĢağıdaki ifadelerin değerlerini bulunuz.
71
145
a) cos
 3sin
b) sin 26100  cos37800
2
2
130
Çözüm:
a) cos
71
145
1
1


 3sin
 cos  35     3sin  72   
2
2
2
2



1
3
1
 1 



 cos  34  1      3sin  72     cos  34     3sin  72   
2
2
2
 2 




3 

3



 cos  34 
 3sin  0  3  3
  3sin  72    cos
2 
2
2
2


b) sin 26100  cos37800  sin  7  3600  900    cos 10  3600  1800 
 sin 900  cos1800  1  1  0
2. TANJANT VE COTANJANT FONKSĠYONLARI
cot 
Cotanjant ekseni
y 1
K
T
tan 

O
1 br
Tanjant ekseni
x 1
Birim çembere x  1 ve y  1 doğrularını çizelim. Ölçüsü  olan açının bitim kenarları
bu doğruları T ve K noktalarında kessin. T notasının ordinatına  nın tanjantı denir ve
tan  ile gösterilir. x  1 doğrusu tanjanat eksenidir.
K noktasının apsisine  nın cotanjantı denir ve cot  ile gösterilir. y  1
doğrusuna kotanjant ekseni denir.
131
cot 
Cotanjant ekseni
y 1
K
T
 
tan 

O
1 br
Tanjant ekseni
x 1
Yukarıdaki Ģekilde görüleceği gibi ,
tan      tan 
ve cot      cot 
dir.
tan  nın bölgelere göre iĢaretinin nasıl değiĢtiğine bakalım.



 
2

x 1
I. Bölgede

0  
2
tan   0
x 1
x 1
II. Bölgede

 
2
tan   0
III. Bölgede
3
  
2
tan   0
3
2
x 1
IV. Bölgede
3
   2
2
tan   0
cot  nın da bölgelere göre nasıl iĢaretinin değiĢtiği ödev olsun.
132
11. BÖLÜM
KOMPLEKS SAYILAR
Tanım:
de 1 sayısı olmadığı için x 2  1  0 denkleminin çözümü yoktur.
Bu durumda reel sayılar kümesini geniĢletmek ihtiyacı doğmuĢtur.
Şimdi , i 2  1  i  1 imajiner sayısını alalım.
 i, latincede hayali,sanal anlamına gelen imaginarius kelimesinin ilk harfidir. 
x 2  1  0 denklemi x 2  1  x 
O halde,
Bu küme,
1 
i olarak bulunur.
den daha geniĢ olmak üzere bir sayı kümesi tanımlayabiliriz.
 z  a  ib / a, b  , i  1 dir.


Biz artık bu kümeye kompleks sayılar kümesi diyeceğiz.
z
i
a
b
imajiner kısım
reel kısım
z  a  ib ifadesinde b  0 ise z  a olacaktır. O halde,  diyebiliriz. Yani,
Kompleks sayılar kümesi reel sayılar kümesini kapsar. Yani, reel sayılar kümesi
kompleks sayılar kümesinin bir alt kümesidir.
Kompleks sayılara örnek olarak,
3
2  3i,  5  i ,3  4i ,  2  i ,  i , 2.4  0.01i sayılarını verebiliriz.
2
2
3
i  1 , i  i  i 2  i   1  i , i 4  i 2  i 2   1   1  1 dir
Bu durumu genelleştirelim. k 
i
4k
1 , i
4 k 1
i , i
4k  2
olsun
 1 , i 4 k  3  i dir.
Örnek:
AĢağıdaki sayıları hesaplayınız.
i 41  ? , i98  ? , i 79  ? , i3007  ?
Çözüm:
i 41  i 410 1  i
i 98  i 424  2  1
i 79  i 416  3  i
i 3007  i 3004  3  i 3004  i 3  i
KOMPLEKS SAYILARDA DÖRT ĠġLEM
1. Toplama iĢlemi
z1  a  ib , z2  c  id
olsun.
z1  z2   a  ib    c  id    a  c   i  b  d 
2. Çıkarma iĢlemi
133
z1  a  ib , z2  c  id
olsun.
z1  z2   a  ib    c  id    a  c   i  b  d 
3. Çarpma iĢlemi
z1  a  ib , z2  c  id
olsun.
z1  z2   a  ib    c  id    ac  bd   i  ad  bc 
4. Bölme iĢlemi
z1  a  ib , z2  c  id
olsun.
z1  a  ib   ac  bd   bc  ad 


i 2
dir.
z2  c  id   c 2  d 2 
c  d 2 
Örnek:
z1  3  2i , z2  1  4i
olsun.
z1  z2   3  2i   1  4i    3  1  i  2  4   4  2i
z1  z2   3  2i   1  4i    3  1  i  2  4   2  6i
z1  z2   3  2i   1  4i    3 1  2   4    i  3   4   2 1   3  8   i  12  2 
 11  10i
z1  3  2i   3 1  2  4    2 1  3  4    3  8  2  12 5 14 5  14i

 2
i 2

i

i 
2
2
z2 1  4i 
17
17
17 17
17
1   4 
1   4 
KOMPLEKS SAYILARIN KARTEZYEN DÜZLEMDE GÖSTERĠLMESĠ
z  a  ib kompleks sayısında a sayısına z 'nin reel kısmı,
b sayısına z 'nin imajiner kısmı denir. ( z )  a ve im( z )  b
şeklinde gösterilir. O halde, z  a  ib sayısını  a, b  şeklinde yazabiliriz.
Her  a, b  sıralı ikilisi bildiğimiz gibi xy  düzleminde bir noktya karşılık
gelmektedir. Bu  a, b  sıralı iklisinin  0, 0  noktasına olan uzaklığına
z  a  ib sayısının modülü veya mutlâk değeri denir.
z nin modülü |z| ile gösterilir. |z|= a 2  b 2 dir.
134
Ġmajiner eksen
 a, b 
|z|
 0, 0 
| z || a  ib | a 2  b2  0
Reel eksen
ĠKĠ KOMPLEKS SAYININ EġĠTLĠĞĠ
z1  a  ib , z2  c  id olsun. a  c ve b  d ise bu iki kompleks
sayıya birbirine eĢittir denir. Yani, a  ib  c  id  z1  z2 dir.
Örnek:
z1  a  2i , z2  4  bi sayılarının eĢit olması için a, b sayıları ne olmalıdır?
Çözüm:
z1  a  2i  z2  4  bi  a  4 ve b  2 olmalıdır.
Örnek:
z1  x   x  2 y  i , z2   2 x  y  1  4i kompleks sayılarının eĢit olması için x, y nedir?
Çözüm:
z1  z2  x   x  2 y  i   2 x  y  1  4i  x   2 x  y  1 ve  x  2 y   4 olmalıdır.
x   2 x  y  1 
x  y  1 
x2

dir.


 x  2 y   4  x  2 y  4 y  3
Örnek.
z1  a  i 44 , z2  i  b  i 53  kompleks sayıları eĢit olduğuna göre, a  b
toplamını bulunuz.
Çözüm:
z1  a  i 44  a  i 411  a  1
z2  i  b  i 53   i  b  i 52 1   i  b  i   ib  1
z1  z2  a  1  ib  1  a  1  1 ve ib  0
a  2 ve b  0 olacağından a  b  2 olur.
BĠR KOMPLEKS SAYININ EġLENĠĞĠ
135
a  ib sayısına z  a  ib sayısının eĢleniği denir. z ile gösterilir.
Buna göre, z  z   a  ib  a  ib   a 2  b 2 | z |2 olduğundan
bir kompleks sayının modülü | z | z  z ilede bulunabilir.
Örnek.
z  3  2i sayısının eĢleniğini bularak düzlemde gösteriniz.
Çözüm:
z  3  2i
2
3
-2
z  3  2i
Örnek.
AĢağıda verilen kompleks sayıların eĢleniklerini bulunuz.
3
5  4i ,  3  i , 7  i , i , 1  i
4
Çözüm:
5  4i  5  4i
3  i   3  i
3
3
7 i 7 i
4
4
i  i
1 i 1 i
NoT: Bir kompleks sayı ile onun eĢleniği x  eksenine göre birbirinin simetriğidir
Örnek:
3x   x  y  i kompleks sayısının eĢleniği 2 x  y  4i olduğuna göre
x, y reel sayılarını bulunuz.
Çözüm:
3x   x  y  i sayısının eĢleniği 3x   x  y  i olacağından,
3x   x  y  i  x  y  4i  3x  x  y ve -  x  y   4
2x  y  0 
  x  1 ve y  6 bulunur.
2 x  y  4
Örnek:
z  3  4i sayısının modülünü bulunuz.
Çözüm:
136
z  z   3  4i  3  4i   32  42  9  16  25
| z | z | z |  25  5 elde edilir.
Örnek:
z  1  i sayısının modülünü bulunuz.
Çözüm:
2
z  z   1  i  1  i    1  12  1  1  2
| z | z | z |  2 elde edilir.
EġLENĠK ĠġLEMĠNĠN ÖZELLĠKLERĠ
1  z1  z2  z1  z2
2  z1  z2  z1  z2
z  z
3  1   1
 z2  z2
4  z  z | z |2
dir.
Bir ikinci derece denkleminde   0 olması halinde denklemin kökleri birbirinin
eĢleniği olan iki kompleks köktür. Buna göre kökleri z  a  ib ve z  a  ib olan
denklemde,  x  z  x  z  0 veya x 2  z  z x  z  z  0 dır.




Örnek:
Kompleks düzlemde bir nokta z olduğuna göre
a) | z | 1
b) z  z  i ifadelerini geometrik olarak gösteriniz
Çözüm:
a) | z | 1  x 2  y 2  1
| z | 1
 x 2  y 2  1 olacaktır.
Yani, yarı çapı 1 br olan
çemberin iç bölgesidir.
z  z  i   x  iy    x  iy   i
x  iy  x  iy  i  2 y  i  i
1
y
1
2
1
1
dir. Yani, y  doğrusudur.
2
2
BĠR KOMPLEKS SAYININ TRĠGONOMETRĠK GÖSTERĠMĠ
y
A  a, b    r cos  , r sin  
137
Şekildeki OA doğrusunun x  ekseni ile pozitif
yönde yaptığı açının ölçüsü  olmak
b
b  r sin 
üzere, A noktası  r ,   koordinatları
r
yardımıyla bulunabilir. A  a, b  noktasına
a  ib kompleks sayısı karĢılık geldiğine
göre bu kompleks sayı  r ,   ile ifade edilebilir.

O
a  r cos 
a
a
a
b
b
, sin   
olmak üzere,

2
2
2
r
r
a b
a  b2


a
b
z  a  ib  r 
i
  r  cos   sin   dir.
2
2
a 2  b2 
 a b
Burada , r sayısı z  a  ib kompleks sayısının modülü ve  yede kompleks sayının
argümenti denir. arg z ile gösterilir.
r  a 2  b2 , cos  
Örnek:
z  1  i kompleks sayısının modülünü ve argümentini bulunuz.
Çözüm:
r  a 2  b 2  12  12  1  1  2 olduğundan
1 
 1
z  1 i  2 
i
 şeklinde yazılabilir.
2
 2
1
1
Buna göre, cos  
ve sin  
olduğundan
2
2


4
olur. O halde,



z  1  i  2  cos  i sin  şeklinde yazılabilir.
4
4

Modul  z   2 ve arg  z  

4
dür.
Örnek:
z1  r1  cos 1  i sin 1  ve z2  r2  cos 2  i sin 2  ise
z1  z2  r1r2  cos 1 cos 2  sin 1 sin 2   i  sin 1 cos 2  sin 2 cos 1 
 r1r2 cos 1  2   i sin 1  2   elde edilir.
Burada çıkaracağımız sonuç, iki kompleks sayının çarpımında modüller çarpılmakta ve
argümentler toplanmaktadır. Kısaca,
z1  z2  z1  z2 ve arg  z1  z2   arg z1  arg z2 dir.
BĠR KOMPLEKS SAYININ KUVVETĠ
138
z1  r1  cos 1  i sin 1  ve z2  r2  cos 2  i sin 2  ise
z1  z2  r1r2 cos 1  2   i sin 1  2  
Eğer, özel olarak
z1  z2  z  r  cos   i sin   olrak alırsak
z  z  r  r  cos 1  2   i sin 1   2    r 2  cos 2  i sin 2  elde edilir.
Bu işleme aynı Ģekilde devam edilirse,
z n  r n  cos n  i sin n  bulunur. O halde,
 r  cos   i sin     r n  cos n  i sin n  dir.
n
r  1 olduğunda Moivre formülü denilen  cos   i sin      cos n  i sin n 
bağıntısı elde edilir. Bu formül daha çok cos n ve sin n ifadelerini cos  ve sin 
n
cinsinde ifade etmek için kullanılır.
Örnek:
cos3 ifadesini cos  cinsinden yazınız.
Çözüm:
Moivre formülünü kullanalım.  cos   i sin      cos3  i sin 3 
cos3   3i cos 2  sin   3cos  sin 2   i sin 3    cos3  i sin 3  iki kompleks sayının
3
eşitlğinden dolayı,
cos3  cos3   3cos  sin 2 
cos3  cos3   3cos  1  cos 2  
cos3  cos3   3cos   3cos3 
cos3  4cos3   3cos  olarak bulunur.
Örnek:
1  i 3 
6
sayısını a  ib Ģeklinde yazınız.
Çözüm:
z  1  i 3  r  12 
 3 
2
1 3  2
1
3
z  2   i
 dir.
2 
2
1
3

ve sin  
olduğundan   tür.
2
2
3



z  2  cos  i sin  olur.
3
3

Burada, cos  
6
 

 



z  1  i 3   2  cos  i sin    26   cos 6   i sin 6  
3
3 
3
3

 
z 6  64  cos 2  i sin 2   64 tür.
6


6
139
BĠR KOMPLEKS SAYININ n. KUVVETTEN KÖKÜ
z  r  cos   i sin   kompleks sayısının n. kuvvetten kökü,
n
 1
n
z  q  cos  i sin   ise  z n    q  cos  i sin    olarak yazalım.
 
1
n
q n   cos  i sin    r  cos   i sin  
n
q n   cos n  i sin n   r  cos   i sin   iki kompleks sayının eĢitliği tanımından
1
q n  r ve n    k  2 olacaktır. O halde,
1
1
  k  2
  k  2 

 i sin
 r  cos   i sin    n =r n  cos
 k  0,1, 2,3...
n
n


elde edilir.
Örnek:
z 3  1 denklemini çözünüz.
Çözüm.
z 3  1  1   cos 0  i sin 0  olarak yazabiliriz.
k  2
k  2 

z k   cos
 i sin

n
n 

0  2
0  2 

z 0   cos
 i sin
   cos 0  i sin 0   1
3
3 

1  2
1  2

z1   cos
 i sin
3
3

2
2
 
 i sin
   cos
3
3
 
1
3

 i
2
2

2  2
2  2  
4
4

z k   cos
 i sin
 i sin
   cos
3
3  
3
3

1
3
1
3 

Ç  1,   i
,  i
 dir.
2
2
2
2 

Örnek:
z 3  i  0 denklemini çözünüz.
1
3

   i
2
2

Çözüm.
140



z 3  i  0  z 3  i  1   cos  i sin  olarak yazalım.
2
2




2 k 
2 k 

2  i sin
2
z k  1  cos
3
3


1
3
k  0 için z0  cos

6
 i sin

6



 k  0,1, 2


3
1
i
2
2
5
5
3
1
 i sin

i
6
6
2
2
9
9
k  2 için z2  cos
 i sin
 i olarak bulunur.
6
6
1
3
1
 3

Ç
 i ,
 i ,  i  dir.
2
2
2
 2

k  1 için z1  cos
141
Download